SCIENCES BAC PRO

SCIENCES PHYSIQUES ETCHIMIQUES Livre du professeur 1 re etT erm PROFESSIONNELLES BAC PRO J-P. Durandeau J-L. Berducou J-C. Larrieu-Lacoste C. Mazeyrie...

139 downloads 1045 Views 7MB Size
1T

re erm et

PROFESSIONNELLES

SCIENCES

BAC PRO

PHYSIQUES ET CHIMIQUES Livre du professeur

J-P. Durandeau J-L. Berducou J-C. Larrieu-Lacoste C. Mazeyrie C. Raynal J-C. Trillaud

Table des matières   1 Comment protéger un véhicule contre la corrosion ? T3 . . . . . . . . . . . . . . . . . . . . . . . . . . . . . . . . . . . . 6   2 Pourquoi éteindre ses phares quand le moteur est arrêté ? T4 . . . . . . . . . . . . . . . . . . . . . . . . . . . . . . . . 9   3 Comment se déplacer dans un fluide ? T5 . . . . . . . . . . . . . . . . . . . . . . . . . . . . . . . . . . . . . . . . . . . . . . . . 13   4 Qu’est-ce qu’une voiture puissante ? T6 . . . . . . . . . . . . . . . . . . . . . . . . . . . . . . . . . . . . . . . . . . . . . . . . . . 17   5 Comment avoir une bonne tenue de route ? T7 . . . . . . . . . . . . . . . . . . . . . . . . . . . . . . . . . . . . . . . . . . . . . . 20   6 Comment faire varier la vitesse d’un véhicule électrique ? T8 . . . . . . . . . . . . . . . . . . . . . . . . . . . . . . . 24   7 Pourquoi le métal semble-t-il plus froid que le bois ? CME4 . . . . . . . . . . . . . . . . . . . . . . . . . . . . . . . . 28   8 Comment se chauffer ? CME4 . . . . . . . . . . . . . . . . . . . . . . . . . . . . . . . . . . . . . . . . . . . . . . . . . . . . . . . . . . . 31   9 Comment économiser l’énergie ? CME5 . . . . . . . . . . . . . . . . . . . . . . . . . . . . . . . . . . . . . . . . . . . . . . . . . . 35 10 Comment protéger l’environnement ? CME5 . . . . . . . . . . . . . . . . . . . . . . . . . . . . . . . . . . . . . . . . . . . . . . . . 39 11 Comment fonctionnent certains dispositifs de chauffage ? CME6 . . . . . . . . . . . . . . . . . . . . . . . . . . . 42 12 Comment fonctionnent un transformateur et une plaque à induction ? CME6 et CME7 . . . . . . . 46 13 À quoi correspondent les bornes d’une prise de courant ? CME7 . . . . . . . . . . . . . . . . . . . . . . . . . . . . 50 14 Comment calcule-t-on la puissance consommée par un appareil monophasé ? CME7 . . . . . . 54 15 Comment peut-on améliorer sa vision ? HS4 et SL4 . . . . . . . . . . . . . . . . . . . . . . . . . . . . . . . . . . . . . . . 57 16 Comment faut-il se protéger des rayons du Soleil ? HS4 . . . . . . . . . . . . . . . . . . . . . . . . . . . . . . . . . . . . 61 17 Quels sont les principaux constituants du lait ? HS5 . . . . . . . . . . . . . . . . . . . . . . . . . . . . . . . . . . . . . . . 64 18 Comment fabriquer un détergent ? Quel est son rôle ? HS6 . . . . . . . . . . . . . . . . . . . . . . . . . . . . . . . . . 70 19 Comment fabriquer un arôme ou un parfum ? HS6 . . . . . . . . . . . . . . . . . . . . . . . . . . . . . . . . . . . . . . . . 73 20 Comment dévier la lumière ? SL1 . . . . . . . . . . . . . . . . . . . . . . . . . . . . . . . . . . . . . . . . . . . . . . . . . . . . . . . . 77 21 Comment transmettre un son ? SL2/SL3 . . . . . . . . . . . . . . . . . . . . . . . . . . . . . . . . . . . . . . . . . . . . . . . . . 80 22 Pourquoi les objets sont-ils colorés ? SL5 . . . . . . . . . . . . . . . . . . . . . . . . . . . . . . . . . . . . . . . . . . . . . . . . . 83 23 Comment fonctionne un haut-parleur ? SL6 . . . . . . . . . . . . . . . . . . . . . . . . . . . . . . . . . . . . . . . . . . . . . . 86 24 Comment fonctionne un microphone ? SL6 . . . . . . . . . . . . . . . . . . . . . . . . . . . . . . . . . . . . . . . . . . . . . . 89 25 Comment une image est-elle captée par un système d’imagerie numérique ? SL7 . . . . . . . . . . . . 93

Couverture : npeg.fr Maquette et mise en page : Nicolas Balbo

© HACHETTE LIVRE 2010, 43, quai de Grenelle, 75905 Paris Cedex 15 ISBN 978-2-01-181111-0 www.hachette-education.com Tous droits de traduction, de reproduction et d’adaptation réservés pour tous pays. Le Code de la propriété intellectuelle n’autorisant, aux termes de l’article L. 122-4 et L. 122-5, d’une part, que les « copies ou reproductions strictement réservées à l’usage privé du copiste et non destinées à une utilisation collective », et, d’autre part, que « les analyses et courtes citations » dans un but d’exemple et d’illustration, « toute représentation ou reproduction intégrale ou partielle, faite sans le consentement de l’auteur ou de ses ayants cause, est illicite ». Cette représentation ou reproduction, par quelque procédé que ce soit, sans autorisation de l’éditeur ou du centre français de l’exploitation du droit de copie (20, rue des Grands-Augustins, 75006 Paris), constituerait donc une contrefaçon sanctionnée par les Articles 425 et suivants du Code pénal.

Avant-propos Le livre du professeur, qui accompagne le manuel de l’élève, comporte les corrigés des activités et des exercices proposés dans chaque chapitre.

Les activités dans le manuel Nous n’avons pas choisi une méthode déductive fondée sur la présentation d’un cours préalable suivi d’exercices d’application, car le dogmatisme qu’elle peut engendrer nous paraît inadapté dans des classes de 1re et Terminales Professionnelles. Nous avons opté au contraire pour une démarche inductive s’appuyant sur des activités, le plus souvent expérimentales, afin de résoudre chaque fois un problème scientifique. Cela permet de mieux impliquer l’élève dans la construction de ses connaissances, le professeur l’accompagnant, plutôt que le précédant. Le cours (l’essentiel) vient ensuite synthétiser les connaissances et apporter les compléments nécessaires sur des sujets qui n’ont pu être abordés dans les activités. Nous avons formulé, dans le manuel, des questions pour chaque activité ; l’élève, en répondant à ces questions, propose des conclusions à la suite de ses observations et mesures. Le professeur peut alors aider à reformuler les résultats qui figurent dans le cours présent dans le manuel. Ces questions jouant un rôle très important dans la démarche, nous proposons les réponses dans le livre du professeur.

Les exercices Dans le livre du professeur, nous fournissons les solutions des exercices figurant dans les cinq rubriques du manuel de l’élève.

• Tester ses connaissances Cette forme de présentation et d’évaluation (Q.C.M.) se répand de plus en plus et présente l’avantage de développer l’esprit critique. Cette rubrique permet à chaque élève de tester rapidement les connaissances fondamentales du chapitre.

• Tester ses capacités Ces exercices simples traitent des capacités énoncées dans le référentiel. Chacun concernant une seule capacité, l’élève peut en vérifier plus facilement l­’acquisition.

• Appliquer le cours L’objectif recherché est maintenant plus ambitieux : il s’agit de vérifier l’aptitude des élèves à appliquer les lois du programme dans un contexte proche de celui des activités et de l’essentiel du cours du chapitre. La résolution de ces exercices nécessite l’acquisition de savoir-faire dans les domaines théorique et expérimental.

• Utiliser ses connaissances Les exercices concernés relèvent d’un niveau de difficulté supérieur : il s’agit d’appliquer ses connaissances et ses savoir-faire dans des contextes différents de ceux des activités et de l’essentiel du cours, en particulier dans la vie professionnelle. Nous avons intégré dans cette rubrique des exercices dont le thème est le « développement durable ».

• Situation problème Ces exercices demandent une plus grande autonomie et davantage de recherche : le travail en groupes se prête particulièrement à la résolution de ces exercices. Le livre du professeur constitue un outil indissociable du manuel, au service des collègues pour le bénéfice de leurs élèves. Nous acceptons bien volontiers leurs suggestions et critiques.

Pour aider l’enseignant dans sa tâche, le Livre numérique permet de projeter en permanence tous les documents nécessaires à la mise en œuvre et à l’exploitation du manuel de l’élève.

3

Programme et modules spécifiques 1. Programme des classes de 1re et Term professionnelles 1.1 Tronc commun LES TRANSPORTS (T)

CONFORT DANS LA MAISON ET L’ENTREPRISE (CME)

HYGIÈNE ET SANTÉ (HS)

SON ET LUMIÈRE (SL)

T3 Comment protéger un véhicule contre la corrosion ?

CME 4 Comment chauffer ou se chauffer?

SL 1 Comment dévier la lumière ?

T4 Pourquoi éteindre ses phares quand le moteur est arrêté ?

CME 5 Peut-on concilier confort et développement durable ?

SL 2 Comment un son se propage-t-il ?

T5 Comment se déplacer dans un fluide ?

SL 3 Comment transmettre un son à la vitesse de la lumière ? HS 4** Comment peut-on améliorer sa vision ?

SL 4** Comment voir ce qui est faiblement visible à l’œil nu ?

** Les premières parties de ces modules développent les mêmes capacités et connaissances ; le professeur traitera l’une ou l’autre au choix.

1.2 Modules spécifiques LES TRANSPORTS (T)

HYGIÈNE ET SANTÉ (HS)

SON ET LUMIÈRE (SL)

T6 Qu’est-ce qu’une voiture puissante ?

CME 6 Comment fonctionnent certains dispositifs de chauffage ?

HS 5 Quels sont les constituants du lait ?

SL 5 Pourquoi les objets sont-ils colorés?

T7 Comment avoir une bonne tenue de route ?

CME 7 Comment l’énergie électrique est-elle distribuée dans l’entreprise ?

HS 6 Quels sont le rôle et les effets d’un détergent ?

SL 6 Comment reproduire un signal sonore ?

T8 Comment faire varier la vitesse d’un véhicule électrique ?

4

CONFORT DANS LA MAISON ET L’ENTREPRISE (CME)

SL 7 Comment une image est-elle captée par un système d’imagerie numérique ?

2. R  épartition des modules spécifiques en fonction des spécialités de baccalauréat professionnel Baccalauréats professionnels Artisanat et Métiers d’Art Communication graphique – Artisanat et Métiers d’Art Marchandisage visuel – Artisanat et Métiers d’Art Métiers de l’enseigne et de la signalétique – Électrotechnique énergie équipements communicants – Micro–Informatique et réseaux : installation et maintenance – Microtechniques – Photographie – Production graphique – Production imprimée – Systèmes électroniques numériques. Aéronautique Mécanicien systèmes-avionique– Aéronautique Mécaniciens systèmes-cellule – Artisanat et Métiers d’Art Horlogerie – Maintenance de véhicules automobiles Voitures particulières – Maintenance de véhicules automobiles Véhicules industriels – Maintenance de véhicules automobiles Motocycles – Maintenance nautique – Maintenance des systèmes mécaniques automatisés Systèmes ferroviaires – Productique mécanique Décolletage – Technicien aérostructure – Technicien d’usinage. Artisanat et Métiers d’Art Arts de la pierre – Artisanat et Métiers d’Art Ebéniste – Artisanat et Métiers d’Art Tapissier d’ameublement – Artisanat et Métiers d’Art Vêtement et accessoires de mode – Aménagement et finition du bâtiment – Carrosserie Construction – Métiers de la mode et industries connexes Productique – Mise en œuvre des matériaux Industries textiles – Mise en œuvre des matériaux Matériaux céramiques – Mise en œuvre des matériaux Matériaux métalliques moulés – Plasturgie – Technicien d’études du bâtiment Études et économie – Technicien d’études du bâtiment Assistant en architecture – Technicien géomètretopographe – Réparation des carrosseries. Environnement nucléaire – Étude et définition de produits industriels – Industrie des pâtes, papiers et cartons – Maintenance des équipements industriels – Maintenance des matériels agricoles – Maintenance des matériels Travaux publics et manutention – Maintenance des matériels Parcs et jardins – Technicien de maintenance des systèmes énergétiques et climatiques – Technicien du froid et du conditionnement de l’air – Technicien en installation des systèmes énergétiques et climatiques. Interventions sur le patrimoine bâti – Ouvrages du bâtiment : aluminium, verre et matériaux de synthèse – Ouvrages du bâtiment : métallerie – Pilotage de systèmes de production automatisée – Réalisation d’ouvrages chaudronnés et de structures métalliques – Technicien constructeur bois – Technicien de fabrication bois et matériaux associés – Technicien de scierie – Technicien du bâtiment organisation et réalisation du gros œuvre – Technicien menuisier-agenceur – Technicien modeleur – Technicien outilleur – Travaux publics. Artisanat et Métiers d’Art Métiers des techniques du verre – Bio-industries de transformation – Esthétique cosmétique parfumerie – Hygiène et environnement – Industries de procédés – Métiers du pressing et de la blanchisserie – Traitements de surface.

Modules spécifiques

SL5 – SL6 – SL7

T6 – T7 – T8

CME6 – CME7 – SL5

T8 – CME6 – CME7

T6 – T7 – CME7

HS5 – HS6

5

1 Comment protéger un véhicule contre la corrosion ? T3

Les activités ■■Activité 1 Quels facteurs favorisent la corrosion du fer ? • Matériel  Un porte-tubes et 6 tubes à essai

• Produits  Des clous en acier Une solution de sulfate de cuivre Réactif : solution de soude Eau, eau bouillie, huile, sel

Expérience 1 Les tubes à essai contenant les clous doivent être préparés par les élèves lors de la séance précédente.

Expérience 2 La réaction chimique est facile à observer, tant avec la paille de fer qu’avec la poudre de fer. On peut transvaser la solution restante (elle a perdu sa couleur bleue) dans un autre tube à essai pour effectuer la recherche des ions Fe2+.

• Réponse aux questions Expérience 1

1. Le dioxygène dissous dans l’eau facilite la corrosion : dans le tube (d), l’eau est privée de dioxygène dissous et le clou reste intact.

2. L’eau est un autre facteur favorisant la corrosion du fer : le clou est attaqué dans les tubes (b) et (c) contenant de l’eau, mais pas dans le tube (a) privé d’humidité.

3. La corrosion la plus forte se fait dans le tube (c) contenant du sel (des ions chlorure). Expérience 2

4. Du cuivre métallique se dépose sur le fer. 5. Réaction : Cu2+ + 2 e– → Cu 6. Le précipité verdâtre montre l’apparition d’ions fer II Fe2+ dans la solution. 7. Réaction : Fe → Fe2+ + 2 e–

■■Activité 2 Comment réaliser la protection du fer contre la corrosion ? • Matériel  Deux béchers de 250 mL Des électrodes de cuivre (1), de fer (2) et de zinc (1) Des fils de connexion  Un multimètre électronique  Une boîte de Petri

• Produits  Un clou peint, un clou entouré de cuivre, un clou entouré de zinc et un clou entouré de magnésium De l’agar-agar Réactifs : phénolphtaléine et solution d’hexacyanoferrate III de potassium

6 • CHAPITRE 1 - Comment protéger un véhicule contre la corrosion ?

Expérience 1 L’expérience se prolonge pendant toute la séance. Ne pas remuer les béchers. On peut rajouter les réactifs au niveau des électrodes pour observer l’apparition des couleurs : couleur rose prise par la phénolphtaléine autour de l’électrode positive (cathode) ; bleue prise par l’hexacyanoferrate autour de l’électrode négative (anode) de fer dans le premier bécher uniquement.

Expérience 2 À réaliser quelques jours à l’avance. On peut observer l’apparition des couleurs au bout de quelques minutes.

• Réponse aux questions  Expérience 1

1. Pile 1 : électrode (+) : le cuivre, électrode (–) : le fer. Pile 2 : électrode (+) : le fer, électrode (–) : le zinc.

2. La coloration rose (milieu devenu basique) apparaît autour des électrodes (+). La coloration bleue (apparition d’ions Fe2+) apparaît autour de l’électrode (–) de fer, dans le premier bécher seulement.

3. Dans la pile 2, le fer s’est oxydé à l’état d’ions fer II. L’électrode de fer constitue la borne (–) de la pile, donc son anode (borne d’entrée du courant).

Expérience 2

4. le clou est oxydé quand il est au contact du cuivre. 5. On peut protéger un objet en acier en le peignant, en le plaçant au contact de zinc ou de magnésium.

6. Des blocs de zinc protègent la coque en acier et les hélices en bronze des bateaux : il faut les remplacer régulièrement, car ces blocs sont corrodés.

Les documents ■■La peinture à l’électricité • Réponse aux questions 1. Le circuit électrique de la cataphorèse est représenté ci-contre.

2. 50 µm = 0,050 mm, soit millimètre.

1/20e

de

3. La polymérisation est une réaction chimique entre des molécules qui se mettent bout à bout pour former une gigantesque molécule appelé « macromolécule » ou « polymère ». Une macromolécule peut être constituée de plusieurs centaines ou milliers de molécules de base.

200 V à 350 V

+ –

anode

+

+ +

cathode

Les exercices Tester ses connaissances

Tester ses capacités

Q.C.M. 1 : A et B 4 : A et C

1. Étude sur la corrosion d’un clou a. Le produit desséchant élimine l’humidité. b. Le clou ne s’oxyde pas, car il n’y a pas d’humidité.

2 : A et C 5 : B

3 : B et C 6 : A et C.

CHAPITRE 1 - Comment protéger un véhicule contre la corrosion ? • 7

+

c. Au niveau de la surface de l’eau, l’air est saturé en vapeur d’eau. d. La présence d’eau favorise la corrosion. e. En faisant bouillir l’eau, on élimine le dioxygène dissous dans l’eau. f. La couche d’huile empêche le dioxygène de se redissoudre dans l’eau. g. Le clou n’est pas attaqué, car il n’y a plus de dioxygène dissous. h. Les ions chlorure Cl– favorisent l’oxydation du fer.

Ag+ Ag+

3. Couple rédox a. L’oxydant Cu2+ est l’oxydant le plus puissant et

Pièce à argenter

Électrode d’argent

2. Une réaction avec le fer Le réducteur est le fer Fe et l’oxydant les ions cuivre II Cu2+.



Ag+

b. À l’anode, l’électrode d’argent s’oxyde : Ag → Ag+ + e– À la cathode, sur la pièce à recouvrir, les ions argent se réduisent et de l’argent se dépose: Ag+ + e– → Ag

b. L’ion Fe2+ ne peut pas oxyder le nickel, car il n’y a

8. Assemblage a. Il y a corrosion en a. et en b. (faible zone de corro-

pas de dépôt de fer sur la lame de nickel. c. Le classement des couples rédox :

b. Schémas montrant les zones d’oxydation :

oxyde le nickel.

sion).

a)

Cu2+

Cu

Ni2+

Ni

Fe2+

Fe

Mg2+

Mg

d. L’ion Fe2+ oxyde le magnésium ; le couple Fe2+/Fe est donc situé au-dessus du couple Mg2+/Mg .

4. Prévoir une oxydoréduction a. On observe une réaction d’oxydoréduction dans le bécher (b) : l’ion Ag+ oxyde le cuivre Cu. b. Sont en jeu les couples Ag+/Ag et Cu2+/Cu.

5. Demi-équations a. Oxydation du zinc : Zn → Zn2+ + 2 e–. b. Réduction de l’ion nickel : Ni2+ + 2 e– → Ni. c. Réduction de l’ion argent : Ag+ + e– → Ag. d. Oxydation du cuivre : Cu → Cu2+ + 2 e–. 6. Équation d’oxydoréduction a. On doit multiplier ces deux demi-équations, res-

b)

zones de corrosion

9. Fissures a. L’acier est protégé par le zinc. b. Le zinc est oxydé, car le couple Zn2+/Zn est situé plus bas que le couple Fe2+/Fe dans l’échelle rédox (l’ion Fe2+ est plus oxydant que l’ion Zn2+). c. L’acier (fer) est oxydé, car le couple Sn2+/Sn est situé au-dessus du couple Fe2+/Fe : Sn2+ est un oxydant plus puissant que Fe2+.

10. La protection cathodique a. Pour protéger la coque en acier et l’hélice en bronze, on plaque des électrodes de zinc sur la coque.

b. Schéma électrique de la protection cathodique :

électrons

pectivement, par n = 3 et n’= 2. b. 6 électrons interviennent dans chaque demiéquation. c. Équation globale: 3 Cu2+ + 2 Al → 3 Cu + 2 Al3+ .

courant

G = Réservoir enterré à protéger Anode Sol

Appliquer le cours 7. À vous de résoudre a. Schéma électrique de la réaction d’argenture :

– +

Pour éviter la corrosion du réservoir, il faut « forcer »

8 • CHAPITRE 1 - Comment protéger un véhicule contre la corrosion ?

les électrons à aller vers le réservoir, d’où la polarité du générateur.

Exercices à caractère professionnel 11. Cataphorèse a. L’anode est reliée à la borne (+) du générateur, la cathode à la borne (–).

b. Des anodes sont reliées à la borne (+) du générateur et la pièce à traiter à la borne (–).

c. Les particules de peinture (+) migrent vers la cathode (–) constituée par la pièce à peindre.

12. Ossature métallique a. La demi-équation du couple Fe2+/Fe :  Fe2+ + 2e–    Fe

b. La demi-équation du couple H+/H :  c. Le couple Fe2+/Fe, l’ion

2H+ + 2e–    H2 + H /H2 étant placé au-dessus du couple H+ oxyde le fer et l’équation globale est : Fe + 2H+ → Fe2+ + H2.

13. Argenture d’un gnomon a. L’oxydant est l’ion Ag+, le réducteur le métal argent Ag.

b. Le dépôt d’argent correspondant à une réduction de l’ion argent : Ag+ + e– → Ag.

c. Volume d’argent : 132 × 0,01 = 1,32 soit 1,3 cm3. d. Masse d’argent : 10,5 × 1,3 = 13,65 g. e. La durée de l’électrolyse est : m 14 = ≈ 11667 s 0, 001.I 0, 001 × 1, 2 soit 3 h 15 min environ. ∆t =

14. Pour chercher des informations a. Luigi Galvani a travaillé sur l’électricité animale (cuisse de grenouille au contact d’un métal)  : on a nommé ce phénomène le galvanisme. La protection du fer par une couche de zinc a été appelée « galvanisation » en son honneur. b. L’armature en acier était recouverte de plaques de cuivre. L’ensemble constituait donc une pile de corrosion et l’effet était accentué par l’effet marin : humidité, sel... Les attaches et les rivets en acier ont cédé. c. Réparation : les armatures ont été remplacées par des barres d’acier inoxydable dont le potentiel était voisin de celui du cuivre et les rivets et attaches ont été isolés.

2 Pourquoi éteindre ses phares

quand le moteur est arrêté ? T4

Les activités ■■Activité 1 Quels sont les principes des piles et des accumulateurs ? Expérience 1

• Matériel Deux plaques de cuivre ; deux plaques de zinc - Des cordons ; pinces crocodile ; un milliampèremètre Un bécher contenant une solution d’acide sulfurique.

Expérience 2 • Matériel Deux plaques de plomb ; un milliampèremètre ; un commutateur Une lampe ; un générateur de tension continue ; des cordons

• Réponses aux questions  Expérience 1

1. Le courant électrique circule dans le circuit si deux électrodes de natures différentes (dissymétriques) plongent dans la solution d’acide sulfurique.

2. Une pile est constituée de deux électrodes métalliques différentes plongeant dans une solution ionique.

3. L’électrode positive est la lame de cuivre. CHAPITRE 2 - Pourquoi éteindre ses phares quand le moteur est arrêté ? • 9

Expérience 2

4. Le milliampèremètre indique la valeur zéro, car les deux électrodes sont de même nature. 5. Lorsque le générateur G débite, des réactions chimiques différentes transforment les électrodes pour créer entre elles une dissymétrie.

6. Ce changement est forcé par l’énergie électrique fournie par le générateur. 7. Un accumulateur joue le rôle d’un générateur en se comportant comme une pile au cours de sa décharge. Il se produit alors une réaction spontanée.

8. Une pile fonctionne spontanément parce qu’il y a une dissymétrie initiale entre deux électrodes plongeant dans deux solutions ioniques (une réaction irréversible se produit conduisant à l’usure de ces produits). Dans un accumulateur, la dissymétrie initiale n’existe pas. On la force en le chargeant. Lors de la décharge, il fonctionne comme une pile et la dissymétrie s’atténue. L’accumulateur ne fonctionne plus lorsque les électrodes redeviennent symétriques : il faut donc le recharger (ici, les réactions sont réversibles). Remarque : ce qui peut troubler l’élève, c’est qu’aujourd’hui les constructeurs vendent les batteries préalablement chargées afin de les rendre utilisables tout de suite : la phase initiale de l’opération n’est plus à faire par le consommateur.

■■Activité 2 Comment redresser une tension ? Expérience 1 - Comment réaliser un redressement monoalternance ?

• Matériel Un générateur ; un interrupteur ; une diode ; une résistance ; un oscilloscope ; des cordons.

Expérience 2 - Comment réaliser un redressement double alternance ?

• Matériel Un générateur ; un interrupteur ; quatre diodes ou un pont de Graëtz ; une résistance ; un oscilloscope ; des cordons.

• Réponses aux questions 1. La tension délivrée par le générateur est alternative. 2. La diode ne laisse passer le courant alternatif qu’une alternance sur deux : elle permet d’obtenir un courant unidirectionnel.

3. Ce schéma symétrique où deux diodes conduisent en même temps permet d’obtenir dans une charge résistive un courant unidirectionnel au cours des deux alternances. La période de la tension redressée est égale à la moitié de celle du générateur.

4. L’ajout d’un condensateur en dérivation avec la résistance permet de réduire l’ondulation de la tension de sortie : il lisse la tension. 5. Cet adaptateur est constitué d’un transformateur qui abaisse la tension du secteur ; d’un redresseur pour la rendre unidirectionnelle ; d’un condensateur pour la lisser et la rapprocher d’une tension continue.

UMH

6. Allure des courbes ci-contre.

10 • CHAPITRE 2 - Pourquoi éteindre ses phares quand le moteur est arrêté ?

UPQ

Les documents ■■Les piles à combustible Réponses aux questions 1. Équation globale : 2 H2 + O2 → 2 H2O 2. Le prix de revient est élevé, car les électrodes sont en platine (métal précieux et rare). 3. L’électrolyte permet la migration des ions H+ donc le passage du courant électrique. 4. Lorsque l’hydrogène est produit par des dérivés d’hydrocarbures, méthanol par exemple, il y a production de CO2 qui est un gaz à effet de serre.

Les exercices Tester ses connaissances Q.C.M. 1 : A et C  ;  2 : A, B et C  ;  3 : C  ;  4 : C  ; 5 : A.

Tester ses capacités

4. Pont de Graëtz a. Voir schéma suivant : 1 D 1

Y

D2

G

1. Pile au citron a. On a une pile, car il y a une réaction d’oxydoréduction spontanée due à la dissymétrie des couples redox Fe2+ / Fe et Cu2+ / Cu. b. La borne positive est l’électrode en cuivre (cathode), l’électrode en fer est la borne négative (anode) : c’est le fil de fer qui libère les électrons.

R

2 D4

D3

b. Les diodes D1 et D3 s’allument lorsque le courant va dans le sens 1.

c. Les diodes D2 et D4 s’allument lorsque le courant va dans le sens 2.

d. C’est l’oscillogramme 2 qui représente la tension

2. Différencier (Voir tableau ci-dessous.)

aux bornes de la résistance R.

3. Rôle d’une diode a. Une diode est dans le sens passant au cours d’une

Appliquer le cours

seule alternance de la tension alternative qui lui est appliquée. Durant l’alternance suivante, elle est dans le sens non passant et c’est l’autre diode qui éclaire : cela explique leur clignotement à très basse fréquence. b. Une diode permet de redresser une tension alternative pour la rendre unidirectionnelle.

5. Résoudre l’exercice : Pourquoi des piles au lithium ? a. Li → Li+ + e– est une oxydation : le lithium est oxydé en cédant un électron. L’électrode au lithium constitue le pôle « – » de la pile. Dipôles

pile

Propositions

accumulateur en charge

accumulateur en décharge

1. Est le siège d’une réaction d’oxydoréduction

x

x

x

2. Est constitué de deux couples redox et d’une (des) solution(s) ionique(s)

x

x

x

3. Fonctionne en générateur

x

x

4. Est le siège de transformations électrochimiques spontanées

x

x

5. Est le siège des transformations forcées avec un apport d’énergie électrique extérieur

x

6. Fonctionne en récepteur

x

7. Fournit une quantité d’électricité au circuit extérieur

x

8. Est le siège d’une réaction irréversible

x

x

CHAPITRE 2 - Pourquoi éteindre ses phares quand le moteur est arrêté ? • 11

b. Quantité de lithium oxydé : m 0,5 n =  = = 72 × 10–3 mol. M 6,9 Cette oxydation transfère 72  ×  10–3 mole d’électron soit une quantité d’électricité : Q = 72 × 10–3 × 96 500 = 6 950 C correspondant à 1,93 Ah. c. La masse des stimulateurs cardiaques doit être la plus faible possible. On choisit le lithium car, à masse égale, il débite une quantité d’électricité 4,7 fois supérieure à l’argent et 7,8 fois supérieure au zinc et au cadmium (l’utilisation du cadmium est abandonnée).

9. Lampe sans pile : conversion énergie chimique - énergie électrique a. Intensité du courant de décharge : P 0,5 = = 0,17 A. U 3 b. La tension nominale de la lampe à DEL est de 3 V. Elle sera donc en sous-tension à partir de 420 s ; la sous-tension sera significative pour l’éclairement à partir de 600 s ce qui correspond à la promesse de la publicité ; cette dernière reste convenable. c. Pour recharger cette batterie, il faudra utiliser la chaîne de conversions suivante : I = 

6. Cycle charge-décharge pour une batterie au

Rotation de la manivelle

plomb a. Au début de la décharge E = 12,7 V. La tension

Énergie Énergie Énergie Énergie → → → mécanique électrique électrique chimique

décroît au cours de la première demi-heure de décharge, se stabilise à 12 V jusqu’à la huitième heure, décroît ensuite jusqu’à 10,7 V. b. Durant la décharge de l’accumulateur au plomb, on a la réaction spontanée : PbO2 + 4 H+ + 2 SO 2− 4 + Pb → 2 PbSO4 + 2 H2O Un des produits de la réaction est le sulfate de plomb. L’expression « les électrodes se sulfatent » provient de ce corps. c. Au cours de la décharge, la masse volumique de l’électrolyte décroît. Sa valeur renseigne sur l’état de la batterie. d. Au cours de la charge, la tension et la masse volumique de la batterie augmentent pour atteindre respectivement 14,2 V et 1 250 g/L.

alternateur

redresseur

batterie en charge

Exercices à caractère professionnel 10. Électricien automobile a. Le moteur tourne à plein régime, l’alternateur convertit de l’énergie mécanique en énergie électrique : cette énergie maintient la charge de la batterie qui fonctionne en récepteur. La tension à ses bornes U = 14 V est supérieure à sa f.e.m. b. La batterie stocke de l’énergie sous forme chimique. La réaction d’oxydoréduction est forcée par le système alternateur - redresseur. c. Au démarrage du moteur thermique, on utilise le démarreur. Le courant qui l’alimente provient de la batterie d’accumulateurs. Voir schéma :

7. F.e.m. d’une pile zinc – nickel a. Schématisation de la pile zinc-nickel : (–) Zn / Zn2+  Ni 2+ / Ni (+)

b. À la cathode, on a :  Ni2+ + 2e– → Ni  C’est une réduction. 0 c. E =  E Ni

2+

0 − E Zn = (–0,25) – (–0,76) = 0,51 V. 2+ /Ni /Zn

8. Lampe sans pile : conversion énergie électrique – énergie chimique a. - La notation 3,6 V correspond à la f.e.m. de la batterie Ni – Mh (3 éléments). - La notation 80 mAh correspond à la quantité d’électricité. - La notation 02 – 03 – 09 est la date de mise en service. - Ne pas jeter avec les déchets ménagers, objet à recycler de façon spécifique. b. L’équation d’oxydoréduction est : Ni(OH)2 + M → NiOOH + MH. c. Intensité moyenne du courant de charge Q 0,50 × 80 ×10 −3 I =  = = 0,04 × 60 = 2,4 A. 1 t 60

Alternateur

Démarreur

A

M

Phares et accessoires

U = 14 V

Retour par la masse

d. Puisque le moteur thermique ne tourne pas, l’alternateur ne fournit pas d’énergie électrique ; c’est la batterie d’accumulateurs qui alimente les phares : lorsque le moteur ne tourne pas et que les phares sont allumés, la batterie se décharge.

11. Installateur de système photovoltaïque a. Durée de fonctionnement : Q 60 × 0,75 = = 12 h. I 3,75 b. Volume d’eau transféré par la pompe : V = 2 × 102 × 12 × 60 = 144 × 103 L soit 144 m3. c. Puissance de crête Pc = 11,5 W soit U = 16 V. t = 

12 • CHAPITRE 2 - Pourquoi éteindre ses phares quand le moteur est arrêté ?

Ce module peut recharger la batterie d’accumulateurs. d. Pour U = 16 V on a I = 0,75 A. e. Durée de la recharge de la batterie : Q' 45 t’ =  = = 60 h. I ' 0,75 Avec ce système, la recharge de cette batterie s’effectue en cinq fois plus de temps que sa décharge (60 ÷ 12 = 5).

12. Cardiologue a. Période des impulsions : T = 0,8 s. b. 75 impulsions sont émises par minute ce qui correspond au rythme cardiaque moyen d’une personne au repos. c. Quantité d’électricité débitée pour 75 impulsions : Q = 75 × 21 × 10–6 × 0,8 = 1,26 × 10–3 C. Pour un an, la quantité d’électricité débitée est : Q’ = 1,26 × 10–3 × 60 × 24 × 365 = 662 C. Durée de vie de cette pile de stimulateur : 1,6 × 3600 t =  ≈ 8 ans 8 mois. 662 d. Une telle durée permet de rendre peu fréquents son remplacement et l’opération correspondante.

13. Couplage de batteries d’accumulateurs a. Couplage 1 : borne positive A ; borne négative B ; tension aux bornes 24 V ; capacité 200 Ah. b. Couplage 2 : borne positive A ; borne négative B ; tension aux bornes 48 V ; capacité 100 Ah.

14. Débranchement de batterie (Démarche d’investigation) La gerbe d’étincelles est due au court-circuit provoqué lorsqu’on a mis en contact, à l’aide de la clé, la carrosserie (masse) de la voiture avec la borne plus de la batterie. Le circuit se représente par le schéma suivant :

E : f.e.m. = 12 V r : résistance de

E, r

la batterie = 0,05 

E 12 = = 240 A. Un tel courant r 0,05 peut endommager la batterie. Il vaut mieux commencer par ouvrir le circuit en débranchant la borne (-).

On a E = r .  I d’où I = 

3 Comment se déplacer dans un fluide ? T5 Les activités ■■Activité 1 Comment mettre en évidence la force de poussée d’Archimède ? • Matériel Une balance électronique - Trois béchers de 100 mL - Un dynamomètre 2 N Deux solides de même volume et de poids différents Deux solides de même poids et de volumes différents

• Produits Pâte à modeler Eau

Expérience 1 À l’aide d’une balance électronique, on prépare deux boules de pâte à modeler de même masse. Dans cette expérience, l’élève doit trouver une forme à la pâte à modeler pour qu’elle ne coule pas. Les béchers contiennent le même volume d’eau pour comparer les volumes d’eau déplacée avec la pâte à modeler en forme de boule et celle en forme de cuvette suffisamment creuse pour qu’elle flotte.

Expérience 2 Il existe des cylindres métalliques (aluminium, fer, laiton, zinc) de volumes identiques et des cylindres de masses égales (Pierron) avec des crochets de fixation permettant de réaliser l’expérience.

CHAPITRE 3 - Comment se déplacer dans un fluide ? • 13

Mesures Poids d’un solide 1 : - lorsqu’il est suspendu dans l’air : 3 N ; - lorsqu’il est totalement immergé dans l’eau : 2,6 N. Avec un second solide 2 de même volume mais de poids plus faible, on a mesuré : - lorsqu’il est suspendu dans l’air : 1 N ; - lorsqu’il est totalement immergé dans l’eau : 0,6 N. Avec un troisième solide 3 de même poids mais de volume plus grand : - lorsqu’il est suspendu dans l’air : 3 N ; - lorsqu’il est totalement immergé dans l’eau : 1,9 N.

• Réponses aux questions Expérience 1

1. La pâte à modeler en forme de cuvette flotte et déplace davantage d’eau. Expérience 2

2. Valeur de la poussée exercée par l’eau : - sur le solide 1 : 3 – 2,6 = 0,4 N ; - sur le solide 2 : 1 – 0,6 = 0,4 N ; - sur le solide 3 : 3 – 1,9 = 1,1 N.

3. Les mesures montrent que la valeur de la poussée exercée par l’eau ne dépend pas du poids du solide, mais du volume du solide immergé.

4. La poussée exercée par l’eau sur un solide immergé est une force verticale dirigée vers le haut. 5. Les bateaux en acier flottent, car leur coque en forme de cuvette permet d’augmenter le volume de la partie immergée dans l’eau, donc d’augmenter la poussée exercée par l’eau. La poussée peut alors compenser le poids.

■■Activité 2 Comment varie la pression au sein d’un liquide ? • Matériel Une grande éprouvette - Un manomètre électronique - Une règle graduée

Mesures Différence de profondeur entre les deux points A et B : h = AB = 20 cm = 0,2 m pA = 1 003 hPa = 100 300 Pa pB = 1 023 hPa = 102 300 Pa pB – pA = 2 000 Pa ρ.g.h = 1 000  ×  10  ×  0,2 = 2 000

• Réponses aux questions 1. La pression augmente lorsque la profondeur augmente. 2. La différence de pression pB – pA est égale au produit ρ.g.h

■■Activité 3 Pourquoi un avion se maintient-il en vol ? • Matériel Une feuille de papier - Une règle

• Réponses aux questions 1. Lorsque l’on souffle sur la partie bombée de la feuille de papier, celle-ci remonte. 2. La pression au-dessous de la feuille de papier est supérieure à la pression au-dessus de la feuille de papier.

3. La force exercée par l’écoulement de l’air sur l’aile d’avion comporte une composante verticale dirigée vers le haut.

14 • CHAPITRE 3 - Comment se déplacer dans un fluide ?

Les documents ■■À quels principes de la Physique obéissent les sous-marins ? • Réponse aux questions  1. On applique la formule pB – pA = ρ.g.h avec h = 10 m : pB – pA = 1 000 × 10 × 10 = 105 Pa ou 1 bar. Donc, la pression dans l’eau augmente de 1 bar lorsqu’on descend de 10 m.

2. L’eau dans les ballasts est à la même pression que l’eau de mer, donc les forces pressantes qui s’exercent sur les deux faces de la coque externe se compensent. La coque externe est donc de faible épaisseur.

3. Lors de la remontée à la surface, le sous-marin diminue son poids en vidant l’eau contenue dans les ballasts par de l’air comprimé. Par contre la poussée d’Archimède ne varie pas tant que le sousmarin est totalement immergé.

Les exercices Tester ses connaissances

de 3 105 hPa. b. la pression indiquée par le manomètre électronique lorsqu’il n’est pas en fonctionnement est de 1 005 hPa, c’est la pression atmosphérique. c. La pression réelle de l’air dans le pneu est de 3 105 hPa soit 3,1 bar. d. Le manomètre électronique mesure la pression absolue, le contrôleur de pression mesure la pression relative. e. Pression absolue = pression relative + pression atmosphérique.

Q.C.M. 1 : A et C ;  2 : A et C ;  3 : A et C  4 : A et B ;  5 : A et C ;  6 : A et C ;  7 : B.

Tester ses capacités 1. Solide immergé La valeur de la poussée d’Archimède est : FA = 16 – 10 = 6 N.

2. Pressiomètre a. La mesure est réalisée en hectopascal (hPa). b. p = 1 031 hPa = 103 100 Pa ou 1,03 bar.

5. Sous-marin en plongée

3. Expérience a. Courbe p = f (h). Il est possible de tracer la courbe à l’aide d’un tableau-grapheur. pression p (Pa)

p = f (h)

6. Plongée sous-marine La pression à 30 m de profondeur est : p = ρ.g.h + patm = 103 × 10 × 30 + 105 = 4 × 105 Pa, soit 4 bar.

7. Aileron de voiture a. La feuille de papier est aspirée par l’air. b. La pression sous la feuille est inférieure à la pres-

105000 104500 104000 103500

sion de l’air au-dessus de la feuille.

103000

c. Le principe de Venturi est mis en évidence dans

102500

cette expérience. Enoncé du principe de Venturi : la pression d’un fluide diminue lorsque la vitesse de son écoulement augmente.

102000 101500 101000 100500 0,05

La différence de pression est : p B – p A = ρ.g.h = 1 030 ×  9,8 × 280 = 2 826 320 Pa ; soit 28,3 bar.

0,10

0,15

0,20

0,25

0,30

profondeur h (m)

Appliquer le cours

b. La pression augmente avec la profondeur.

8. Le Nautile a. Pression de l’eau : p = ρ.g.h + patm

4. Pression de l’air a. La pression indiquée : - par le contrôleur lors de la mesure est de 2,1 bar ; - par le manomètre électronique lors de la mesure est

= 1 030 × 10 × 6 000 + 105 = 619 × 105 Pa (ou 619 bar). b. Aire d’un hublot : S = πR² = π × 0,06² = 1,13 × 10–2 m².

CHAPITRE 3 - Comment se déplacer dans un fluide ? • 15

Force pressante : F = p × S = 619 × 105 × 1,13 × 10–2 = 699 470 N.

9. À vous de résoudre 

a. Poids P du cylindre : force verticale appliquée au centre de gravité G du cylindre, dirigée vers le bas, de valeur égale à 1,7 N.  Force T exercée par le dynamomètre : force verticale dirigée vers le haut, de valeur égale à 1,7 N. b. À l’échelle 1 cm pour 0,5 N, les deux forces sont représentées par des flèches de 3,4 cm de longueur (échelle du dessin : 50%). c. À l’équilibre, le cylindre est soumis à trois  forces : - le poids P du cylindre : force verticale appliquée au centre de gravité G du cylindre, dirigée vers le bas, de valeur égale à 1,7 N ;  - la force T exercée par le dynamomètre : force verticale appliquée au crochet de fixation du cylindre et du dynamomètre, dirigée vers le haut ; 

a)

qui s’exerce sur la boule entre deux eaux diminue et devient inférieure à son poids. La boule tombe au fond du tube de verre. c. Les boules situées au fond du tube indiquent des températures plus élevées que la température ambiante.

Exercices à caractère professionnel

T

11. Vanne d’un réservoir a. p = ρ.g.h + patm = 860 × 10 × 1,2 + 95 000 = 105 320 Pa. La pression eu niveau de la vanne est égale à 105 320 Pa. b. F = p.S = 105 320 × 3 × 10–3 = 316 N. La valeur de la force pressante exercée par le gasoil est égale à 316 N. c. La vanne est adaptée car 316 N < 400 N.

G P

12. Ballon-sonde

b)

4  × π × 0,753 = 1,77 m3. 3 P = ρhélium.g .V = 0,18 × 10 × 1,77 = 3,19 N.  La valeur du poids P du ballon est égale à 3,19 N. b. FA = ρair.g .V = 1,29 × 10 × 1,77 = 22,83 N. c. À l’équilibre : T = FA – P = 22,83 – 3,19 = 19,64 N.  La valeur de la tension T du fil est égale à 19,64 N. 19,64 d. M =  = 1,964 kg ou 1 964 g. 10 La masse maximale du capteur que peut soulever le ballon est de 1 964 g.

a. Volume du ballon : V = 

T G

FA

P

- la poussée d’Archimède FA exercée par l’eau : force verticale appliquée au centre de poussée C confondu avec G, dirigée vers le haut. d. FA = ρeau.g.V = 103 × 10 × 100 × 10–6 = 1 N. La valeur de la poussée d’Archimède est égale à 1 N. e. 1,7 – 1 = 0,7 N. La valeur de la force verticale exercée par le ressort est égale à 0,7 N.  f. À l’échelle 1 cm pour 0,5 N, la force P est représentée par une flèche de 3,4 cm de longueur, la force   FA par une flèche de 2 cm de longueur et la force T par une flèche de 1,4 cm de longueur (échelle du dessin : 50%). g. Le dynamomètre indique 0,7 N.

10. Thermomètre de Galilée a. La boule située entre deux eaux est soumise : - à son poids : force verticale, dirigée vers le bas ; - à la poussée d’Archimède. : force verticale, dirigée vers le haut, de valeur égale au poids de la boule. b. Lorsque la température augmente, la masse volumique du liquide contenu dans le tube de verre diminue, donc la valeur de la poussée d’Archimède

13. Remontée d’une amphore a. Valeur du poids de l’amphore : P = m.g = 25 × 10 = 250 N Valeur de la poussée d’Archimède exercée par l’eau de mer sur l’amphore : FA = ρeau.g.V = 1 030 × 10 × 15 × 10–3 = 154,5 N. b. L’expression de la valeur F’A de la poussée d’Archimède qui s’exerce sur le réservoir de volume V du parachute : F’A = ρeau.g.V = 10 300 × V On néglige le poids du parachute. Pour remonter l’amphore, il faut : F’A = P – FA = 250 – 154,5 = 95,5 N. 95,5 Soit V =  = 9,3 × 10–3 m3, soit 9,3 L. 10300 Le réservoir du parachute doit contenir au moins 9,3 L d’air pour remonter l’amphore.

14. Ceinture de lest a. P = m.g = 95 × 10 = 950 N. Le poids du plongeur est égal à 950 N.

b. FA = ρeau.g.V = 103 × 10 × 102 × 10–3 = 1 020 N. La valeur de la poussée d’Archimède est égale à 1 020 N.

16 • CHAPITRE 3 - Comment se déplacer dans un fluide ?

c. Le plongeur ne coule pas, car la poussée d’Archimède est supérieure à son poids. 70 d. 1 020 – 950 = 70 N, m =  = 7 kg. 10 Le plongeur devra accrocher à sa ceinture 7 kg de lest pour pouvoir descendre.

15. Aile de planeur a. Comme S2 < S1, alors

S1

> 1, par conséquent

2 S2 S  1   > 1, donc p1 – p2 est positif.  S2 

16. Paquebot (Situation problème) a. Le tirant d’eau d’un bateau est la profondeur à laquelle il s’enfonce dans l’eau.

b. Schéma de la situation : H = 12 m, L = 290 m, l = 30 m.

H h

H h L



vue de face

Ainsi p1 > p2, la pression de l’air au-dessous de l’aile est supérieure à la pression de l’air au-dessus de l’aile. La zone de dépression au-dessus de l’aile aspire l’aile vers le haut. 2 1 b. p1 – p2 =   × 1,2 × 50² ×  ( 2 −1 = 4 500 Pa.   2 c. F = (p1 – p2) . S = 4 500 × 10 = 45 000 N. La valeur de la force exercée sur une aile est égale à 45 000 N.

)

vue de coté

Le paquebot flotte donc la valeur de son poids est égale à la valeur de la poussée d’Archimède : P = FA. P = m.g = 72 × 103 × 103 × 10 = 72 × 107 N FA = ρ.g.V = ρ.g.l .L .h Soit ρ.g.l.L.h = P, le tirant d’eau du paquebot est 72 ×107 P donc : soit h =  =  = 8 m. ρ.g .l.L 1030 ×10 × 30 × 290 Le paquebot pourra emprunter le canal de 12 m de profondeur.

4 Qu’est-ce qu’une voiture puissante ? T6 ■■Activité 1 Comment calculer le moment d’un couple ? • Matériel

Tableau de résultats en prenant g = 10 N/kg La distance d est réglée à 30 cm.

- Couplemètre - Masses marquées de 100 g, de 200 g

Masse (kg) 0,1 0,2 0,3

• Réponses aux questions 1. Un couple ℳ est proportionnel à la valeur F de la force commune. Il est également proportionnel à la distance d entre les droites d’action des deux forces.

Force F (N) 1 2 3

Couple ℳ (N.m) 0,30 0,60 0,90

Rapport

 F

0,30 0,30 0,30

2. La relation qui permet de calculer un couple ℳ est : ℳ = F.d. 3. On a ℳ = F.r. 4. Un couple moteur est le couple disponible sur l’arbre du moteur lorsque le moteur tourne à un régime donné.

■■Activité 2 Quelle est la puissance d’un moteur ? • Tableau des valeurs : n (tr/min)

1 500

2 000

2 500

n (tr/s)

25

33,3

ℳ (N.m)

225

260

35 × 103

54 × 103

P (W)

3 000

3 500

4 000

4 500

5 000

41,7

50

58,3

255

237

215

66,7

75

83,3

187

162

137

67 × 103

75 × 103

79 × 103

78 × 103

76 × 103

72 × 103

CHAPITRE 4 - Qu’est-ce qu’une voiture puissante ? • 17

• Courbe représentant la puissance P en fonction du régime moteur n (tr/min) ci-contre.

Couple  (Nm)

Puissance P (kW)

275

80

250

70

225

60

200

50

contrôler le régime moteur.

175

40

2. Pmax = 80 kW et ℳmax = 260 N.m

150

30

3. Le couple moteur dépend de

125

20

• Réponses aux questions 1. Le tachymètre permet de

la vitesse de rotation du moteur. 110 ch représentent la puissance 1 500 2 000 2 500 3 000 3 500 4 000 4 500 maximale du moteur (souvent utilisée par les commerciaux pour indiquer la puissance du moteur).

Régime moteur

5 000 n (tr/min)

4. Les régimes moteur permettant une utilisation optimale des performances couple-puissance sont tels que : 2 000 tr/min < n < 3 500 tr/min.

Les documents ■■La turbocompression • Réponses aux questions 1. La turbine et le compresseur sont réunis par un axe rigide : ils tournent à la même fréquence de rotation.

2. On peut augmenter la quantité air – carburant d’un moteur thermique en augmentant la cylindrée du moteur. Si on augmente l’alésage (diamètre intérieur du cylindre) ou la course du piston, le moteur devient plus encombrant et plus lourd. - Un moteur turbocompressé est moins encombrant qu’un moteur atmosphérique de même puissance. - En compressant de l’air, il apporte en altitude plus de puissance qu’un moteur atmosphérique qui subit la raréfaction de l’oxygène. - Il est moins polluant, plus silencieux et consomme moins.

3. Il faut refroidir l’air comprimé dans un système air/air ou air/eau car, lors de sa compression, la température augmente. Le fait de le refroidir augmente sa densité et améliore ainsi le remplissage (gavage) des cylindres en mélange air/carburant : le rendement du moteur augmente.

4. La puissance développée par un moteur est proportionnelle à la quantité de carburant consommée en une seconde. Il faut qu’il y ait suffisamment d’air nécessaire à cette combustion : la turbocompression remplit cette fonction en augmentant la quantité d’oxygène.

Les exercices Tester ses connaissances Q.C.M. 1 : C ;  2 : A ;  3 : B ;  4 : A et B.

Tester ses capacités 1. Clé à bougie a. Dans les trois cas, on a :

18 • CHAPITRE 4 - Qu’est-ce qu’une voiture puissante ?

ℳ = F.d = 100 × 0,30 = 30 N.m.  

b. La valeur du moment du couple ( F , F ’ ) est indépendante de la position de l’axe de rotation (∆).

2. Régler et exercer un couple de serrage à l’aide d’une clé dynamométrique Valeur d’une force du couple :  10 2 F =  = = 7,1 × 103 N. d 14 ×10 −3

3. Moteur de Mégane a. Couple maximal ℳmax = 340 N.m  pour une fréquence de rotation égale à 2 200 tr/min. 2200 b. P = 2 π.n.ℳ = 2 π × × 340 = 78 × 103 W = 78 kW. 60 c. Pmax = 110 kW correspond à un couple de 260 N.m. Cette puissance se produit pour une fréquence de rotation de 4 000 tr/min : elle est supérieure à celle qui correspond au couple maximal (2 200 tr/min).

EC

f

2

 100  1 1 =  mv2 =  × 920 ×  = 355 × 103 J. 2 2  3,6 

b. P = 

∆E c ∆t

=

ΩM = R.R’.2π.n = 0,220 × 2π × 

M 130 = = 591 N.m. R.R' 0,220 D b. Vitesse du véhicule : V = ΩR . = 62,2 × 0,30 2 = 18,7 m/s soit 67 km/h.  c. ℳR =  M Au démarrage ou en montée, puisque R.R' ℳR doit être élevé, il faut que R.R’ soit faible. - Prenons R.R’ = 0,078 pour un démarrage en 1re : pour un régime moteur de 2 700 tr/min, ΩM = 2 × π × 2 700/60 = 283 rad/s  et  ΩR = 0,078 . ΩM D D V = ΩR . = 0,078.ΩM . 2 2 = 0,078 × 283x 0,30 = 6,62 m/s = 6,62 × 3, 6 = 24 km/h. - Prenons R.R’ = 0,142 pour une montée en 2nde : pour un régime moteur de 2 700 tr/min, ΩM = 2 × π × 2 700/60 = 283 rad/s. D D V = ΩR . = 0,142.ΩM . 2 2 = 0,142 × 283x 0,30 = 12,06 m/s = 12,06 × 3,6 = 43 km/h. ΩR = ΩM.R.R’ : sur autoroute, la vitesse angulaire des roues doit être la plus élevée possible donc on choisira R.R’ = 0,366 ; le rapport de boîte correspondant est 39/31 pour rouler en 5e. D D V = ΩR . = 0,366 . ΩM . = 0,366 × 283 × 0,30 2 2 = 31,07 m/s = 31,07 × 3,6 = 112 km/h. (pour un régime moteur de 2 700 tr/min).

Appliquer le cours 5. Résoudre l’exercice a. P = 400 ch correspond à 400 × 736 = 294 × 103 W. La vitesse angulaire : 8500 = 890 rad/s. 60 P 294 ×103 b. Couple-moteur : ℳ =  = = 330 N.m. Ω 890 c. Énergie cinétique : 2  100  1 1 Ec =  m.v² =   × 1 450 ×   = 559 × 103 J.  2 2  3,6  d. Puissance moyenne : ∆E c 559 ×103 − 0 P =  = = 122 × 103 W soit 122 kW. 4,6 ∆t Ωm = 2π.n = 2π.

6. Convertir a. 110 ch correspond à 80 960 W soit 109 hp. = 6 chevaux fiscaux.

130  80,960  + 45  40 

1,6

7. Couple de déblocage Valeur de la force F = 

8. Le foret

 0,90 = = 32,1 N. d 28 ×10 −3

ℳ = 3 × F × r = 3 × 103 × 10–2 = 30 N.m.

Exercices à caractère professionnel 9. Transmission par engrenage Z1 50 = = 2. Z 2 25 b. Fréquence de rotation de l’arbre de sortie : Ω n si R =  2 = 2 alors n2 = R . n1 = 2 × 1 500 Ω1 n1 = 3 000 tr/min soit 50 tr/s. c. Couple sur l’arbre de sortie :   100 si R =  1 alors ℳ2 =  1 =  = 50 N.m. 2 2 R

a. Rapport : R = 

2700 = 62,2 rad/s. 60

ℳR = 

355 ×103 = 27,3 × 103 W soit 27,3 kW. 13

b. Puissance fiscale Pa = 

Ω2 Z1 44 = = = 3,38. Ω1 Z 2 13 b. Si n1 = 108 tr/min alors n2 = R  ×  n1 = 3,38  ×  108 365 = 365 tr/min et Ω2 = 2.π.n2 = 2π ×  = 38,2 rad/s. 60 0,70 c. Vitesse du vélo : v =  38,2 × = 13,4 m/s soit 2 48,3 km/h.

a. R = 

11. Du régime moteur à la vitesse du véhicule a. En troisième R.R’ = 0,220. On a ΩR = R.R’.

4. Performance d’une Twingo a. Au départ E C = 0 ; à 100 km/h : i

10. Transmission par chaîne

12. Rendement moteur-transmission a. Puissance fournie : Pm = 210 – 100 = 110 kW. b. Rendement thermique : ηth = 

Pm Pth

=

110 = 0,52 soit 210

52 %. c. Puissance disponible sur l’arbre des roues : P ηm =  R donc PR = ηm . Pm = 0,89 × 110 = 98 kW, Pm soit 133 ch. d. Volume du carburant utilisé pour produire de la puissance utile sur l’arbre des roues : 6 × 0,52 = 3,1 L. CHAPITRE 4 - Qu’est-ce qu’une voiture puissante ? • 19

13. Kilomètre départ arrêté Énergie cinétique acquise au kilomètre : 2  150  1 1 = 1,04 × 106 J. Ec =  mv2 =   × 1 200 ×   2 2  3,6  Masse d’essence consommée : mess = 0,8 × 0,2 = 0,16 kg. Énergie thermique libérée : Eth = 45 × 106 × 0,16 = 7,2 × 106 J. E 1,04 ×106 = 0,14 soit Rendement global : η =  c = E th 7,2 ×106 14 %.

14. Conduite adaptée (démarche d’investigation) a. Le régime moteur classique est limité entre 2 000 tr/min et 3 500 tr/min. Il en est de même pour le couple moteur. Les rapports de boîte permettent d’adapter aux conditions d’utilisation du véhicule le couple et la vitesse angulaire qui sont transmis aux roues.  • On a ℳR =  M avec ℳR couple transmis aux R.R' roues, ℳM couple moteur, R rapport de boîte, R’ rapport de pont. Au démarrage, phase d’accélération,

le couple ℳR au niveau des roues est élevé donc on sélectionne un rapport de boîte faible : celui de la 1re. • ωR = ωM × R.R’ avec ωR vitesse angulaire des roues, ωM vitesse angulaire du moteur, R rapport de boîte, R’ rapport de pont. Sur autoroute, la vitesse angulaire des roues doit être élevée donc on choisit un rapport de boîte élevé : celui de la 5e ou 6e.

b. Une voiture nerveuse est une voiture qui possède une bonne accélération. Il lui faut donc disposer d’un bon couple à bas régime. Elle ne doit pas attendre de monter en régime pour disposer d’un couple important.

c. On appelle reprise le passage du bas régime d’un moteur à un régime supérieur sans utilisation du changement de vitesse. Un véhicule qui a de bonnes reprises accélère rapidement.

d. Avec 140 ch, on dispose d’un couple plus élevé qu’avec 110 ch et à masse égale, on a de meilleures reprises. La masse du véhicule intervient : une moto de 110 ch démarre plus rapidement qu’une voiture de 140 chevaux.

5 Comment avoir une bonne tenue de route ? T7 Les activités ■■Activité 1 Comment expliquer l’écrasement d’un pneu sous-gonflé ? • Matériel Un vélo - Des feuilles de papier blanc - Un pèse-personne - Un contrôleur de pression

• Produits De l’alcool.

Expérience Aire des surfaces de contact : S1 = 6,8 cm², S2 = 7,1 cm² Aire totale : S = 13,9 cm² Masse du vélo : 13 kg Masse de l’élève : 62 kg Masse totale : 75 kg Poids total : P = 750 N P 750 Pression p exercée sur le sol : p =  = ≈ 5,4 × 105 Pa = 5,4 bar. S 13,9 ×10 −4

• Réponses aux questions

1. La pression p exercée sur le sol par l’ensemble « vélo-élève » est égale à la pression de l’air contenu dans la chambre à air du vélo.

2. Un pneu sous-gonflé est écrasé, car la pression de l’air dans le pneu est faible et la surface de contact du pneu avec le sol est grande.

20 • CHAPITRE 5 - Comment avoir une bonne tenue de route ?

■■Activité 2 Comment mesurer la période d’une oscillation ? • Matériel Un capteur de force - Un ordinateur + interface EXAO - Un support Une boîte de masses marquées - Un ressort de raideur connue

Expérience Ressort de raideur : k = 15,4 N/m Lorsque la masse est écartée de 3 cm puis de 6 cm de sa position de repos, les enregistrements obtenus montrent que les oscillations du système oscillant ont même période. Les mesures obtenues sont données dans le tableau : Masse

m = 100 g

m’ = 200 g

Période T

0,49 s

0,70 s

Fréquence f

2 Hz

1,4 Hz

0,51 s

0,72 s



m k

• Réponses aux questions 1. La période de l’oscillateur est T = 0,49 s pour une masse de 100 g. 2. La fréquence f des oscillations du système oscillant ne dépend pas de l’amplitude des oscillations. 3. La période T des oscillations augmente lorsque la masse m augmente. 4. Les périodes mesurées vérifient la formule T = 2π

m . k

■■Activité 2 Quelle est l’influence de l’amortissement sur les oscillations ? • Matériel Le même matériel que dans l’activité 2 Un grand bécher rempli d’eau

Expérience Avec la masse de 100 g, on obtient une période T = 0,53 s . La période des oscillations amorties est légèrement supérieure à la période du système oscillant librement dans l’air.

• Réponses aux questions 1. Lorsque l’oscillateur oscille dans l’eau, l’amplitude des oscillations diminue : les oscillations sont qualifiées « d’oscillations amorties ».

2. Les suspensions d’une automobile limitent les oscillations du véhicule. Elles apportent davantage de confort et de sécurité.

Les documents ■■Les pneumatiques • Réponse aux questions 1. La part des caoutchoucs dans la composition d’un pneu est la suivante : 5 % de caoutchouc naturel et 15 % de caoutchoucs synthétiques.

2. La pression réelle de l’air dans un pneu est de 3,1 bar lorsque le manomètre de gonflage indique 2,1 bar.

3. Un pneu froid est un pneu qui n’a pas roulé. La pression augmente avec la température. Pour une mesure avec un pneu chaud, il faut retrancher 0,3 bar pour avoir la pression équivalente recommandée par le constructeur. Il est donc conseillé de contrôler la pression des pneus à froid.

CHAPITRE 5 - Comment avoir une bonne tenue de route ? • 21

4. La pression d’un pneu sur-gonflé est importante. Pour une même force pressante, la surface de contact du pneu avec le sol est donc réduite.

5. L’indication 4 indique que le pneu photographié doit être monté sur une jante de 16 pouces de diamètre.

6. L’indice de charge est un code numérique qui correspond à la charge maximale qu’un pneu peut supporter (1). Ainsi, l’indice de charge 95 correspond à une charge maximale de 690 kg autorisée par le pneu. - L’indice de vitesse est un code alphabétique qui correspond à la vitesse maximale à laquelle un pneu peut rouler (2). Ainsi, l’indice de vitesse W correspond à une vitesse maximale de 270 km/h.

92

630

U

200

93

650

H

210

94

670

V

240

95

690

ZR

> 240

96

710

W

270

97

730

Y

300

98

750

99

775

(2)

(1)

Les exercices Tester ses connaissances

7. Calculer une pression

Q.C.M. 1 : A et B ;  2 : B et C ;  3 : B ;  4 : B et C ;  5 : B et C ;  6 : B.

a. p = 

Tester ses capacités

pression est donc dix fois plus petite, soit 6 × 104 Pa.

1. Avec un chronomètre La période de l’oscillateur est : T = 0,72 s.

2. À partir d’un enregistrement EXAO La période d’une oscillation est : T = 1,38 – 0,518 = 0,862 s.

3. Enregistrements a. Les deux mouvements oscillatoires ont la même période mais des amplitudes différentes.

b. La période est : T = 2,11 – 1,17 = 0,94 s. 1 1 = = 1,06 Hz. T 0,94 d. La fréquence f ne dépend pas de l’amplitude des oscillations.

c. La fréquence est : f = 

4. Fréquence propre 800 ms = 0,8 s ; la fréquence propre est : 1 1 f =  = = 1,25 Hz. T 0,8

5. Période d’un mouvement oscillatoire La période de l’oscillateur est : T = 

1 1 = = 0,4 s. f 2,5

6. Expérience a. Lorsque la force pressante augmente, la pression augmente.

b. Lorsque la surface pressée diminue, la pression augmente.

c. La relation ainsi vérifiée est : p = 

F . S

F 3000 =  = 6 × 105 Pa. S 50 ×10 −4

b. L’aire de la surface pressée est multipliée par 10, la

8. Calculer une force pressante 90 bar = 90 × 105 Pa ; 20 cm² = 20 × 10–4 m². La valeur de la force exercée par le vérin hydraulique est : F = p.S = 90 × 105 × 20 × 10–4 = 18 000 N.

9. Pelle mécanique Aire de la surface pressée : S = 2 × 2,4 × 0,6 = 2,88 m². La force pressante est égale au poids de la pelle mécanique : F = P = m.g = 13 000 × 10 = 130 000 N. La pression exercée est donc : F 130000 p =  = = 45 139 Pa ou 0,45 bar. S 2,88

Appliquer le cours 10. À vous de résoudre a. Pression de l’air à l’intérieur des pneus : - avant d’avoir roulé : 2,1 + 1 = 3,1 bar ; - après avoir roulé : 1,8 + 1 = 2,8 bar. b. A la température de 18 °C, la température absolue est T = 18 + 273 = 291 K. Le volume du pneu étant considéré constant, la forp.V p mule = Cte s’écrit = Cte. T T 3,1×105 2,8 ×105 = . On a donc : 291 T 5 2,8 ×10 × 291 = 263 K ou – 10 °C. Soit T =  3,1×105

22 • CHAPITRE 5 - Comment avoir une bonne tenue de route ?

11. Période d’un oscillateur m 0,2 a. Période : T = 2π = 2π = 1,5 s. k 3,5 30 b. = 20. L’oscillateur élastique va effectuer 1,5 20 oscillations.

12. Calculer la raideur d’un ressort 6 = 0,4 s. 15 1 1 Fréquence propre du pendule : f =  = = 2,5 Hz. T 0,4

a. Période : T = 

2

4 π .m 4 π 2 × 0,15 m = donc k =  = 37 N/m. k T2 0, 4 2 La raideur du ressort est k = 37 N/m.

b. T = 2π

13. Exploiter un enregistrement a. L’oscillateur est amorti, car l’amplitude des

16. Roue mal équilibrée a. 80 ÷ 3,6 = 22. La vitesse du véhicule est de 22 m/s. b. Fréquence des vibrations du volant : f = 

v 22 = = 12 Hz. π.D π × 0,6

17. Test des amortisseurs a. Les oscillations imposées à la carrosserie au niveau

oscillations diminue.

b. Le mouvement est pseudopériodique. c. La pseudo-période est T = 0,4 s. m 0,05 = 2π = 0,39 s. k 12,8 e. Les valeurs de la pseudo-période T et de la période propre To sont voisines.

d. Période propre To = 2π

Exercices à caractère professionnel 14. Pneu correctement gonflé a. Le pneu sur-gonflé exerce la pression la plus importante sur le sol, car la surface de contact est la plus petite. b. Il faut gonfler correctement les pneus d’un véhicule pour que la surface de contact entre le sol et le pneu soit grande de façon à avoir une meilleure adhérence.

15. Pneumatique de formule 1 a. Poids de la Formule 1 : P = m.g = 605 × 10 = 6 050 N.

b. Valeur de la force pressante exercée sur une roue : 6050 = 1 512,5 N. 4 c. Pression absolue dans les pneus: p = 2 bar Aire de la surface totale d’adhérence sur la route : S = F/p S = 6050 /(2 × 105) = 0,0302 m2 = 302 cm2. F = 

Aire de la surface d’adhérence par pneu : 302/4 = 75 cm2. d. Aire de la surface de la bande de roulement de chaque pneu en contact avec le sol : 35 × 27 = 945 cm2. Rapport entre l’aire d’adhérence et l’aire de la surface de la bande de roulement en contact avec le sol : 75/945 = 8 % . Ce rapport est nettement inférieur à 1. La route présente des aspérités et l’adhérence des pneus s’effectue sur ces aspérités et non sur toute la surface.

des roues avant cessent rapidement, au bout de 0,6 s, alors que pour les roues arrière il faut 1,4 s pour que les oscillations cessent. Les amortisseurs des roues avant sont donc plus performants que les amortisseurs des roues arrière. b. L’amortissement est plus faible pour les roues arrière.

18. Variation de pression dans un pneu (Situation problème) La pression diminue avec la température, donc la pression du pneu contrôlé à 20 °C va diminuer à la température de 10 °C. Pour calculer cette baisse de pression, on applique la p.V = Cte. formule : T Comme le volume du pneu est supposé constant, on p = Cte. écrit T 2 ×105 p =  , soit p = 1,9  ×  105 Pa ou On a donc : 283 293 1,9 bar. Lorsque la température passe de 20 °C à 10 °C, la pression baisse donc de 0,1 bar. Pour compenser cette baisse de pression en hiver, on augmente la pression de gonflage du pneu de 0,1 à 0,2 bar. Un pneu sous-gonflé se déforme et risque d’éclater.

CHAPITRE 5 - Comment avoir une bonne tenue de route ? • 23

6 Comment faire varier la vitesse d’un véhicule électrique ? T8

les Activités ■■Activité 1 Comment évolue la fréquence de rotation d’un moteur à courant continu ? • Matériel Un générateur continu à tension variable (30 V – 5 A) Un banc moteur – génératrice - Le système Pierron comporte un couple mètre Un ampèremètre – Un voltmètre Un fréquencemètre (ou oscilloscope) Une charge (rhéostat 10 Ω – 5 A) Des cordons.

Expérience 1 : Étude du moteur à vide Pour mesurer la fréquence de rotation de l’arbre, on utilise un aimant placé sur le manchon de couplage. Lorsqu’il tourne, il y a naissance dans une bobine d’une tension induite (loi de Faraday) : sa fréquence indique la fréquence de rotation du E (V) moteur. 12 Pour que le moteur tourne à vide, il faudrait 10 le découpler de la génératrice mais cela n’est 8 pas prévu dans ce type de matériel. On se contentera de débrancher le rhéostat. En 6 augmentant la tension, on relève les mesures 4 de U, I et n. 2 On calcule la valeur de E = U – R.I n (tr/s) 0 pour obtenir le tableau suivant et le 0 20 40 60 80 100 120 140 160 180 200 graphique ci-contre. E (V)

0

1,7

2,6

4,1

5,7

8,7

10,9

n (tr/s)

0

25

40

64

92

142

180

Expérience 2 : Étude du moteur en charge Mettre le rhéostat aux bornes de la génératrice sur sa valeur maximale de 10 Ω ; en diminuant cette résistance, l’intensité du courant dans la génératrice augmente. C’est ainsi que la charge du moteur croît. Pour ce moteur, l’intensité du courant ne doit pas dépasser 3,5 A. On obtient le tableau suivant et le graphique ci-contre. I (A) ℳR × 103 (N.m)

1,6 8,9

2,1 13

2,5 16

2,9 20

3,4 24

 R × 10 3 (N.m)

30 25 20 15 10 5 0

I (A) 0

0,5

1

1,5

24 • CHAPITRE 6 - Comment faire varier la vitesse d’un véhicule électrique ?

2

2,5

3

3,5

4

• Réponses aux questions Expérience 1

1. Une fonction linéaire lie E et n et on peut écrire E = kn. À vide, la f.e.m. du moteur est proportionnelle à sa fréquence de rotation.

2. Pour faire varier la fréquence de rotation n d’un moteur, il faut modifier sa tension d’alimentation U.

Expérience 2

3. Lorsque le couple résistant ℳR imposé par la charge du moteur augmente, l’intensité du courant appelé par ce moteur augmente. Les accroissements du couple résistant sont proportionnels à ceux de l’intensité du courant.

■■Activité 2 Comment évolue la fréquence de rotation d’un moteur asynchrone ? Expérience 1 : Principe d’un moteur asynchrone

• Matériel Une platine de trois bobines et une cage d’écureuil mobile autour d’un axe vertical Une alimentation triphasée très basse tension Un tachymètre Des cordons.

Expérience 2 : Variation de la fréquence de rotation

n (tr/min)

2 000 1 800 1 600

• Matériel Un variateur monté sur une platine Un moteur asynchrone Des cordons. Ici, on se sert d’un dispositif utilisé par les professeurs d’enseignement professionnel. Le graphique n (tr/min) en fonction de f (Hz).

• Réponses aux questions

1 400 1 200 1 000 800 600 400 200 0

f (Hz) 0

10

20

30

40

50

60

70

Expérience 1

1. La partie inductrice est constituée par les trois bobines alimentées par des tensions alternatives. Elles produisent un champ variable. La partie induite est constituée par la cage d’écureuil qui est le siège de courants induits engendrant un couple électromagnétique.

2. La fréquence de rotation de la cage d’écureuil est légèrement inférieure à la fréquence f de la tension appliquée aux bobines.

Expérience 2

3. f et n sont reliés par une relation de proportionnalité (la fonction est linéaire). 4. On peut faire varier la fréquence de rotation d’un moteur asynchrone en modifiant la fréquence de la tension qui alimente les pôles des bobines inductrices. On obtient une droite qui ne passe pas rigoureusement par l’origine. Cela est dû aux pertes liées à la conversion d’énergie électrique en énergie mécanique, frottement, …

CHAPITRE 6 - Comment faire varier la vitesse d’un véhicule électrique ? • 25

Les documents ■■Les variateurs électroniques • Réponses aux questions 1. La fréquence de rotation d’un moteur à courant continu se réalise par variation de la tension d’induit (cette façon est la plus employée). On peut remplacer le hacheur par un potentiomètre mais sa commande serait plus délicate et de l’énergie thermique serait dégagée par effet joule abaissant ainsi le rendement de l’opération.

2. On peut agir sur la fréquence de la tension d’alimentation des bobines pour modifier la fréquence de rotation d’un moteur asynchrone. (En fait, on conserve U/f constant pour maintenir le flux constant.) Le hacheur réalise en courant continu la même fonction qu’un transformateur en courant alternatif (il fait évoluer la tension en réalisant une conversion dont le rendement est voisin de 1 ; ce n’est pas le cas du potentiomètre).

Les exercices Tester ses connaissances

E (V) 30

Q.C.M.

25

1 : B ; 2 : A et B ; 3 : A et B ;

20

4 : A et C ; 5 : A et C ; 6 : B et C.

15

Tester ses capacités

10 5

1. Moteur Leroy-Somer

0

a. ℳR = f (I)

n (tr/s) 0

50

100

150

200

250

300

b. E est proportionnelle à la fréquence de rotation n

 (N.m) 20

car l’ajustement du nuage de points est une droite qui passe par l’origine du repère (la fonction E = f (n) est linéaire : E ≈ 0,11 × n).

4. Moteur asynchrone à vide f = 2 soit f = 2n. n b. La vitesse du moteur varie proportionnellement à la fréquence de la tension d’alimentation des bobines du stator.

a. On a 0

I (A) 0

5

10

15

20

b. Lorsque le couple résistant augmente, l’intensité du courant appelé par le moteur à courant continu augmente proportionnellement.

5. Moteur asynchrone en charge a. Pour ∆ℳ = 28 N.m, on a ∆n = 60 tr/min.

2. Modélisation

b.

a. U : tension d’alimentation du moteur ; I : intensité de courant appelé par le moteur ; E : f.e.m. du moteur ; R : résistance interne du moteur ; RI : chute de tension due à la résistance interne du moteur.

b. On a la relation U = E + R.I c. E = U – R.I = 120 – 10 × 0,25 = 120 – 2,5 = 117,5 V.

3. À l’atelier a. Représentation graphique de E = f(n)

∆n 60 = = 0,04 soit 4 %. n 1500 c. La fréquence de rotation du moteur est inférieure de 4 % à la fréquence de synchronisme. Elle en reste donc voisine : on dit qu’il y a glissement.

Appliquer le cours 6. Résoudre l’exercice : moteur de concasseur a. Fréquence de synchronisme : ns = 

26 • CHAPITRE 6 - Comment faire varier la vitesse d’un véhicule électrique ?

f 60 = = 20 tr/s, p 3

soit 1 200 tr/min. ∆n = ns – n = 1 200 – 1 150 = 50 tr/min. ∆n 50 = = 0,042 soit 4,2 %. Le glissement g =  n 1200 b. Vitesse angulaire nominale :

60 ×1000 = 50 Hz. 1200

7. Ventilateur a. La droite qui représente ℳM = f (n) passe par A(25 ; 0) et B(22,2 ; 20).

b. ℳR = 0,028n2 donc la caractéristique est un arc de parabole.

M B P

14,7 10

 R = 0,028 n2

5 0

A 0

21

19

21

n (tr/min)

484

967

967

967

500

f (Hz)

25

50

50

50

26

b. Couple moteur : P 12 × 103 × 60 = = 119 N.m. Ω 2 × π × 967

Pour n = 1 400 tr/min, on a : E1 = U – R.I = 240 – 1,5 × 16 = 216 V. Avec 150 V et 12 A, on a E2 = 150 – 1,5 × 12 = 132 V. Parce que la force électromotrice et la fréquence de rotation du moteur sont proportionnelles, on peut écrire (à flux constant) : E1 n1 E .n 132 ×1440 = soit n2 =  2 1 = = 880 tr/min. E 2 n2 E1 216

11. Au spectacle a. La relation de proportionnalité qui lie n et f est : 1200 n × f = 30 × f   soit f =  40 30 avec n en tr/min et f en Hz. b. Réglage de la fréquence :

30

20

10

n = 

 M (N.m)

25

4

10. Véhicule électrique

P 17300 = = 144 N.m. Ω 120 c. On désire n2 = 1 000 tr/min. La fréquence de la tension se fera pour ℳR = 

35

2

ℳm = 

1150 = 120 rad/s. Ω = 2π.n = 2π ×  60

f 2 = 

d (m)

22

22,9

24

25

n (tr/s)

Spectateur(s)

0

1 à 80

80 à 200

250 à 500

n (tr/min)

175

610

910

1 490

n (Hz) f = 30

5,8

20,3

30,3

49,7

c. Sa vitesse de synchronisme est de 1 500 tr/min soit 25 tr/s. On a alors p = 

f 50 = = 2. C’est un moteur n 25

c. Coordonnées du point de fonctionnement :

tétrapolaire (2p = 4).

P(22,9 ; 14,7). La fréquence de rotation du rotor est de 22,9 tr/s soit 1 374 tr/min. Son couple moteur est : ℳM = 14,7 N.m.

12. Pour chercher : a. Fonction des différents éléments du moteur de la

Exercices à caractère professionnel 8. Enrouleuse et pompe volumétrique a. Couple de démarrage du moteur asynchrone : ℳdm = 10 N.m.

b. Pour la pompe volumétrique ℳp = 0 N.m on a

ℳdm > ℳdp : le démarrage du moteur est possible. Pour l’enrouleuse ℳde = 17 N.m on a ℳdm < ℳde : le moteur ne démarre pas. c. Point de fonctionnement (1 460 ; 5) ; la fréquence de rotation est de 1 460 tr/min et le couple correspondant est 5 N.m. d. Puissance mécanique : Pmec = ℳ  .   Ω = 5 × 2π × 

1460 = 765 W. 60

9. Palan de grue a. Fréquence de la tension d’alimentation :

page d’entrée et de son électronique : 1) le boîtier d’interconnexion permet de distribuer le courant au moteur et à tous les composants. Il sert de relais pour le courant de charge rapide. 2) Le chargeur adapte le courant pour alimenter la batterie d’accumulateurs au lithium – ion, ce qui permet une autonomie importante de l’ordre de 160 km actuellement. 3) L’onduleur est utilisé pour alimenter et commander le moteur. Il convertit le courant continu en courant alternatif triphasé (un convertisseur assure l’alimentation du réseau accessoire 14 V). 4) Le réducteur remplace ici la boîte de vitesse : il permet d’adapter la vitesse du véhicule à la vitesse de rotation du moteur et de démultiplier la fréquence de rotation de l’arbre moteur. 5) Le moteur triphasé convertit l’énergie électrique en énergie cinétique. Sa puissance maximale : 70 kW (95 ch) ; fréquence de rotation maximale : 12 000 tr/min ; rendement : 95 %.

CHAPITRE 6 - Comment faire varier la vitesse d’un véhicule électrique ? • 27

6) Le superviseur assure la communication entre l’électronique du moteur et les différents éléments (batterie, réducteur, conducteur, ..).

b. Différents dispositifs de traction sont étudiés pour répondre : - à la raréfaction des carburants issus des énergies fossiles (pétrole, gaz) ; - à la diminution de la pollution et des nuisances urbaines ; - aux problèmes liés à l’augmentation des émissions des gaz à effet de serre (GES). Connu depuis 1890, le véhicule électrique se développe aujourd’hui. On distingue : 1) la traction électrique classique correspondant au système : batteries → gestion électronique du courant → moteur électrique → roues. Afin de répartir la masse, les batteries sont constituées de plusieurs blocs disposés à différents endroits du véhicule. 2) La traction bi-mode correspond à la traction thermique classique hors des villes et à la traction électrique classique en ville. 3) La traction hybride : un moteur thermique et un moteur électrique sont associés pour agir ensemble, en complément ou séparément. Au freinage ou

en décélération, l’énergie cinétique de la voiture est convertie en énergie électrique par le moteur qui se transforme en générateur pour recharger la batterie de traction.

13. Adapter un moteur de jouet (Démarche d’investigation) I On peut utiliser un montage I1 C potentiométrique U pour obtenir une UM I2 M tension réglable UM à partir d’une tension d’entrée disponible à la sortie d’une batterie ou d’un adaptateur par exemple. En déplaçant le curseur C, le potentiomètre fonctionne comme diviseur de tension mais cette solution présente deux inconvénients majeurs outre l’encombrement : - on utilise un conducteur ohmique donc dissipation de chaleur par effet joule (perte) ; - de plus, si la charge du moteur augmente, la fréquence de rotation du moteur va diminuer, voire s’effondrer. En effet, si le couple résistant augmente, l’intensité appelée I1 augmente entraînant ainsi une chute de la tension.

7 Pourquoi le métal semble-t-il plus froid que le bois ? CME4

les Activités ■■Activité 1 Tous les matériaux conduisent-ils la chaleur de la même façon ? Expérience 1

• Matériel Trois fils de métaux différents : fer, cuivre et aluminium - Une plaque de cuivre Un bec Bunsen - Cavaliers de paraffine

Expérience 2 

• Matériel Une tige de bois, une tige de fer - Deux manchons de papier - Un bec Bunsen

• Réponses aux questions Expérience 1

1. Les cavaliers commencent à fondre sur les fils, près de la flamme du bec Bunsen.

28 • CHAPITRE 7 - Pourquoi le métal semble-t-il plus froid que le bois ?

2. Les cavaliers ne fondent pas tous en même temps. À un instant donné, le nombre de cavaliers fondus est le plus grand sur la tige de cuivre et le plus petit sur la tige de fer.

3. Le métal qui conduit le mieux la chaleur est le cuivre, puis l’aluminium, puis le fer. Expérience 2

4. Le manchon de papier commence à brûler sur la tige de bois. Un manchon identique sur la tige de cuivre et chauffé pendant le même temps est à peine roux. Le métal conduit mieux la chaleur que le bois.

5. Lorsqu’on pose le pied nu sur un carreau, de la chaleur est transférée du pied au carreau. Cette chaleur, conduite ensuite par le carreau est évacuée dans l’espace environnant. Le pied qui se refroidit éprouve une sensation de froid. Plus le matériau du carreau est conducteur de la chaleur, plus cette évacuation est rapide et plus la sensation de froid est grande. La sensation de froid est plus intense avec un carreau de métal qu’avec un carreau de bois.

■■Activité 2 La quantité de chaleur fournie à un corps dépend-elle de la nature de ce corps ? • Matériel Ballon et chauffe-ballon - Thermomètre électronique - Support et noix Chronomètre - Eau, pétrole, huile

• Réponses aux questions 1. Pour une même masse et une même élévation de température, la durée de chauffage décroît de l’eau au pétrole et à l’huile. La quantité de chaleur fournie dépend de la nature du corps.

2. Lorsqu’on fournit de la chaleur à un corps, on n’élève pas toujours sa température. Lorsque ce corps est pur, il peut changer d’état physique à température constante (fusion, vaporisation, sublimation).

Les exercices Tester ses connaissances

Tester ses capacités

rience, la durée de chauffage est deux fois plus faible avec le pétrole qu’avec l’eau. À puissance égale fournie, cela montre que la quantité de chaleur fournie est deux fois plus faible avec le pétrole. On peut en déduire que la capacité thermique massique du pétrole est deux fois plus faible que celle de l’eau, ce que l’on vérifie dans les tables de la page 76 du manuel.

1. Décrire une expérience 1. a. On chauffe une masse d’eau avec un chauffe-

2. De l’eau et de l’huile a. On a fourni la même quantité de chaleur à la

ballon pendant des durées différentes.

même masse m de liquide : Q = m.ce.∆θe = m.ch.∆θh. ∆θh = 3∆θe . Donc ∆θh/∆θe = ce/ch = 3 b. Lorsqu’on transfère de la chaleur à un corps, l’élévation de sa température est grande lorsque sa capacité thermique massique est faible.

Q.C.M. 1 : A et C ; 2 : B ; 3 : C ; 4 : B et C ; 5 : A et C ; 6 : C ; 7 : A, B et C.

b. L’élévation de température est de 10 °C. 2. a. Pour les deux expériences, les masses et les élévations de température sont les mêmes.

b. Les durées sont différentes et il s’agit de deux liquides différents. Les durées sont différentes, car les quantités de chaleur fournies sont différentes. Pour une même masse de liquide et une même élévation de température, la chaleur fournie dépend de la nature du liquide.

3. Il faut consulter la table des capacités thermiques massiques exprimées en J/(Kg.°C). D’après l’expé-

3. Grandeurs et unités a. Q : quantité de chaleur ; m  : masse ; c : capacité thermique massique, θ1 et θ2 : températures. b. Q en joule (J) ; m en kilogramme (kg) ; c en joule par kilogramme et par degré (J/(kg.°C)), θ1 et θ2 en degré Celsius (°C).

CHAPITRE 7 - Pourquoi le métal semble-t-il plus froid que le bois ? • 29

c. La capacité thermique massique caractérise le matériau. d. La formule n’est pas utilisable pour un corps pur qui change d’état physique, car dans ce cas la température ne varie pas.

4. Reconstituer la chronologie d’une expérience a. Ordre des schémas : c) ; a) ; b). b. Initialement l’eau chaude se trouve dans le cristallisoir.

c. Dans l’état final, les températures sont égales dans l’eau du cristallisoir et celle du ballon.

d. Voir schéma :

rimentale (50 °C), en raison des fuites thermiques vers l’extérieur des blocs.

7. Loi de Fourrier a. D’après la relation de Fourrier, on peut extraire la conduction thermique λ : e .Φ A.(θ 2 − θ1 ) λ s’exprime donc en (J/s)/ (m.°C) ou en W/ (m.°C). b. Le flux thermique est : - divisé par 2 si la paroi a son épaisseur doublée ; - triplé si la différence de température entre les deux parois est triplée ; - diminué si l’on remplace le matériau par un autre de plus faible conductivité thermique.

λ=

8. Chauffons l’eau

transfert thermique

Appliquer le cours 5. Conduction thermique des métaux a. Les têtes d’allumette s’enflamment, car les branches métalliques de l’étoile conduisent la chaleur produite par la flamme du bec Bunsen. b. Ordre d’allumage des allumettes : cuivre, aluminium, zinc, fer selon l’ordre décroissant des coefficients de conduction thermique.

On compare les différents schémas au premier. - Calcul de t1 : L’écart de température passe de 20 °C à 40 °C avec la même masse ; il est donc doublé. La quantité de chaleur à fournir est donc doublée et la durée de chauffage aussi. Donc t1 = 2 × 4 = 8 minutes. - Calcul de t2 : La masse est divisée par 2 avec la même variation de température. La quantité de chaleur à fournir est donc divisée par 2 et la durée de chauffage aussi. Donc t2 = 4/2 = 2 min. - Calcul de t3 : La masse est divisée par 2 et l’écart de température est doublé. La quantité de chaleur à fournir est donc inchangée et la durée de chauffage aussi. Donc t3 = 4 min.

9. Capacité thermique massique du cuivre a. Quantité de chaleur reçue par l’eau : Q = 0, 4 × 4 180 × (17,3- 16,3) = 1 672 J.

6. Étude expérimentale d’un transfert

b. La quantité de chaleur perdue par le cylindre de

thermique par conduction a. On a calorifugé la valise pour ne considérer que

cuivre est égale à la quantité de chaleur reçue par l’eau : Q = 1 672 J. c. Q = 79,3 × 10–3 × (71,0 – 17,3) c = 1 672, c désignant la capacité thermique massique du cuivre. Soit 4,26c = 1 672. Donc c = 1 672/4,26 = 392,5 ≈ 390 J/(kg.°C).

les échanges thermiques entre les deux blocs en acier inox. b. La température du bloc 1 décroît, car il cède de la chaleur au bloc 2. c. La température du bloc 2 croît, car il reçoit de la chaleur du bloc 1. d. Il s’agit d’un transfert par conduction, car les deux blocs sont au contact l’un de l’autre. e. Les températures tendent vers une valeur commune pour atteindre l’équilibre thermique. f. Durée pour atteindre l’équilibre thermique de l’ordre de 200 s = 3 min 20 s. La température d’équilibre est 50 °C. g. Température d’équilibre théorique θe : θ +θ On a l’égalité : c(θ1 – θe) = c(θe – θ2). D’où θe =  1 2 2 θe =  79,0 + 26,0 = 52,5 °C. 2 On observe une légère différence avec la valeur expé-

Exercices à caractère professionnel 10. Résistance thermique d’une paroi a. Paroi

placoplâtre

polystyrène

béton

Épaisseur e en m

1 × 10–2

8 × 10–2

14 × 10–2

Coefficient de conductivité thermique λ en W/ (m.°C)

0,46

0,039

1,75

Résistance thermique R en m2/(W.°C)

0,02

2,05

0,08

30 • CHAPITRE 7 - Pourquoi le métal semble-t-il plus froid que le bois ?

Résistance thermique du mur : 0,02 + 2,05 + 0,08 = 2,15 m2/(W.°C)

b. Résistance thermique du bois remplaçant le béton : 0,08 m2/(W.°C) Épaisseur du bois : e = R.λ = 0,08 × 0, 15 = 0,012 m = 12 mm.

c. On voit qu’une épaisseur de 14 cm de béton peut être remplacée par 12 mm de bois. le bois est un isolant thermique bien plus efficace que le béton.

11. Performance énergétique d’une maison a. Consommation en kilowatt de la maison Fioul : 1 500 × 10/ 1 000 = 150 kWh Électricité : 9 000 kWh Total : 9 150 kWh

b. Consommation forfaitaire pour la production d’eau chaude : 4 000 kWh.

Énergie annuelle hors la production d’eau chaude : E = 9 150 – 4 000 = 5 150 kWh. c. Indice énergétique de la maison : I = E/S = 5 150 / 100 = 51,5 kWh/m2/an d. L’isolation thermique est bonne. Il s’agit d’une maison à faible consommation. Il n’y a pas lieu d’envisager des travaux d’isolation supplémentaires.

12. Un problème de météorologie L’eau possède une forte capacité thermique massique. Donc pour une même quantité de chaleur, la variation de température de l’eau est beaucoup plus faible que pour d’autres liquides. Les océans jouent le rôle d’un tampon thermique dans les régions côtières en atténuant les températures par temps chaud et en limitant les baisses de température par temps froid. Dans les régions côtières, il fait moins chaud l’été et moins froid l’hiver qu’à l’intérieur des terres.

8 Comment se chauffer ? CME4 Les activités ■■Activité 1 Comment utiliser l’électricité pour chauffer un liquide ? • Matériel - Deux thermoplongeurs de puissances 500 W et 1 000 W  - Un bécher 500 mL  - Un thermomètre électronique ; un compteur électrique portatif ; un chronomètre.

• Réponse aux questions 1. Le thermoplongeur transforme l’énergie électrique en énergie thermique. 2. La relation entre l’énergie électrique consommée E, la puissance P et la durée de fonctionnement t est E = P . t.

3. À partir de 250 mL d’eau à 20 °C, et pour atteindre la température de 100 °C, on constate que les énergies électriques consommées par les deux thermoplongeurs sont voisines de 20 Wh.

4. Pour faire bouillir un litre d’eau, une bouilloire de 2 200 W consomme autant d’énergie qu’une autre de 1 500 W.

■■Activité 2 Comment utiliser un hydrocarbure pour chauffer un liquide ? • Matériel - Deux flacons de 250 mL ; un entonnoir ; une trompe à eau  - Un briquet à gaz  - Tube, raccords, bouchons ; papier aluminium ; une boîte à boisson en aluminium  - Un thermomètre électronique. CHAPITRE 8 - Comment se chauffer ? • 31

Produits Sulfate de cuivre anhydre ; eau de chaux ; bougie sur un support.

Expérience 1 Dans le 1er flacon placer le tube d’arrivée des gaz pratiquement au contact du sulfate de cuivre anhydre. L’eau formée lors de la combustion apparaît au début sous forme de buée se déposant sur l’entonnoir. Peu à peu, le sulfate de cuivre anhydre bleuit aux abords du tube. L’eau de chaux se trouble.

Expérience 2 On a photographié l’expérience avec le papier d’aluminium découpé afin d’observer le montage expérimental. Partir d’eau à 10 °C.

• Réponse aux questions Expérience 1

1. La coloration bleue du sulfate de cuivre anhydre met en évidence la présence d’eau. 2. C’est le dioxyde de carbone qui trouble l’eau de chaux. 3. L’équation de la combustion s’écrit : Expérience 2

2 C4H10 + 13 O2 → 8 CO2 + 10 H2O

4. L’eau reçoit de l’énergie thermique car sa température s’élève. 5. Cette énergie thermique provient de la combustion de la paraffine.

Les documents ■■Au cœur des villes, un chauffage écologique à partir du réseau d’assainissement • Réponse aux questions 1. Les avantages d’utiliser le réseau des égouts comme système de chauffage sont nombreux : il est sans nuisance et sans danger, sans flamme, il ne rejette pas de gaz à effet de serre et contribue à diminuer la facture énergétique.

2. Dans la ville de Zurich, l’économie annuelle en énergie fossile est de l’ordre de 540 000 litres de fuel domestique. 3. La réduction des émissions de CO2 est de l’ordre de 1 400 tonnes par an.

Les exercices Tester ses connaissances Q.C.M. 1 : B ;  2 : A et C ;  3 : B ;  4 : A et B ;  5 : C ;  6 : C.

A

A

C

V

W

B D R

Tester ses capacités 1. Le montage expérimental On peut utiliser deux montages :

32 • CHAPITRE 8 - Comment se chauffer ?

montage voltmètreampèremètre

montage avec un wattmètre

2. Le fer à repasser a. La puissance dissipée par effet joule est : U2 236 2 55696 =  = = 1 989 W. 28 R 28 b. L’énergie dissipée par effet joule est : U 2 .t 55696 × 2,5 E =  =  = 4 973 Wh. R 28 P = 

3. La plaque signalétique d’un grille pain Les grandeurs électriques indiquées sur la plaque signalétique du grille-pain sont : - la tension efficace d’utilisation en volt : 220-240 V - la puissance absorbée en watt : 1850-2200 W ; - la fréquence du courant du secteur en hertz : 50/60 Hz.

4. Combustion de l’alcool à brûler a. Schéma du montage : a)

b)

sulfate de cuivre anhydre

eau de chaux

aspiration

coton

b. Le sulfate de cuivre anhydre permet de caractériser l’eau : il bleuit à son contact. L’eau de chaux permet de caractériser le dioxyde de carbone. Elle se trouble en sa présence.

5. Le brûleur à gaz La combustion du méthane libère de l’énergie thermique car la température de l’eau s’élève, celle-ci se met à bouillir et passe à l’état de vapeur d’eau.

6. Combustion du propane L’équation de combustion du propane s’écrit : C3H8 + 5 O2 → 3 CO2 + 4 H2O

Appliquer le cours 7. À vous de résoudre La masse d’heptane consommée aux 100 km est : m = 720 × 7,5 × 10–3 = 5,4 kg = 5 400 g. L’équation de combustion complète de l’heptane s’écrit : C7H16 + 11 O2 → 7 CO2 + 8 H2O La masse molaire de l’heptane est : M(C7H16) = 7 M(C) + 16 M(H) = 84 + 16 = 100 g/mol. Le nombre de moles d’heptane consommées aux 5400 100 km est : n =  = 54 mol. 100

L’équation chimique montre que lorsqu’une mole d’heptane est consommée, il se dégage 7 moles de CO2. Il s’est donc dégagé : 7 × 54 = 378 moles de CO2. La masse molaire du CO2 est : M(CO2) = M(C) + 2M(O) = 12 + 32 = 44 g/mol. Il s’est donc dégagé : 44 × 378 = 16 632 g de CO2 aux 100 km soit 166, 32 g/km. L’acheteur devra donc payer un malus puisque le dégagement de CO2 est supérieur à 160 g/km.

8. Le radiateur électrique a. La relation entre la puissance électrique P, la tension U et l’intensité I est P = U . I. b. L’intensité efficace du courant qui traverse le radiateur est : P 3000 I =  =  = 13 A. U 230 c. La relation entre l’énergie électrique consommée E et la puissance électrique P est E = P . t. d. Le temps d’utilisation en heures est : 20 1 (20 minutes = h =  h) 60 3 1 7 2 h 20 min = 2 +  =  h. 3 3 L’énergie électrique consommée par le radiateur est : 7 E = 3 000 × = 7 000 Wh = 7 kWh. 3

9. Économies d’énergie pour la planète a. La puissance des lampes à incandescence est P1 = 3 × 100 W + 3 × 75 W + 3 × 40 W = 645 W. L’énergie électrique consommée en 365 jours est : 645 × 2 × 365 = 470 850 Wh ou 471 kWh. b. La puissance des lampes basse consommation est : P2 = 3 × 20 W + 3 × 15 W + 3 × 9 W = 132 W. L’économie d’énergie réalisée en 365 jours est (645 – 132) × 2 × 365 = 374 490 Wh = 374,5 kWh.

10. Téléviseur en veille a. Lorsqu’on regarde la télévision 4 heures par jour, la dépense d’énergie électrique pendant 365 jours est : E = 100 × 4 × 365 = 146 000 Wh = 146 kWh. b. Le coût est : 146 × 0,1 = 14,60 €. c. Lorsque le téléviseur reste en veille 20 h par jour, la dépense d’énergie électrique pendant 365 jours est : E = 10 × 20 × 365 = 73 000 Wh = 73 kWh. d. Le coût de cette énergie gaspillée est : 73 × 0,1 = 7,30 €.

11. La cafetière de camping-car a. La cafetière reçoit de l’énergie électrique. b. Cette énergie électrique est transformée en énergie thermique.

c. La puissance P dissipée par effet joule est : P = 

U2 122 144 =  = = 360 W. R 0,4 0,4

CHAPITRE 8 - Comment se chauffer ? • 33

d. L’énergie dissipée par effet joule pendant 10 minu10 1 tes est (10 minutes = h =  h) : 60 6 E = 

U 2 .t 144 =  = 60 Wh. R 0,4 × 6

40 500 = 11,25 kWh = 11 250 Wh. 3600 b. Nombre d’heures d’alimentation : 11 250 = 112,5 heures. 100

12. Chaleur dégagée par la combustion de l’alcool à brûler a. La quantité de chaleur nécessaire pour chauffer la

16. Retour sur investissement a. On a dû changer 8 fois la lampe à incandescence

boîte à boisson est : Q1 = 100 × 0,897 × (80-20) = 5 382 J. b. La quantité de chaleur nécessaire pour chauffer l’eau contenue dans la boîte à boisson est : Q2 = 400 × 4,18 × (80-20) = 100 320 J. c. La quantité de chaleur nécessaire pour chauffer l’ensemble boîte et eau est : Q = Q1 + Q2 = 105 702 J. d. Le pouvoir calorifique de l’alcool à brûler est : 105 702 = 30 200 J/g. 3,5

b. Le coût de l’énergie consommée est :

13. Combustion du pentane a. L’équation bilan de la réaction s’écrit : C5H12 + 8 O2 → 5 CO2 + 6 H2O

b. Masse molaire du pentane :

M(C5H12) = 5M(C) + 12M(H) = 72 g/mol. 3,6 3,6 g de pentane représentent : = 5 × 10–2 mole. 72 L’équation montre que la combustion de 1 mole de pentane nécessite 8 moles de dioxygène. La combustion de 5.10–2 mole de pentane nécessite 8 × 5 × 10–2 = 4.10–1 mole de dioxygène. Soit un volume de 24 × 4.10–1 = 9,6 L de dioxygène.

14. Réchauffer l’eau de la piscine a. Puisque l’énergie électrique est totalement transformée en énergie thermique, la valeur reçue par l’eau en un jour est Q = 3 000 × 24 = 72 000 Wh = 72 kWh = 72 × 3 600 kJ b. Le coût journalier du chauffage est 72 × 0,1 = 7,20 €. c. L’augmentation de la température de l’eau est : (1 m3 d’eau a une masse de 103 kg et 100 m3 une masse de 105 kg et la capacité massique de l’eau est 4,18 kJ/kg.°C). Q 72 × 3600 Δθ =  =  = 0,62 °C. m.c 4,18 ×105

15. Valorisation énergétique

soit un coût 8 × 0,5 = 4 €. 0,075 × 8 000 × 0,1 = 60 €.

c. La dépense totale est 60 + 4 = 64 €. d. Le prix d’achat de la lampe est 9 €. e. Le coût de l’énergie consommée est : 0,015 × 8 000 × 0,1 = 12 €. f. La dépense totale est 12 + 9 = 21 €. g. L’économie réalisée est 64 – 21 = 43 €.

Exercices à caractère professionnel 17. Les chaudières à plaquettes ou à granulés de bois a. Le combustible utilisé est soit les plaquettes soit les granulés de bois.

b. La chaudière est alimentée par une vis sans fin. c. La consommation moyenne d’une maison de 100 m2 moyennement isolée est de 20 000 kWh. L’économie réalisée entre les granulés de bois et le gaz naturel est : (0,0946 – 0,075) × 20 000 = 392 €.

18. Et si on se chauffait avec le soleil ? a. Dans ce type de maison, le circuit de chauffage est situé dans le sol.

b. En cas d’ensoleillement insuffisant, c’est une chaudière qui produit de la chaleur.

c. Le montant de l’économie est 4 600 × 0,1 = 460 €.

19. Incinération des déchets Nombre d’habitants que l’on pourrait alimenter : 140 ×106 = 140 000. 1000

20. Situation problème C’est Séverine qui a raison car R =  Si P = 1 000 W, R = 

230 2 = 52,9 Ω. 1000

Si P = 2 000 W, R = 

230 2 = 26,45 Ω. 2000

a. Le nombre de kWh dégagé par la combustion de 1 kg de polystyrène est :

34 • CHAPITRE 8 - Comment se chauffer ?

U2 P

9 Comment économiser l’énergie ? CME5 Les activités ■■Activité 1 Les matériaux ont-ils des pouvoirs isolants ou conducteurs de la chaleur différents ? • Matériel - Plaques de bois d’épaisseur 2, 4, 6 et 8 mm  - Plaques de différents matériaux (polystyrène, liège, bois, verre …) de même épaisseur (8 mm)  - Un thermomètre électronique ; une lampe de puissance 100 W.

• Réponse aux questions Expérience 1

1. Les distances d1 et d2 doivent être conservées pour se placer dans les mêmes conditions expérimentales et pouvoir ainsi comparer les résultats.

2. La plaque qui s’oppose le plus au transfert de chaleur est la plaque la plus épaisse. Expérience 2

3. Parmi les quatre matériaux choisis dans la leçon, c’est le verre qui favorise le plus le transfert de chaleur.

4. La plaque qui la plus grande résistance thermique est celle en polystyrène. 5. Le matériau le plus isolant est le polystyrène. 6. On peut conclure que les matériaux ont des pouvoirs isolants différents.

■■Activité 2 Comment établir le bilan énergétique de la salle de classe ? • Réponse aux questions 1. La surface du plafond et du sol est 80 m² ; la surface des 2 portes est 4 m² ; la surface des 5 fenêtres est 5 m² ; la surface des murs est (25 – 4) + (25 – 5) = 41 m². 2. Le flux thermique pour chaque paroi est : 80 ×14 80 ×14 = 560 W. Plafond : = 373 W. 2 3 5 ×14 4 ×14 = 140 W. Portes : = 43 W. Fenêtres : 0,5 1,3 Le flux total pour la classe est donc de : 560 W + 373 W + 287 W + 140 W + 43 W = 1 403 W Sol :

Murs :

41×14 = 287 W. 2

3. La puissance minimale que doivent fournir les radiateurs pour équilibrer les pertes thermiques par conduction est de l’ordre de 1 400 W ou 1,4 kW.

4. Si les radiateurs fonctionnent 6 mois de 30 jours dans l’année et 15 heures par jour, la consommation minimale d’énergie dépensée pour chauffer la salle de classe est : E = P . t = 1,4 × 6 × 30 × 15 = 3 780 kWh.

5. Pour établir un bilan énergétique plus précis, il faudrait tenir compte des transferts thermiques par convection, par rayonnement, par pont thermique.

CHAPITRE 9 - Comment économiser l’énergie ? • 35

Les documents ■■DOC 1 : les chaudières a condensation • Réponse aux questions 1. La valeur de la chaleur latente de condensation de l’eau est 2 258 kJ/kg. 2. Les pourcentages des pertes en énergie dans une chaudière classique sont de l’ordre de 18 %. Dans une chaudière à condensation ils sont de l’ordre de 3 %.

3. Les pertes d’énergie dans les fumées sont plus importantes dans une chaudière classique que dans une chaudière à condensation car elles sont rejetées directement dans l’atmosphère. Dans une chaudière à condensation, une partie de l’énergie des fumées a servi à réchauffer l’eau du circuit de chauffage.

4. énergie latente non utilisée

111 % par rapport à PCS

111 % par rapport à PCS

1,5 %

pertes dans les fumées

1%

0,5 % pertes par

rayonnement

CHAUDIÈRE À CONDENSATION

rendement G97 %

CHAUDIÈRE CLASSIQUE

rendement G82 %

énergie latente non utilisée

11 %

pertes dans les fumées

6%

1%

pertes par rayonnement

■■DOC 2 : la maison « zen » • Réponse aux questions  a. La résistance thermique d’une paroi composée de plusieurs matériaux est égale à la somme des résistances thermiques de chaque matériau. La résistance thermique de la paroi présentée au doc. 1, composée de 9,4 cm de KLH et de 15 cm de liège est : e e 9,4 ×10 −2 15 ×10 −2 R =  1 +  2 =  + = 0,67 + 3,57 = 4,24 m².K/W λ1 λ2 0,14 0,042 b. Le triple vitrage de la maison « ZEN » a un coefficient de transmission surfacique environ 10 fois plus petit qu’un simple vitrage. Il est donc 10 fois plus performant. Ce coefficient est aussi 5,6 fois plus petit que celui d’un double vitrage. Il est donc plus de 5 fois plus performant.

36 • CHAPITRE 9 - Comment économiser l’énergie ?

Les exercices Tester ses connaissances

Appliquer le cours

Q.C.M. 1 : A et B ;  2 : A ;  3 : A ;  4 : A et C ;  5 : B et C.

8. Maîtriser les grandeurs physiques des thermiciens Matériau λ (W/m.K)

Tester ses capacités

Épaisseur e (m)

R (m².K/W)

Béton / 1,75

0,15

0,09

1. Plaque signalétique d’un appareil

Bois de sapin / 0,12

0,05

0,42

La puissance électrique des appareils est un paramètre qui renseigne le consommateur sur les capacités de l’appareil à fournir du travail ou de la chaleur. En revanche la plaque signalétique ne renseigne jamais sur l’énergie qui dépend du temps d’utilisation.

Brique monomur / 0,18

0,20

1,1

Laine de verre / 0,041

0,20

4,88

2. Plaquette pour un chauffage Il s’agit d’indications de puissance.

3. Abonnement électrique L’indication « abonnement 9 kW » représente la puissance électrique disponible. L’indication « consommation 1 320 kWh » représente l’énergie électrique consommée.

4. Radiateur de chauffage central a. L’énergie restituée dans la pièce pendant une heure de fonctionnement est : 1 200 Wh = 1,2 kWh.

b. Le rendement du radiateur : η = 

Er 1,2 =  = 0,78 = 78 %. E c 1,53

5. Résistance thermique du béton a. La formule de la résistance thermique d’un maté-

e λ R : résistance thermique en m².K/W ; e : épaisseur de la paroi en mètre (m) ; λ : conductivité thermique du matériau en W/m.K. b. La résistance thermique d’un mur en béton de 0,2 20 cm d’épaisseur est R =  = 0,11 m².K/W. 1,75 riau est : R = 

6. Résistance thermique d’un mur a. La résistance thermique du mur est : R = R1 + R2 + R3

b. la résistance thermique du mur est : 0,01 0,1 0,2 +  +  0,46 0,041 0,84 = 0,02 + 2,44 + 0,24 = 2,7 m².K/W R = 

7. Flux thermique à travers une paroi en bois a. Flux thermique à travers une paroi en bois : 0,15 ×10 × 20 = 300 W. 0,1 b. Le flux thermique est exprimé en watt. Φ = 

9. Économie d’énergie Les murs d’un atelier sont en bois de sapin d’épaisseur 5 cm dont la conductivité est λ = 0,15 W/m.K La surface des murs est de 100 m². La température extérieure est de 273 K et la température de l’atelier 291 K. a. Résistance thermique des murs non isolés : 0,05 R =  = 0,33 m².K/W. 0,15 b. Flux thermique à travers les murs non isolés : λ . A.(θ 2 − θ1 ) 0,15 ×100 ×18 Φ1 =  =  = 5 400 W. e 0,05 c. La résistance thermique des murs isolés est : 0,33 + 3,15 = 3,48 m².K/W. Le nouveau flux thermique des déperditions à travers les murs isolés est : A(θ 2 − θ1 ) 100 ×18 Φ1 =  =  = 517 W. R 3,48

10. Rendement d’un chauffe-eau solaire a. Le transfert de l’énergie du Soleil au panneau solaire s’effectue par rayonnement.

b. Pendant une heure, il a circulé 20 L d’eau soit une masse m = 20 kg. La quantité de chaleur absorbée par l’eau circulant dans le capteur est : Q = m.c (θ2-θ1) = 20 × 4,18 × 25 = 2 090 kJ 2090    =  = 0,58 kWh. 3600 c. La puissance thermique du chauffe-eau lors de l’essai d’une heure est 0,58 kW. d. La puissance solaire disponible est 800 × 2 = 1 600 W. L’énergie solaire « consommée » pendant une heure est Ec = 1 600 × 1 = 1 600 Wh = 1,6 kWh. Le rendement du chauffe-eau solaire est le rapport entre l’énergie restituée à l’eau Er et l’énergie solaire consommée Ec : E 0,58 η =  r =  = 0,36 soit 36 %. Ec 1,6

CHAPITRE 9 - Comment économiser l’énergie ? • 37

11. Nature du matériau constituant un mur a. Conductivité thermique λ de ce mur : Φe 2480 × 0,18 =  = 1,75 W/(m.K) A(θ 2 − θ1 ) 15 ×17 b. Ce mur est en béton plein. λ = 

Exercices à caractère professionnel 12. Double et triple vitrage a. La résistance thermique du vitrage 4-12-4 est : 2 × 0,004 0,012 +  0,81 0,025 = 0,010 + 0,480 = 0,490 m².K/W. La résistance thermique du vitrage 4-12-4-12-4 est : 3 × 0,004 2 × 0,012 +  R2 =  0,81 0,025 = 0,015 + 0,960 = 0,975 m².K/W. b. C’est le triple vitrage qui assure la meilleure isolation thermique. R1 = 

13. Épaisseur d’un isolant a. Épaisseur e (mm)

R (m².K/W)

U (W/m².K)

220

5,80

0,17

200

5,25

0,19

150

3,95

0,25

100

2,60

0,38

75

1,95

0,51

60

1,55

0,65

b. Représentation des variations de U en fonction de l’épaisseur e. c. Graphiquement, lorsqu’on passe d’une épaisseur de 60 à 70 mm le coefficient de transmission surfacique passe de 0,65 à 0,53 soit une diminution de 0,12 W/(m².K) : U (W/m2.K) 0,7 0,6 0,5 0,4

14. Utilisation d’abaque a. La conductivité thermique du matériau étudié au point A est 0,5 W/(m.K). Son épaisseur est 5 cm. Sa résistance thermique est 0,1 m².K/W b. L’épaisseur minimale de liège à placer est 12 cm. c. D’après l’abaque, la résistance thermique de cet isolant est de l’ordre de 1,1 m².K/W. D’après l’abaque, on doit rajouter 4 cm de laine de verre (c’est-à-dire une épaisseur au total de 90 mm) pour doubler cette résistance thermique et parvenir à R = 2,2 m².K/W. Le résultat est conforme à celui trouvé par le calcul : 0,045 R =  = 1,125 m².K/W. 0,04 Si on double cette résistance thermique on obtient : R = 2,25 m².K/W. Cette valeur correspond à une épaisseur de e = R.λ = 2,25 × 0,04 = 0,09 m soit 9 cm.

15. Diagnostic de performance énergétique a. La consommation en énergies finales de cette habitation est de 12 953 kWh par an. La consommation en énergie primaire est de 33 419 kWh. Elles sont données en kWh. b. Non, cette habitation ne possède pas une climatisation l’été puisqu’il n’y a pas d’énergie de refroidissement. c. La consommation conventionnelle de cette habitation est de 434 kWh/(m².an). Elle est donnée en kWh énergie primaire/(m².an). La superficie de cette 33419 habitation est de l’ordre de = 77 m². 434 d. L’estimation des émissions de CO2 est de 26 kg/ (m².an). Sa valeur pour la superficie totale de l’habitation est 77 × 26 = 2 002 kg.

16. Investigation

0,3 0,2 0,1 0

Lorsqu’on passe d’une épaisseur de 200 à 210 mm, le coefficient de transmission surfacique passe de 0,19 à 0,18 soit une diminution de 0,01 W/(m².K). d. Le plus grand gain d’isolation est obtenu pour une épaisseur de 60 mm. e. La forme de la courbe montre qu’en effet, ce sont les premiers centimètres qui sont les plus efficaces puisqu’une variation de 10 mm d’épaisseur entre 60 et 70 mm entraîne une variation environ dix fois plus importante qu’entre 200 et 210 mm.

0

0,1

0,2

e (m)

38 • CHAPITRE 9 - Comment économiser l’énergie ?

Les professionnels évoquent des rendements supérieurs à 100 % pour les chaudières à condensation car ils gardent pour référence le PCI, le pouvoir calorifique inférieur, qui est l’énergie thermique dégagée à l’exclusion de la chaleur latente de condensation de la vapeur d’eau qui s’échappe et que l’on considère irrécupérable.

10 Comment protéger l’environnement ? CME5 Les activités ■■Activité 1 Qu’est-ce qu’une pluie acide ? • Matériel - Une bouteille à combustion ; un ballon à fond plat ; une trompe à vide - Un têt à combustion ; des bouchons ; un tube de verre - Du soufre ; du bleu de bromothymol

• Réponse aux questions 1. La solution dans le ballon au début de l’expérience est neutre. (pH ≈ 7) 2. La dissolution du dioxyde de soufre acidifie la solution. Le pH de la solution diminue. 3. Lors de la combustion du soufre contenu dans le pétrole et ses dérivés, de l’oxyde de soufre se dissout dans l’eau présente dans l’atmosphère : cette eau devient acide.

4. La combustion des oxydes d’azote ou de produits chlorés contribue aussi à l’apparition des pluies acides en provoquant l’apparition dans l’atmosphère d’acide sulfurique et d’acide chlorhydrique.

5. Réduire la combustion des matières fossiles et de leurs dérivés (carburant, charbon, matières plastiques) permet de lutter contre l’apparition des pluies acides. Pour cela, il est conseiller de trier les déchets plastiques afin qu’ils soient recyclés plutôt que brûlés mais aussi d’utiliser autant que possible des moyens de locomotion n’utilisant pas de dérivés du pétrole.

■■Activité 2 Pourquoi et comment adoucir l’eau ? • Matériel - Des bandelettes de dureté ; des tubes à essai - Une solution d’oxalate d’ammonium

• Réponse aux questions Expérience 1

1. L’eau issue d’une résine échangeuse d’ions doit être douce. Expérience 2

2. On doit trouver du calcium dans l’eau du robinet. 3. La résine échangeuse d’ions abaisse la concentration en ions calcium et magnésium en les remplaçant par des ions sodium.

Expérience 3

4. L’eau d’un appareil de chauffage ne doit pas être dure pour ne pas entartrer les canalisations. 5. Adoucir l’eau consiste à enlever les ions calcium et magnésium du milieu. Le moyen le plus simple est d’utiliser des résines échangeuses d’ions.

Les documents ■■Le recyclage des matières plastiques en France • Réponse aux questions 1. La masse de déchets plastiques collectés en France augmente d’environ 10 % par an. 2. Les Français doivent notamment progresser dans la collecte des bouteilles plastiques.

CHAPITRE 10 - Comment protéger l’environnement ? • 39

3. Les différents polyéthylènes ajoutés au polypropylène constituent environ 73 % de la masse totale des plastiques recyclés.

4. 95 % de la Renault Modus sont recyclables et 20 % de la quantité de plastique utilisé constituent du plastique recyclé.

Les exercices Tester ses connaissances

7. Différencier quatre polymères. La matière flotte-t-elle sur l’eau ?

Q.C.M. 1 : C ;  2 : C ;  3 : C ;  4 : C ;  5 : B et C ;  6 : C ;  7 : A et C.

oui

Tester ses capacités

PE

non flamme verte au test de Belstein oui

1. Mesurer le pH

PVC

a. Il est possible de mesurer précisément le pH avec

non La matière est-elle attaquée par l’acétone ?

un pHmètre, moins précisément avec du papier pH. On peut aussi utiliser des indicateurs colorés pour déterminer la « zone de pH » d’une solution.

b. Voir le manuel.

non

oui

autres

PS

8. Quel est le bon logo ?

2. Calculer le pH

5

a. Le logo du polypropylène est b. Le logo numéroté 2 représente le Pehd (polyéthy-

a. pH = –log(2,5 × 10–5) = 4,6. b. pH = –log(2,5 × 10–9) = 8,6.

Produit

pH

Caractère acido-basique

lène haute densité) et celui numéroté 4 représente le Pebd (polyéthylène basse densité). Ils se différencient donc par leur densité.

Soda

2,5

acide

c. Le PVC (polychlorure de vinyle) produit un déga-

café

5

acide

gement de dichlore lorsqu’il brûle.

Lait

6,8

Quasi neutre, légèrement acide

Sang

7,5

basique

Soude

13

basique

3. Acide ou base ?

4. Dosage de HCl par NaOH

Son logo est :

Appliquer le cours 9. Recyclage des plastiques en France

a. VBE = 11 mL. b. CA × VA = CB × VBE  ; CA = 

3

C B ×V BE 10 −2 ×11×10 −3 =  = 1,1×10–2 mol/L. VA 10 ×10 −3

5. Exploitation de l’activité 2 La présence de calcium et/ou magnésium se traduit par un précipité blanc. C’est donc dans le tube de droite qu’étaient présents les ions Ca2+ et Mg2+.

6. Comparaison des duretés de plusieurs eaux Plus l’eau est dure moins la hauteur de mousse est importante. L’eau la plus dure est dans le tube 2 ; l’eau du tube 1 est un peu moins dure et l’eau du tube trois est la plus douce.

40 • CHAPITRE 10 - Comment protéger l’environnement ?

Symbole

Nom

%

masse recyclée en 2007 (en tonnes)

Pebd

Polyéthylène basse densité

24

246 000

PET

Polyéthylène téréphtalate

20

205 000

Pehd

Polyéthylène haute densité

15

153 750

PP

Polypropylène

14

143 500

PVC

Polychlorure de vinyle

6

61 500

PS

Polystyrène

5

51 250

ABS

Acrylonitrile Butadiène Styrène

2

20 500

PSE

Polystyrène expansé

1

10 250

PC

Polycarbonate

1

10 250

PA

Polyamide

1

10 250

PU

Polyuréthane

0

0

11

112 750

Autres (mélanges)

c. CA2 = 10–pH = 10–7,2 = 6,31 × 10–8 mol/L d. Quantité n1L à rajouter par litre : n1L = CA2 – CA1 ; Quantité nac à rajouter dans la piscine : nac = (CA2 – CA1) × 115 000 = 2,68 × 10–3 mol. 2,68 ×10 −3 = 1,165 × 10–3 mol/L V 2,3 pHsol pHmoins = –log C = –log (1,165 × 10–3) = 2,9.  C = 

a. Il n’y a pas de limite légale à la dureté de l’eau, car une eau très dure est toujours potable. b. Une eau très dure induit des problèmes d’entartrage des canalisations. c. 1 °F correspond à une concentration en ions calcium et magnésium de 10–4 mol/L. La valeur maximale de la concentration totale en ions calcium et magnésium des eaux d’Île de France, mesurée en 2008 est de 36 × 10–4 mol/L.

11. Apprendre à résoudre a. Équation de dissociation de l’acide chlorhydrique dans l’eau : HCl + H20 → H3O+ + Cl–

b. [H3O+] = 2 × 10–3 mol/L c. pH = –log([H3O+]) = 2,7. d. On peut diminuer le pH de la solution en augmentant la quantité d’ions H3O+ par litre de solution donc en versant une solution plus acide dans la solution de départ.

12. Analysons une bouteille d’eau C Mg 202 43 + =  + 4 2,4 4 4 = 50,5 + 17,9 = 68,4. b. L’eau de Vittel est très dure. c. Dans le réseau d’eau chaude, l’eau de Vittel boucherait très rapidement la canalisation. c. L’eau de Vittel est légèrement basique. d. Le symbole du polyéthylène téréphtalate est PET. C Ca

Le logo est

2+

2+

1

13. La couleur de l’hortensia a. Si la fleur est bleue, le sol est acide. b. Si la fleur est rose ou rouge, le pH est environ égal à 7.

c. Il faut acidifier la terre si on souhaite avoir des fleurs bleues.

Exercices à caractère professionnel 14. Entretien de l’eau d’une piscine a. Le produit contenu dans le bidon marqué « pH moins » est un acide (sulfurique en général).

b. CA1 = 10–pH = 10–7,4 = 3,98 × 10–8 mol/L

=

f. La quantité d’acide versé nac est deux fois moins

10. Qualité de l’eau

a. TH = 

nac

importante que la quantité d’ions oxonium apportée. nac = 

n 2,68 ×10 −3 = = 1,34 ×10 −3 mol . 2 2

15. Le rôle des résines échangeuses d’ions a. Une eau dure peut être à l’origine de l’entartrage de canalisations.

b. L’adoucissement d’une eau consiste à en enlever les ions calcium Ca2+ et magnésium Mg2+.

c. Une résine échangeuse d’ions échange les ions calcium et magnésium par des ions sodium. Cette eau peut donc être trop chargée en ions sodium pour des personnes devant suivre un régime alimentaire pauvre en sel. d. Un adoucisseur d’eau ne doit être installé que sur les circuits alimentant les équipements où l’eau est chauffée.

16. Dosage de la dureté totale d’une eau a. C EDTA .VEDTA = C Ca

2+

+ Mg 2+

.V

.V C 10 −2 × 5, 8 × 10 −3 donc C Ca 2+ + Mg 2+ = EDTA EDTA = V 20 × 10 −3 = 2,9 × 10–3 mol/L. b. 1 °F correspond à une concentration en ions calcium et magnésium de 10–4 mol/L. Ici, TH = 29 °F c. Une eau est très dure à partir de TH = 30 °F. Cette eau est donc assez dure. Il faut l’adoucir avant de l’utiliser dans des circuits de chauffage.

17. Démarche d’investigation a. Le pH de l’océan diminue. Il devient plus acide. b. Ce phénomène est en grande partie dû à la dissolution dans l’océan de grandes quantités de dioxyde de carbone. c. L’industrie et la consommation de masse provoquent la production de grandes quantités de dioxyde de carbone qui se dégage dans l’atmosphère puis est, en partie, absorbé par les océans. d. Cette acidification constitue une modification du milieu marin qui peut amener la disparition de nombreuses espèces animales. Cela pose notamment des problèmes pour la calcification des coquilles des crustacés.

CHAPITRE 10 - Comment protéger l’environnement ? • 41

11 Comment fonctionnent certains dispositifs de chauffage ? CME6

Les activités ■■Activité 1 Comment varie la pression d’un gaz en fonction de son volume ? De sa température ? Expérience 1

• Matériel  Une seringue graduée de 60 mL ; un manomètre électronique ; un tuyau vinyle

• Expérience Avec certains matériels, il faut tenir compte du volume d’air dans le tuyau en vinyle reliant la seringue au manomètre. Résultats relevés : p (hPa)

1 020

1 224

1 530

1 748

V (mL)

60

50

40

35

Expérience 2

• Matériel  Un manomètre électronique ; un thermomètre électronique Un chauffe-ballon ; un ballon avec bouchon à deux trous ; un tuyau vinyle.

• Expérience Résultats relevés : θ (°C)

21

36

51

70

85

p (hPa)

1 115

1 170

1 225

1 300

1 350

T (K)

294

309

324

343

358

p (Pa)

111 500

117 000

122 500

130 000

135 000

p T

379,2

378,6

378,1

379,0

377,1

• Réponse aux questions Expérience 1

1. La pression de l’air augmente quand le volume diminue, à quantité de matière constante et à température constante.

2. Le produit p.V est constant. Expérience 2

3. La pression augmente avec la température, à volume constant et à quantité de matière constante. p est constant, voisin de 378 Pa/K. T 5. La relation correcte est p.V = k.T, validée par les deux expériences.

4. Le quotient

42 • CHAPITRE 11 - Comment fonctionnent certains dispositifs de chauffage ?

■■Activité 2 Comment déterminer une pression ou une vitesse d’écoulement d’un fluide ? Expérience 1

• Matériel  Une éprouvette ; un réservoir cylindrique avec sortie à la base ; un support (boy) Un tuyau vinyle avec pince ; un chronomètre

• Expérience Diamètre intérieur du réservoir : D = 7,2 cm. Aire S de sa section droite : S = 41 cm2. V (cm3)

123

205

287

t (s)

4,4

7,3

10

Baisse de niveau

h (cm)

3

5

7

Débit volumique

V Q= (cm 3 /s) t

30

28,1

28,7

h (cm/s) t

0,7

0,7

0,7

v . S (cm3/s)

28,7

28,7

28,7

Volume d’eau écoulé Durée de l’écoulement

Vitesse moyenne d’écoulement

v=

Expérience 2

• Matériel  Un manomètre électronique ; un tube de Venturi percé de trous de diamètre 3 mm pour différentes sections ; un aspirateur ; un embout de gonflage de ballon.

• Expérience Les trous permettant les mesures de pression sont percés sur des sections de diamètres : 25 mm, 41 mm et 55 mm du tube de Venturi. Les mesures ont été réalisées sur la position 4 (sur 5) d’un aspirateur de débit théorique maximal de 49 dm3/s. Position des trous

1

Diamètre (mm) Section S (m2)

2

3

41

25

55

S1 = 1,3 × 10–3

S2 = 4,9 × 10–4

S3 = 2,4 × 10–3

Pression p (hPa)

p1 = 995

p2 = 968

p3 = 998

Vitesse d’écoulement v (m/s)

v1 = 26,1

v2 = 69,6

v3 = 14,2

On calcule d’abord v2 en appliquant la formule donnée dans le manuel : v2 = 69,6 m/s. On calcule ensuite le débit : Q = S2.v2 = 34 dm3/s. Ce débit est le même pour les trois diamètres. On en déduit v1 = Q/S1 et v3 = Q/S3

• Réponse aux questions  Expérience 1

1. Q ≈ v . S, soit 28,7 cm3/s environ. Expérience 2

2. La pression de l’air est la plus faible au niveau de la section du tube de plus faible diamètre : pour D = 25 mm, p = 968 hPa avec notre matériel.

3. La vitesse d’écoulement de l’air est la plus grande au niveau de la section du tube de plus faible diamètre, là où la pression est la plus faible : pour D = 25 mm, v = 69,6 m/s.

CHAPITRE 11 - Comment fonctionnent certains dispositifs de chauffage ? • 43

Les documents ■■Le chauffage central • Réponse aux questions 1. Les éléments chauffants sont : le plancher chauffant, les radiateurs à eau. 2. Énergie

Description du combustible

Énergie renouvelable (ou non)

Stockage du combustible

Coût de l’installation

Coût de fonctionnement

Bois

Bûches, Plaquettes, granulés

Oui et non polluante

Nécessaire et volumineux

Onéreux, amorti en 10 ans

Peu onéreux mais variable selon les régions

Énergie solaire

Pas de combustible

Oui, non polluante

Non

Très onéreux

Négligeable

Géothermie

Pas de combustible

Oui et non polluante

Non

Onéreux

Peu onéreux

Fioul

Liquide visqueux

Non, polluante

Oui, en citerne

Onéreux

Onéreux et en augmentation constante

Gaz naturel

Gaz sous pression, en citerne ou gaz de ville

Non, polluante

Oui dans le cas d’une citerne

Onéreux

Onéreux et en augmentation constante

Électricité

Pas de combustible

Non, non polluante

Non

Peu onéreux

Onéreux

■■La pompe à chaleur • Réponse aux questions 288 soit COP 19 . Cette valeur est très théorique et ne comprend pas les énergies 288 − 273 absorbées par les satellites : ventilateur, pompes, …. Le COP brut d’une pompe à chaleur est généralement compris entre 2 et 5.

1. COP 

2. Pour un COP de 3, la pompe à chaleur fournit 3 kWh lorsqu’elle absorbe 1 kWh électrique.

Les exercices Tester ses connaissances Q.C.M. 1 : C

2 : A, B et C

3 : C

4 : A

5 : B et C

6 : B et C.

Tester ses capacités 1. Mesures de pression a. Expérience 1 : patm = 998 hPa ou 99 800 Pa.

3. À température constante a. p (hPa)

880

1 007

1 173

1 760

V (mL)

40

35

30

25

20

p (Pa)

88 000

100 700

117 300

140 800

176 000

V (m3)

4 × 10–5 3,5 × 10–5 3 × 10–5 2,5 × 10–5 2 × 10–5

p.V (en Pa et m3)

3,52

3,52

3,52

b. Expérience 2 : pM = 1 027 hPa ou 102 700 Pa.

b. Le produit p.V est constant.

2. Cartouche de CO2

4. Lance à incendie

n.R.T p= = 6,19 ×107 Pa soit 619 bar. V

1 408

3,52

3,52

Section : S = 7,07 × 10–4 m2. Vitesse d’écoulement de l’eau : v = Q/S = 57 m/s.

44 • CHAPITRE 11 - Comment fonctionnent certains dispositifs de chauffage ?

5. Pompes hydrauliques 100 ×10 −3 a. Première pompe : Q =  = 8,3 × 10–4 m3/s . 2 × 60 b. Seconde pompe : Section : S = 1,26 × 10–3 m3 et le débit est : Q = v.S = 6,3 × 10–4 m3/s soit 2,3 m3/h.

6. Vitesses et pressions dans un tube de Venturi a. p1 – p2 = 12 hPa = 1 200 Pa. b. Vitesse d’écoulement de l’air :

1,3 ×(1 −

(3 ×10 −4 )2 −4 2

(12 ×10 )

≈ 44,4 m/s )

c. La pression est plus faible et la vitesse d’écoulement plus grande lorsque le diamètre de la section du tube est plus petit.

7. Tube de Pitot a. pA – pB = 1000 × 9,8 × 55 × 10–3 = 539 Pa. b. Vitesse de l’air : v B =

2 × 539 ≈ 28,8 m/s . 1,3

c. Le tube de Pitot est le capteur utilisé pour mesurer la vitesse d’un avion (il mesure en fait la vitesse d’écoulement de l’air autour de l’avion).

8. Nettoyeur haute pression a. S = 1,43 × 10–6 m2 ;

Q = S2

2,22 ×10

−4

 6 ×10 −3  π× 2  

a. Détente : À pression constante,

V = Cte d’où T

b. Compression : À température constante, p.V = Cte, d’où p3 =

p2 .V2 105 × 2,54 = = 1,27 ×105 Pa . 2 V3

13. Chauffe-eau solaire a. La relation de Bernoulli donne : 1 2 1 1 v + g.z1 = v 22 + g.z 2 soit v12 = g.z 2 − g.z1 . 2 1 2 2 Après calcul, v1 = 24,2 m/s. b. Le débit volumique Q = v.S = 24,2 × 1,2 × 10–4 = 2,9 × 10–3 m3/s ou 2,9 L/s. La durée de vidange est : t = V/Q = 172 secondes.

Exercices à caractère professionnel 14. Pression de pneumatique

0,8 Q = 0,8 m3/h =  = 2,22 × 10–4 m3/s ; 3600 Q v =  = 155,4 m/s. S b. La conservation du débit : v1.S1 = v2.S2 = Q entraîne v 2 =

12. Détente et compression V2 V1 373 = et V2 = 2 × = 2,54 L . T2 T1 294

2 ×1200

v2 =

- le volume d’air contenu dans la seringue diminue ; - la quantité de matière reste constante ; - la pression de l’air augmente ; - la température de l’air reste constante.

2

≈ 7,85

La vitesse de l’eau dans la buse est de 7,8 m/s.

9. Conduite de section variable

a. n =

p.V 3,2 ×105 × 25 ×10 −3 = = 3,29 mol . 8,31× 293 R.T

b. T =

p.V 3,5 ×105 × 25 ×10 −3 = = 320 K . 3,29 × 8,31 n.R

c. Si on gonfle les pneus à chaud, on ne peut pas respecter les pressions préconisées et on effectue un sous-gonflage.

15. Nettoyage au jet d’eau a. Débit : 6 000 L/min = 0,1 m3/s. b. Section : S1 =

Q 0,1 = = 0,0125 m 2 soit 125 cm2 et v1 8

La relation de Bernoulli donne : p 1 2 pA 1 v + + g.z A = v B2 + B + g.z B soit : 2 A ρ ρ 2

le diamètre vaut D = 12,6 cm au millimètre près. p p 1 1 c. Bernoulli : v12 + 1 = v 22 + 2 donne : 2 ρ 2 ρ

p 1 0,8 ×105 1 3,6 2 + + 9,8 × 3 = 1,22 + B + 9,8 × 7,5 . 2 1000 1000 2 La pression en B est pB = 41 660 Pa (0,42 bar).

p 1 2 1 105 8 + 1 = 80 2 + 2 1000 2 1000 et p1 = 3,27 × 106 Pa soit 33 bar à l’unité près.

Appliquer le cours

16. Ventilation a. Section : S = 3,14 × 10–2 m2 ; débit : Q = 0,025 m3/s.

P : pression en pascal ; V : volume en mètre cube ; n : quantité de matière en mole ; R : constante des gaz parfaits en J/(K.mol) ; T : température en kelvin.

Q 0,025 Vitesse d’écoulement : v = = = 0,796 m/s S 3,14 ×10 −2 soit 0,8 m/s à 0,1 près. p p 1 1 b. Bernoulli : v12 + 1 + g.z1 = v 22 + 2 + g.z 2 2 ρ ρ 2

11. Compression, détente

donne :

10. Équation d’état des gaz parfaits

Lorsque l’on comprime très lentement l’air contenu dans le corps d’une seringue reliée à un manomètre :

p 1 2 101300 1 0 + + 9,8 × 2 = 0,8 2 + 2 + 9,8 × 0 2 1,3 1,3 2

d’où p2 = 101 325 Pa.

CHAPITRE 11 - Comment fonctionnent certains dispositifs de chauffage ? • 45

c. La différence de pression est négligeable.

19. Filtration d’eau de piscine

17. Réfrigération de l’eau

Étude du filtre a. Débit : Q = V/t = 80÷4 = 20 m3/h. b. Section du filtre : S = Q/v = 20÷50 = 0,4 m2.

a. En 1 : V =

n.R.T 18 × 8,31× 272 = = 0,214 m 3 . p 1,9 ×105

b. En 2 : T2 =

p2 .V2 8,5 ×105 × 63 ×10 −3 = = 358 K n.R 18 × 8,31

ou 85 °C.

c. De Q = m.c.∆t, on tire m = 240 × 103 ÷ (4180 × 5)

D2 4S , on tire D = = 0,71 . 4 π Le diamètre du filtre est de 71 cm.

c. De S = π.

Étude de la canalisation

= 11,5 kg soit un débit de 11,5 L/min.

d. Débit dans un skimmer : 20 ÷ 3 = 6,67 m3/h soit

18. Branchements de radiateurs

7 m3/h à l’unité près. e. Sur le graphique, la perte est ∆p = 0,2 bar.

a. Section du tuyau dans le radiateur : Sr = 5,0 × 10–5 m2. Section du tuyau dans la chaudière : Sc = 3,14 × 10–4 m2. Débit : Q = 0,018÷ 60 = 3 × 10–4 m3/s. D’où la vitesse d’écoulement de l’eau dans la chaudière : 3 ×10 −4 vChaudière = = 0,955 m/s . 3,14 ×10 −4 b. Branchement en dérivation : les sections des tuyaux s’additionnent : S = 4 × 5,0 × 10–5 = 2 × 10–4 m2. Vitesse d’écoulement de l’eau dans les radiateurs : 3 ×10 −4 vRadiateur = = 1,5 m/s . 2 ×10 −4 c. Branchement en série : la section du tuyau est constante : S = 5 × 10–5 m2. Vitesse d’écoulement de l’eau dans les radiateurs : 3 ×10 −4 vRadiateur = = 6 m/s . 5 ×10 −5

20. Phénomène de Venturi a. Entre les deux feuilles, la vitesse de l’air est supérieure à la vitesse de l’air circulant au dos des feuilles. La pression entre les feuilles est donc inférieure à la pression au dos des feuilles (pression atmosphérique) et les deux feuilles se rapprochent jusqu’à se toucher. b. La vitesse de l’air au-dessus du tube vertical est importante alors qu’elle est nulle à l’intérieur du tube. La pression au-dessus du tube est inférieure à celle à l’intérieur du tube et l’eau est aspirée vers le haut, puis projetée vers l’avant, en fines gouttelettes par le souffle d’air. c. La vitesse de l’air au-dessus de la feuille est supérieure à celle de l’air au-dessous de la feuille : la pression de l’air au-dessus de la feuille est donc inférieure à celle de l’air au-dessous et la feuille est poussée vers le haut.

12 Comment fonctionnent un transformateur et une plaque à induction ? CME6 et CME7

Les activités ■■Activité 1 Comment produire un courant induit ? Comment déterminer le sens du courant induit ? Expériences 1 et 2

• Matériel  Un moteur et un aimant permanent fixé sur son axe Une bobine de 400 à 1 200 spires ; un aimant permanent ; des fils de connexion Un oscilloscope ou un système Exao avec un capteur de tension

46 • CHAPITRE 12 - Comment fonctionnent un transformateur et une plaque à induction ?

• Réponse aux questions 1. Lorsque l’aimant, entraîné par le moteur, tourne devant la bobine, la tension observée sur l’écran est alternative.

2. Une tension apparaît lorsque l’aimant est en mouvement. On n’observe aucune tension si l’aimant est immobile.

3. Lorsque la fréquence de rotation de l’aimant augmente, la fréquence et l’amplitude de la tension alternative apparue aux bornes de la bobine augmentent.

4. le sens du courant dépend du pôle de l’aimant et du mouvement réalisé (approche ou recul de l’aimant). On peut énoncer la loi si on a pris soin de noter le sens d’enroulement du fil électrique constituant la bobine.

■■Activité 2 Comment fonctionne un transformateur ? Expériences 1 et 2

• Matériel  Un générateur alternatif délivrant une intensité efficace de 5 à 10 A ; un oscilloscope Un transformateur démontable avec deux bobines (400 spires et 200 spires par exemple) Une spire à fusion ; des fils de connexion ; des baguettes d’étain.

• Réponse aux questions  Expérience 1

1. Un transformateur ne modifie pas la fréquence d’une tension sinusoïdale : 50 Hz au primaire et au secondaire.

2. Pour N1 = 400 spires et N2 = 200 spires, on relève U1 = 12,2 V et U2 = 6,1 V. On dit que le transformateur est « abaisseur » de tension car U2 < U1.

3. La spire à fusion est une bobine à une seule spire : elle est parcourue par un courant de très grande intensité qui entraîne la fusion de l’étain dans la gorge de la spire.

4. Dans un transformateur, la bobine secondaire est le siège d’une tension induite. Une plaque à induction fonctionne sur le même principe : le primaire est une bobine placée sous la plaque de cuisson, le secondaire est constitué par le récipient en acier qui se comporte comme une spire unique en court-circuit.

Les documents ■■Le transport de l’électricité • Réponse aux questions 1. Il faut transporter l’électricité sous de très grandes tensions : ainsi, les intensités des courants parcourant les câbles sont beaucoup plus basses (voir l’exercice 13) et les pertes en ligne par effet Joule (PJ = R.I2) sont plus faibles. On élève la tension de 20 kV à 400 kV à l’aide de transformateurs élévateurs placés à la sortie de la centrale électrique. On abaisse les tensions de 63 kV à 230 V à l’aide de transformateurs abaisseurs de 63 kV à 20 kV, puis de 20 kV à 230 V.

2. La puissance perdue par effet Joule est PJ = 10,5 × (1 500)2 = 2,36 × 107 W.

La puissance transportée est P = 3 × 400 × 103 × 1 500 = 1,8 × 109 W. PJoule Le rapport = 0, 013 soit 1,3 %, taux bien inférieur aux 5 % correspondant aux conditions Ptransportée normales.

CHAPITRE 12 - Comment fonctionnent un transformateur et une plaque à induction ? • 47

Les exercices Tester ses connaissances

bornes des enroulements primaire et secondaire du transformateur. b. m = U2/U1 = 12,3/ 228 = 0,054. c. Le transformateur est abaisseur de tension, car m < 1.

Q.C.M. 1 : A et B ;  2 : A et B ;  3 : A, B et C ;  4 : A ;  5 : A et B.

Tester ses capacités 1. Recherche des pôles

Appliquer le cours

S

N

6. Aimant en U N S

S

S

BB

N

N

S B

I (1)

(2)

(1)

I P

B

BC

S

(2)

P

I

7. Les faces d’une bobine

N N

S (3)

B

N

B B

N

S

S I

I

3. Loi de Lenz (1)

S

N

8. Calculs de champ magnétique

(2)

N

S

S

N

a.

S

N

n B 0°

Bobine fixe

S

S

(4)

N

Aimant fixe

N

S

N

S

Aimant fixe

4. Soudure par points Lorsque les deux pointes sont en contact, le circuit de la bobine est fermé. Celle-ci ne comporte que 5 spires : l’intensité du courant dans le secondaire est très importante et assure la fusion de l’étain dans la gorge de la spire.

5. Abaisseur ou élévateur a. Les voltmètres mesurent les tensions efficaces aux

B

B 45°

n

Bobine fixe (3)

N

M

P

N

I

S B

(4)

P

BA

N

C

2. Sens du champ magnétique

A

90°

n

b. ΦB = N.B.S.cos  θ. Pour θ = 0°, Φ(0) = 5 × 10–2 × 20 × 10–4 × 50 = 5 × 10–3 Wb. Pour θ = 45°, Φ(45) = Φ(0).cos45°= 3,54 × 10–3 Wb. Pour θ = 90°, Φ(90) = 0 Wb. c. Sur un quart de tour, θ passant de 0° à 90°, Φ(90) − Φ(0) = –  5 × 10–3 Wb.

9. Courant induit Expérience 1 : la bobine 1 est parcourue par un courant alternatif ; une tension induite alternative de même fréquence apparaît aux bornes de la bobine 2. Expérience 2 : l’éloignement de la bobine 1 fait varier le champ magnétique, donc le flux, dans la bobine 2. Cette variation de flux entraîne l’apparition d’une tension induite dans cette seconde bobine.

48 • CHAPITRE 12 - Comment fonctionnent un transformateur et une plaque à induction ?

Expérience 3 : lorsque l’interrupteur est fermé, un champ magnétique apparaît dans la bobine 1. Pendant la durée d’apparition de ce champ, le flux varie dans la bobine 2 et une tension induite se crée.

10. À vous de résoudre a. Rapport de transformation : m = Usecondaire/Uprimaire = 18,3/ 117 = 0,156.

b. m = N2/N1 . Donc N2 = m.N1 = 0,156 × 448 = 70 spires.

11. Étude d’un transformateur a. U1max = 3,1 × 100 = 310 V ; U2max = 3,6 × 10 = 36 V. b. U1 = 219 V ; U2 = 25,5 V. c. Rapport de transformation : m = 0,116. d. I1 = U2.I2 ÷ U1 = 25,5 × 0,24 ÷ 219 = 0,028 A. e. S = U2.I2 = 6,12 VA. f. La puissance fournie est insuffisante pour faire

c. Le « réfractaire » est la matière de très faible conductivité thermique constituant le creuset. Il n’est chauffé que par conduction au contact du métal en fusion. d. Pour les aciers (ρacier > ρcuivre) : 1 300 Hz, pour les alliages de cuivre : 200 Hz.

15. Thermoscellage par induction a. La spire inductrice a une forme circulaire et le même diamètre que le col du pot.

b. Non, c’est le courant induit dans la feuille d’aluminium qui provoque l’apparition de courants induits qui échauffent et fondent le polymère au contact du col. c. Schéma :

Capsule dévissée

fonctionner la lampe de 20 W.

Exercices à caractère professionnel 12. Transformateur de centrale électrique a. 98 MVA : puissance apparente ; 11,5 kV : tension efficace au primaire ; 136 kV : tension efficace au secondaire ; 50 Hz : fréquence des tensions électriques. b. m = 136/ 11,5 = 11,83 > 1. Donc le transformateur est élévateur de tension. S 98 ×106 c. I 2 = = = 721 A. U 2 136 ×103 S 98 ×106 I1 = = = 8 522 A. U 2 11,5 ×103 L’intensité I2 du courant dans les lignes est plus faible afin de limiter les pertes par effet Joule.

13. Transport du courant a. I =

200000 P = = 966 A. U .cos ϕ 230 × 0, 9

Pertes Joule : PJ = R.I2 = 100 × 9662 = 9,33 × 107 W. Avec un rapport : PJ/P = 9,33 × 107/200 × 103 = 467. Le système ne peut pas fonctionner. b. Pour U = 20 kV, on trouve I = 11,1 A et PJ = 12 320 W avec un rapport PJ/P = 0,06 Dans ce second cas, avec un transport du courant sous haute tension, les pertes par effet Joule ne représentent que 6 % de la puissance transportée.

14. Four à induction a. Le four ne présente pas de circuit magnétique fermé à l’intérieur de la bobine inductrice.

b. Le métal à fondre sert d’induit.

Polymère

Cartonnette Aluminium Col

16. Freinage TELMA a. C’est le phénomène d’induction électromagnétique qui explique l’apparition des courants de Foucault dans les disques. Ceux-ci créent des forces qui s’opposent aux causes qui leur donnent naissance et tendent à ralentir la rotation des disques, donc du véhicule. b. L’échauffement des disques est dû à l’effet Joule créé par les courants de Foucault. On ne peut donc utiliser ce freinage que temporairement, en aide au freinage traditionnel.

17. Utiliser des appareils électriques a. Un transformateur ne peut fonctionner qu’avec une tension alternative dans le primaire, assurant des variations du flux magnétique. En outre, alimentée en courant continu, la bobine primaire peut être détériorée. b. Il faut d’abord onduler la tension continue de 12 V en tension alternative, puis élever celle-ci en une tension alternative de tension efficace supérieure (avec un transformateur élévateur), enfin redresser et lisser la tension alternative obtenue en tension continue de 48 V.

CHAPITRE 12 - Comment fonctionnent un transformateur et une plaque à induction ? • 49

13 À quoi correspondent les bornes d’une prise de courant ? CME7

les Activités ■■Activité 1 Quels conducteurs se cachent derrière les bornes des prises de courant ? • Réponse aux questions 1. La prise du doc 1-a est raccordée à un réseau monophasé, car elle ne contient que trois bornes : une pour la phase, une pour le neutre et une borne de protection électrique. La prise du doc 1-b est raccordée à un réseau triphasé car elle contient trois bornes connectées aux trois phases, une borne pour le fil de neutre et une borne connectée au conducteur de protection électrique.

2. Pour la prise monophasée, la phase est connectée sur la borne Ph et le neutre sur la borne N. Pour la prise triphasée, les trois phases sont connectées sur Ph1, Ph2 et Ph3 et le neutre sur la borne N.

3. Les bornes d’une prise de courant sont appelées bornes de phase, borne de neutre ou borne de protection électrique. Les bornes de phase et de neutre permettent de transporter l’énergie électrique. La borne de protection électrique fait partie du dispositif de protection des personnes.

■■Activité 2 Quelles sont les tensions efficaces d’une alimentation triphasée ? • Matériel Une alimentation triphasée ; un multimètre ; Deux fils de connexion.

• Réponse aux questions 1. Les tensions simples efficaces sont égales. Les tensions composées efficaces sont égales. 2. La relation entre U et V est U = V 3 .

■■Activité 3 Quelles sont les caractéristiques des tensions instantanées mesurées aux bornes d’une alimentation triphasée ? • Matériel Une alimentation triphasée ; système exao + 3 capteurs de tension

• Réponse aux questions 1. Les trois courbes obtenues sont des sinusoïdes. 2. Elles ont même période et même amplitude. Elles sont décalées les unes par rapport aux autres d’un tiers de période.

3. La réponse dépend de l’alimentation utilisée : généralement, une alimentation bien construite est parfaitement équilibrée.

4. Les tensions fournies par les prises de courant sont sinusoïdales de fréquence 50 hertz. Pour une alimentation triphasée, les trois tensions simples ont même période et même amplitude et sont déphasées l’une par rapport à l’autre d’un tiers de période. Les trois tensions composées ont même période et même amplitude et sont déphasées l’une par rapport à l’autre d’un tiers de période.

50 • CHAPITRE 13 - À quoi correspondent les bornes d’une prise de courant ?

Les documents ■■Champ magnétique tournant dans un moteur asynchrone triphasé 1. Oui, le sens du champ magnétique dans une bobine est directement lié au sens du courant dans la bobine.

2. Pour changer le sens de rotation d’un champ tournant il faut inverser le branchement de deux phases sur le générateur.

3. Oui, le nombre de paires de pôles p au stator influe directement sur le fréquence de rotation du rotor d’un moteur (voir Doc page 146 et chapitre 6).

4. Oui car si les phases ne sont pas connectées dans le bon ordre, le moteur tournera « à l’envers ».

Les exercices Tester ses connaissances Q.C.M. 1 : A, B, C ;  2 : B ;  3 : A, B, C ;  4 : C ;  5 : A, B, C ;  6 : C.

Tester ses capacités 1. Prise trois conducteurs a. Fil bleu : conducteur de neutre ; Fil marron : conducteur de phase ; Fil vert-jaune : conducteur de protection. b. Le fil vert-jaune doit être connecté à la borne mâle, le fil bleu à la borne de gauche et le fil marron à la borne de droite.

2. Recopier en corrigeant a. Les cinq conducteurs d’une prise triphasée sont : 3 phases, 1 neutre, 1 protection.

b. Les conducteurs de neutre doivent être bleus. c. Le conducteur de protection doit être vert-jaune. d. Le conducteur de protection n’est pas relié au réseau du fournisseur d’électricité.

même amplitude et le même décalage par rapport à v1. b. u12 = vcalc et donc u12 = v1 - v2.

Appliquer le cours 6. Faut-il un conducteur de protection ? Affirmations corrigées : - Il y a un conducteur de protection sur cette prise (borne femelle). - Benjamin n’est pas protégé contre les risques d’électrisation, car sa fiche ne dispose pas de conducteur de protection (sauf si sa console comporte un dispositif d’isolement : appareil de classe 2 ou 3). - La console n’est pas protégée contre les surintensités. (L’image ne permet pas de le dire) - La console peut fonctionner avec cette prise.

7. Représentation vectorielle d’une tension π 2 donc U1max =  400 2 = 566 V. Pour une échelle de 1 cm pour 200   V, la longueur du vecteur U 1 sera de 2,8 cm. π Le déphasage ϕ1 =  − donc le 2   vecteur U 1 sera vertical vers le bas.

a. u1(t) =  400 2 sin(100πt − ) ,

3. Exploitation activité 3 a. T = 39,5 ms b. d1 = 13 ms ; d2 = 13,06 ms ; d3 = 13,3 ms. a. ϕ1 =

5. Construction d’une tension composée a. Oui u12 et vcalc ont la même période : 40 ms, la

2πd1 2π × 13 = = 2, 07 rad ; ϕ2 = 2,08 rad ; T 39, 5

ϕ3 = 2,12 rad.

4. U ou V ? a. La tension simple efficace se note V. b. Pour une alimentation triphasée, on peut écrire

U2

U1

U = V 3 et Umax = Vmax 3 .

u2(t) =  230 2 sin(100πt ), donc U2max =  230 2 = 325 V. 

nies par la même alimentation triphasée ont la même fréquence et ont des tensions efficaces différentes. d. Une tension composée se mesure entre la phase 1 et la phase 2 et entre deux phases.

Pour une échelle de 1 cm pour 200V, la longueur du vecteur U 2 sera de 1,6 cm.   Le déphasage ϕ2 = 0 donc le vecteur U 2 sera horizontal vers la droite. b. u2 est en avance sur u1.

c. Une tension simple et une tension composée four-

CHAPITRE 13 - À quoi correspondent les bornes d’une prise de courant ? • 51

c. La tension efficace U1 est égale à 400 V et la tension efficace U2 à 230 V.  U On remarque que 1 = 3 . U1 est donc la tension U2

– V2 A U12

composée et U2 la tension simple.

8. Déterminer l’amplitude et la fréquence du signal. a. v V1

V2

V1

π 3

H

2π 3

2π 3

V3

O2π 3

t

O

V3

V2

Vérifier par exemple que :

b. L’amplitude vaut trois divisions soit : Umax = 3 × 5 = 15 V. c. La période T vaut six divisions soit T = 6 × 0,5 1 1 = 3 ms. La fréquence f = = = 333 Hz T 3 ×10 −3

9. Apprendre à résoudre : choix du couplage a. Réseau 400 V/690 V : « 400 V » représente la tension simple du réseau d’alimentation mesurée entre une phase et le neutre et exprimée en volt. « 690 V » représente la tension composée du réseau mesurée entre deux phases et exprimée en volt. b. Les enroulements doivent être soumis à 400 V ce qui correspond à la tension simple du réseau. C’est le montage étoile qui permet de soumettre les enroulements à la tension simple du réseau. c. Schéma du montage étoile :

U12 = V12 +V22 − 2 ×V1 ×V2 × cos(V1 ;V2 ) On peut aussi considérer le demi-triangle équilatéral OAH : 3 OH = OA.sin(π/3)= OA 2 U OA = V et OH = U/2 ; d’où = 3 V

11. Tensions sur la calculatrice (tice) π v1(t) =  325sin(314t − ) 3 π v2(t) =  325sin(314t + ) 3

a.

100 0 -0,02

V1

-0,01

-100

0

0,01

-300

V3 Recepteur triphasé

b.

600

u

N 400

10. Relation entre U et V a.

0,02

-200

Ph2

Ph3

V1

200

Ph1 V2

V2

300

1 2 3 N

V2

V1

200

u12

0

v2

v1

b. u12 = v1 - v2. c. Pour vérifier la relation U = V 3 , mesurer les longueurs des vecteurs et vérifier par un calcul que U = 3 ou utiliser la trigonométrie en utilisant V ­l’angle entre V1 et V2 :

-0,02

-0,01

0

0,01

0,02

-200 -400

c. On mesure l’amplitude Umax = 563 V, le décalage d ≈ 1,7 ms d’où ϕ =

2π × d 2π × 1, 7 = = 0, 53 rad avec v1 T 20

en avance sur u.

52 • CHAPITRE 13 - À quoi correspondent les bornes d’une prise de courant ?

d. L’écriture algébrique de la tension u(t) est donc :

14. Montage triangle

π u(t) = 563sin(314t – – 0,53) = 563sin(314t – 1,58) 3 NB Pour le professeur : la construction de Fresnel permet de trouver u rapidement : π u = 325 3 sin(314t – ) 2 et l’on retrouve bien l’expression précédente.

a.

Exercices à caractère professionnel 12. Plaque à bornes a. Le couplage réalisé sur la plaque à bornes de la photo est un montage étoile.

b. Les enroulements du moteur sont alors soumis à une tension simple. c. Montage des plaques pour un branchement « triangle » : barrette

3 Α 2

Α

Α

1

7 = =4A 3 3 c. Dans le couplage triangle, chaque dipôle est soumis à une tension composée soit 400 V.  U 400 R= = = 100 Ω . J 4

b. J =

I

15. Étude d’un moteur asynchrone triphasé (d’après bac) a. Chaque enroulement doit être soumis à 230  V, donc à la tension simple du réseau d’alimentation. Il faut donc réaliser un couplage étoile. P P 0, 75 b. Pu = 0,75 kW et η =  u donc Pa =  u = = 1, 07 Pa η 0, 7 d’où Pa = 1,1 kW arrondi à la centaine de watt.

c. Pa = U.I. 3 cosϕ, donc : I= Ph1

Ph2

Ph3

Enroulements du moteur

13. Éléments chauffant a. C’est la tension Ph1

u12 composée du réseau qui doit alimenter chacun des conducu31 Ph2 teurs ohmiques du radiateur. u23 b. Résistance mesuPh3 rée à l’ohmmètre : Recepteur triphasé La résistance Re équivalente aux deux branches en parallèles se calcule avec la formule : 1 1 1 3 = + = . Re R R + R 2R

Si Re = 88 Ω, R = 

3 × Re 2

=

3 × 88 = 132 Ω . 2

c. R



R R

Pa U 3 cos ϕ

=

1100 400 × 3 × 0, 83

= 1, 91 A .

On vérifie bien la valeur donnée dans la dernière colonne du tableau.

16. Les risques électriques : Seuils de courants dangereux a. Les effets du courant sont ressentis à partir de 0,5 mA. Ils sont dangereux à partir de 30 mA.

b. Un courant de 200 mA est dangereux à partir de 400 ms soit 0,4 s. c. un courant de 50 mA durant 500 ms : effets non mortels. • un courant de 400 mA durant 50 ms : effets non mortels. • un courant de 1 A durant 20 ms : risque de brûlures graves et de décès. d. La valeur 30 mA est indiquée car elle correspond à l’intensité de déclenchement des disjoncteurs différentiels présents sur les tableaux électriques des maisons.

17. Équilibrer une installation Les centres d’usinage et le moteur triphasé sont équilibrés sur les trois phases. Les tubes fluorescents consomment 1 200 watts soit autant que la perceuse à colonne ou chacun des compresseurs. On peut donc connecter la perceuse à colonne et 6 tubes sur la phase 1, un compresseur et 7 tubes sur la phase 2 et l’autre compresseur et 7 tubes sur la phase 3.

CHAPITRE 13 - À quoi correspondent les bornes d’une prise de courant ? • 53

14 Comment calcule-t-on la puissance consommée par un appareil monophasé ? CME7

Les activités ■■Activité 1  Comment visualiser la puissance instantanée absorbée par un appareil électrique ? • Matériel Un générateur de tension alternative ; une résistance Un système exao ; un capteur de mesure de tension ; un capteur de mesure d’intensité

• Réponse aux questions 1. Oui u(t) et i(t) ont la même fréquence. 2. Oui, la puissance p(t) est périodique. Non, la période de p(t) est deux fois plus petite que celle de u(t) et i(t).

■■Activité 2 Comment mesurer la puissance moyenne consommée par un appareil en monophasé ? • Matériel Un générateur de tension alternative ; un ampèremètre ; un voltmètre ; un wattmètre Une résistance ; une bobine

• Réponse aux questions Expérience 1

1. Expression de la puissance dans un dipôle résistif, en courant alternatif : P = U . I. Expérience 2

2. Non, la puissance P absorbée par la bobine n’est pas égale au produit U.I. Expression de la puissance dans un dipôle quelconque, en courant alternatif : P = U.I.cos  ϕ.

3. La puissance consommée par un appareil électrique en alternatif dépend de la tension d’alimentation U, du courant I qui le traverse et du cos  ϕ appelé le facteur de puissance et caractéristique de l’appareil.

■■Activité 3 Comment déterminer l’intensité instantanée du courant appelé par un ensemble de récepteurs ? • Matériel Un générateur de tension alternative ; un dispositif de mesure exao ; deux capteurs d’intensité Une résistance ; une bobine

Expérience 1 - Afin de visualiser l’intensité i3, créer une nouvelle variable i3(t) telle que i3(t) = i1(t) + i2(t)

• Réponse aux questions Expérience 1

1. Non, en courant alternatif, cette loi utilisant des valeurs efficaces ne s’applique pas. On peut le I 1 m ax

I I + 2 m ax ≠ m ax . 2 2 2 2. Oui, les courants ont la même fréquence. vérifier dans ce cas en vérifiant que

54 • CHAPITRE 14 - Comment calcule-t-on la puissance consommée par un appareil monophasé ?

Expérience 2

3. Oui, en courant alternatif, la loi d’additivité des courants s’applique, pour tout dipôle, sur les intensités instantanées.

Les exercices Tester ses connaissances

b. « 330 µF » indique que la capacité du condensa-

Q.C.M. 1 : B ;  2 : A et C ;  3 : A et B ;  4 : A et B ;  5 : A, B, C

a.

teur est égale à 330 microfarads.

P

Tester ses capacités 1. Réalisation du montage

R

G

R

24V~ u (t)

C

W

M système EXAO

i (t)

2. Produit d’une tension par un courant. a. Les grandeurs u(t) et i(t) ont la même fréquence, mais p(t) n’a pas la même fréquence.

b. On peut établir que p(t) = u(t) .i(t).

3. Le wattmètre joulemètre a. Un wattmètre mesure une puissance moyenne ; b. Les montages corrects du wattmètre sont ceux des

7. Lire une plaque signalétique Tension nominale de fonctionnement : de 220 à 240 volts ; fréquence nominale de fonctionnement : 50 ou 60 hertz. La puissance nominale de l’appareil est de 1 000 watts avec un maximum à 1 200 watts. L’indice de protection électrique est IPX4. Appareil étanche aux projections d’eau.  : Appareil protégé par isolation double et symbole isolation renforcée.

schémas a et c.

8. Rendement

4. Somme de courants alternatifs.

a. η =

a. Les courants ont tous la même période 20 ms, donc ils ont tous la même fréquence (50 Hz).

b. Non, la somme des valeurs maximales I1max et I2max (3 et 5 DIV) n’est pas égale à la valeur maximale de la courbe somme (7,5 DIV).

Appliquer le cours 5. Influence de condensateurs en parallèle a. Si un condensateur est placé en parallèle du moteur, le déphasage entre la tension et l’intensité diminue. C’est donc l’oscillogramme 2 qui correspond à la mesure faite avec le condensateur. b. La période T de l’alimentation est égale à 20 ms et la fréquence f = 1/T = 50 hertz. c. On mesure un décalage d = 2,5 ms. 2π × d 2π × 2, 5 Déphasage : ϕ = = = 0, 79 rad. T 20

6. Étude d’un circuit électrique a. Ce circuit comporte un générateur de courant alternatif, un interrupteur, une résistance et un condensateur.

C1 = 330 µF

Pu

P 500 donc Pa = u = = 562 . η 0, 89 Pa En réalité, la puissance absorbée est de 562 watts. b. Le vendeur confond énergie consommée et puissance. Si la puissance absorbée est de 562 W, le client consommera 562 wattheures (Wh) en une heure ou 562 joules par seconde. c. Schéma complété. 562 W

500 W

moteur 500 W 62 W

9. Fendeuse à bois a. U : Tension efficace de fonctionnement en volt ; f : fréquence de la tension d’alimentation en hertz ; I : Intensité nominale en ampère ; cos  ϕ : facteur de puissance de la fendeuse. b. P = U.I.cos  ϕ = 230 × 15 × 0,87 = 3 001 W S = U.I = 230 × 15 = 3 450 VA ϕ = cos–1(0,87) = 0,52 rad soit 29,5°

CHAPITRE 14 - Comment calcule-t-on la puissance consommée par un appareil monophasé ? • 55

c. Échelles :

Exercices à caractère professionnel

S = 3,45 cm

1 cm pour 1 000 W 1 cm pour 1 000 VA 1 cm pour 1 000 var

Q

ϕ = 29,5°

Le côté Q a une longueur P = 3 cm de 1,7 cm. Donc Q = 1 700 var d. En appliquant le théorème de Pythagore dans le triangle des puissances, on obtient : Q =  S 2 − P 2 = 3450 2 − 30012 = 1702 var

10. La loi sur le paiement de la puissance a. Exemple de longueur pour les côtés du triangle : P = 5 cm, Q = 5 × 0,4 = 2 cm

b. Sur le graphique, n mesure ϕ = 22° et S = 5,4 si P = 5.

S

13. À l’hôpital a. Puissance consommée par un lit : P1 = U.I.cos  ϕ = 230 × 1,5 × 0,79 = 272,55 W. Pour l’ensemble des lits : Pmax = 22 × P1 = 22 × 272,55 = 5 996 W. b. Énergie maximale consommée par jour : Emax = Pmax × durée d’utilisation. La durée maximale d’utilisation dmax est égale à : 2 dmax = (24 × 60)× = 221,5 minutes soit 3,69 heures. 13 Par conséquent : Emax = 5 996 × 3,69 = 22 125 Wh = 22,1 kWh. c. Coût mensuel : Cm = 22,1 × 30 × 0,0803 = 53 €.

14. Station de pompage – Théorème de Boucherot b. Puissance active de chaque moteur : Pumoteur

S =  P 2 +Q 2 = 52 + 22 = 5,39. VA.      −1 Q −1 0, 4 P ) = tan −1(0, 4) = 21, 8 ° ou ou ϕ = tan ( ) = tan ( P P 0,38 rad.

3150 = = 3987  W. η 0, 79 Puissance apparente de chaque moteur : P 3987 = 5538 VA. Smoteur =  amoteur = cos ϕ 0, 72 Puissance active de chaque tube fluorescent : P 60 = 60 W. Patube =  utube = η 1 Puissance apparente de chaque tube : P 60 = 75 VA. Stube =  atube = cos ϕ 0, 80 c. Puissance réactive absorbée par chaque moteur :

11. Apprendre à résoudre 

2 2 Qmoteur =  Sm − Pam = 5538 2 − 3987 2 = 3843 oteur oteur

a. La puissance apparente est :

= 3843 var. Puissance réactive absorbée par chaque tube :

Q

ϕ P Au dessous de ϕ = 22°, on ne paie pas de taxe. Q 2 c. ϕ = tan–1( ) = tan–1( )= 21,8° ; P 5

S = U.I = 230 × 0,52 = 119 VA. b. Le facteur de puissance est : P 100 cos  ϕ = =  = 0,84 donc ϕ = 0,573 rad. S 119 c. La puissance réactive est Q = S × sin  ϕ ; Q = 119 × sin(0,573) = 64 var.

12. Le coût du jeu a. Puissance consommée par la console : E 60 ×103 = = 200 W. t 5 × 60 b. Énergie consommée par jour : E/J = 200 × 2 = 400 Wh (wattheures) soit 400 × 3 600 = 1,44 × 106 = 1,44 MJ. Énergie consommée par an : E/an = E/J × 365 = 146 000 Wh = 146 kWh soit 146 × 103 × 3 600 = 525 × 106 = 525 MJ. c. Le coût annuel de la consommation électrique C est de : C = 146 × 0,0803 = 11,72 € P=

Pamoteur = 

2 2 − Patube = 752 − 60 2 = 45 var. Qtube =  Stube

d. Puissance active absorbée par l’installation : Patot = ΣPa. Patot = 7 × Pamoteur + 20 × Patube = 7 × 3 987 + 20 × 60 = 29 109 W. Puissance réactive absorbée par l’installation : Qtot = ΣQ. Qtot = 7 × Qmoteur + 20 × Qtube = 7 × 3 843 + 20 × 45 = 27 801 var. e. Puissance apparente de l’installation : 2 2 Stot =  Patot +Qtot = 291092 + 278012 = 40252 VA.

f. Intensité efficace du courant dans la ligne d’aliS 40252 = = 175  A. U 230 Facteur de puissance global de cette installation : P 29109 = 0,723 . cos  ϕ =  atot = Stot 40252 mentation : I = 

56 • CHAPITRE 14 - Comment calcule-t-on la puissance consommée par un appareil monophasé ?

g. Non, le facteur de puissance de cette installation ne convient pas. Pour remédier à ce problème, il est possible de placer des condensateurs en parallèle des moteurs afin de diminuer le déphasage entre la tension d’alimentation et le courant et ainsi augmenter le facteur de puissance.

15. Caractéristiques d’un moteur (d’après bac) a. Pu = 1 100 W et η = 76,8 % avec Pa =  Pa = 

1100 = 1432 . 0,768

Pu

η

Pa = 

E 47000 = = 979  W. t 48

b. Énergie utile : Ep = m.g.h = 600/2 × 9,81 × 10 = 29 430 J. Puissance utile : Pu = 29 430/48 = 613 W P 613 c. Rendement η du moteur : η =  u = = 0,63. Pa 979 Le moteur a un rendement de 63 %.

donc :

17. Scie radiale a. Grandeur et unité : 230 V : tension de l’alimen-

soit Pa = 1 430 W arrondie à la dizaine. b. Pa = U.I.cos  ϕ avec le facteur de puissance P 1430 cos  ϕ = 0,84 d’où I =  = = 7, 4 A U.cos ϕ 230 × 0, 84 arrondi à 10–1. c. Oui un fusible de calibre 10 A est suffisant car le calibre doit être légèrement supérieur à l’intensité nominale. d. PN = 2π.n.ℳ avec ℳ le moment du couple utile nominal du moteur et PN la puissance nominale sur l’arbre moteur (1,1 kW) et n la fréquence nominale de 1429 rotation du moteur en tr/s (n =  = 23,8 tr/s). 60 PN 1100 = = = 7,35 soit ℳ = 7,4 N.m. 2π.n 2π.23,8

16. Moteur d’une grue a. Puissance consommée : E = P × t avec P en watt si E est en joule et t en seconde ou E en wattheure et t en heure.

tation à utiliser ; 50/60 Hz : fréquence de la tension d’alimentation à utiliser ; 1,8 kW : puissance nominale utile en kilowatt ; cosϕ : facteur de puissance de l’appareil. b. Puissance utile : Pu = Pa × η = 2 225 × 0,8 = 1 780 W. Cette valeur correspond approximativement à celle affichée sur la plaque. c. Intensité efficace du courant électrique dans le câble d’alimentation : P 2225 I =  = = 11, 5  A. U.cos ϕ 230 × 0, 84

18. Intensités efficaces (Démarche d’investigation) Oui cela est possible car ce sont les courants instantanés qui s’ajoutent. Si on prend le cas extrême de deux courants en opposition de phase, la somme des deux courants est nulle. C’est le cas dans le neutre d’un réseau triphasé équilibré.

15 Comment peut-on améliorer sa vision ? HS4 et SL4

Les activités ■■Activité 1 Quelles sont les caractéristiques d’une lentille convergente ? • Matériel Une plaque de polystyrène Une boîte à lumière munie d’un peigne à 3, 4 ou 5 fentes Une lentille convergente et une lentille divergente Une page de texte.

• Réponse aux questions 1. Une seule des deux lentilles fait converger le faisceau de lumière : celle qui est plus épaisse en son centre et nommée « lentille convergente ».

2. La lentille convergente grossit le texte observé. CHAPITRE 15 - Comment peut-on améliorer sa vision ? • 57

■■Activité 2 Comment l’œil forme-t-il une image sur la rétine ? • Matériel Une bougie sur son support Une lentille convergente (15 à 30 cm de distance focale) sur son support Un écran (papier calque) sur support Un mètre de salle de classe.

• Réponse aux questions 1. Pour cette position, l’image obtenue sur l’écran est plus petite que l’objet. Elle est renversée. 2. Pour obtenir une image nette d’un objet plus rapproché, il faut utiliser une lentille plus convergente donc plus épaisse en son centre.

3. Le cristallin de l’œil se comporte comme une lentille convergente. L’image formée sur la rétine est renversée et de très petite dimension. Pour observer un objet plus proche, l’œil doit accommoder (voir fiche DOC) : le cristallin modifie sa forme et devient plus bombé.

■■Activité 3 Peut-on toujours obtenir une image avec une lentille convergente ? • Matériel Un banc d’optique : source de lumière, porte-objet + objet, porte-lentille + lentille, écran…

• Expérience Quelques résultats obtenus : OA

OA’

OA

OA'

1 1 1 − = OA' OA OF '

OF '

60 cm

43 cm

–60 cm

43 cm

0,0400

25,0 cm

La distance focale de la lentille étudiée est de 25 cm.

• Réponse aux questions 1. L’image est renversée. 2. L’image est plus grande que l’objet quand la lentille est proche de l’objet. Quand on rapproche la lentille de l’objet, l’image s’éloigne et sa taille augmente.

3. Pour grossir un objet, on utilise une lentille convergente. - Si on place la lentille à une distance de l’objet supérieure à sa distance focale, mais dans la position la plus proche, l’image obtenue sur l’écran est renversée et agrandie. - Si on place la lentille à une distance de l’objet inférieure à sa distance focale, l’image obtenue est virtuelle (du même côté que l’objet et non observable sur un écran), droite et grossie : c’est la loupe de l’activité 1. NB pour le professeur On utilise en optique deux notions qui ne doivent pas être confondues : le grandissement et le grossissement. A ’B ’ . Le grandissement γ concerne la taille relative des objets et des images : γ = AB α' Le grossissement se réfère à l’œil : G =  α α est l’angle sous lequel l’œil voit l’objet, ce dernier étant placé à la distance de 25 cm de l’œil ; α’ est l’angle sous lequel l’œil voit l’image à travers l’instrument d’optique, ici la loupe.

58 • CHAPITRE 15 - Comment peut-on améliorer sa vision ?

Les documents ■■L’œil et la vision • Réponse aux questions 1. La distance minimale de vision nette correspond au « punctum proximum » ; la distance maximale de vision nette correspond au « punctum remotum ».

2. Pour observer un objet proche, le cristallin devient de plus en plus convergent, sa distance focale diminue : il devient plus épais en son centre.

Les exercices Tester ses connaissances

1.

B

Q.C.M. 1 : A ;  2 : A et C ;  3 : A et B ;  4 : A et C ;  5 : A.

F’ A’ F

A

Tester ses capacités

O

1. Lentilles convergentes a. Les lentilles convergentes sont L1 et L2. b. le texte est grossi, il est donc observé à travers une lentille convergente.

B’

B’

2.

A F

2. Rayons lumineux a. Le point de convergence est le foyer principal

O

A’

F’

B

image de la lentille.

b. On peut observer l’image de la lampe très éloignée (4 à 5 mètres de la lentille) et former son image sur la feuille de papier ; la distance focale est la distance entre la lentille et la feuille de papier.

3. Choisir un protocole a. Le montage 1 est le bon car la distance entre l’objet et la lentille est supérieure à la distance focale de 25 cm.

lentille

tre optique O, le rayon incident parallèle à l’axe optique et le rayon incident passant par le foyer principal objet F.

5. Loupes Loupe 1 : un objet mesurant 4 mm (objet) donne une image de 8 mm. Loupe 2 : un objet mesurant 4 mm (objet) donne une image de 10 mm. C’est donc la loupe 2 qui grossit le plus.

Appliquer le cours

objet

F’ F

O

6. Modélisation de l’oeil

image

P

P

b. Il faut tracer le rayon incident passant par le cen-

b. Pour obtenir une image nette, il faut déplacer l’écran jusqu’à obtenir un « P » renversé et net.

4. Schémas à compléter

rétine ...............

pupille ........................

cristallin ........................ nerf optique ...........................

a. et c. : CHAPITRE 15 - Comment peut-on améliorer sa vision ? • 59

7. Vergence

12. Image virtuelle a. Lectures : OA = −3 cm ;

1 1 = 4 δ ; C2 = = 20 δ a. C1 = 0, 25 0, 05 1 1 = = 0,1 m soit 10 cm  ; b. f1 = C1 10 1 1 = = 0, 5 m soit 50 cm. f2 = C2 2

OA' = −7, 5 cm ;

OF ' = 5 cm ; OA et OA' sont négatives. −7, 5 = 2, 5 . Il est −3 OA positif et l’image est droite par rapport à l’objet.

b. Le grandissement : γ =

OA '

=

Exercices à caractère professionnel

8. Schématiser

13. Loupe F’ F

O

F’ F

F’ O

F

O

O

1 1 = = 20 δ. f 0, 05 b. O :F centreO optique, F : foyer principal objet, F’  : foyer principal image de la lentille. c. Construction :

a. La vergence de la loupe F’ est : C =

F’

B’

F

9. Construire a. Le symbole de la lentille est :  . C’est une lentille convergente. b. OF’ est la distance focale f ’. 1 1 = = 0,1 m soit 10 cm . c. f = C 10 d. Construction des rayons lumineux issus de B :

B F’ A

F

a. OM = −200 cm  ; MN = 25 cm  ; OF ' = 2 cm 1 1 1 1 1 1 − = − = et OM ' = 2, 02 cm. ; OM ' OM OF ' OM ' −200 2

b. OM ' est positif donc l’image est réelle et se place sur le capteur de l’appareil photo. OM ' 2, 02 c. Le grandissement vaut : γ = = = −0, 0101 . OM −200

A’

O

A

F’

L’image A’B’ est virtuelle, droite et plus grande que l’objet. d. L’image se forme à l’infini lorsque A est placé au foyer principal objet F.

B’ infini B

O

10. À vous de résoudre

figure 1

B

figure 2

AF

O

F’

14. Contrôle d’impression a. Schéma :

B’ B A’

F

A

O

La taille de l’image est : M ' N ' = γ .MN ≈ −0, 252 cm. soit 2,52 mm.

11. Image réelle a. Lectures : OA = −24 cm ; OA' = 48 cm ; OF ' = 16 cm ; OA est négative. OA ' 48 b. Le grandissement : γ = = = −2 . Il est négaOA −24 tif et l’image est renversée par rapport à l’objet.

b. L’image est virtuelle. c.

1 1 1 1 1 1 − = ; − = ; OA ' OA OF ' OA' −2 3

d’où OA' = -6 cm.

60 • CHAPITRE 15 - Comment peut-on améliorer sa vision ?

F’

d. γ =

OA ' OA

=

−6 = 3. −2

Le grandissement γ =

e. A ' B ' = γ . AB = 3 × 0, 5 = 1, 5 cm. L’image est 3 fois plus grosse que l’objet et justifie l’intérêt d’utiliser un compte-fils pour observer un motif.

15. Schématiser une situation 1 1 a. f = = = 0, 2 m soit 20 cm de distance focale. C 5 b. et c. Schéma (échelle : 50%) :

L

OA '

O

33 = −0, 66. −50

16. Avec deux lentilles a., b.

L1écran intermédiaire

B

O1 A

F’1 A1

F1

L2

O2

F2

F’2

B1

F’ F

=

La taille de l’image est : A ' B ' = γ . AB = −6, 6 cm. L’image est réelle et renversée.

B A

OA

A’ B’

c. Par mesure : OA' = 33 cm.

B’ infini c. L’image A’B’ est renvoyée à l’infini ; elle est virtuelle et renversée. Elle permet d’observer un objet lointain sous un plus grand angle. Une lunette astronomique ne permet pas d’observer les objets « droits ».

16 Comment faut-il se protéger des rayons du Soleil ? HS4

Les activités ■■Activité 1 Peut-on décomposer la lumière blanche ? • Matériel Un projecteur de diapositives ou une sorce de lumière blanche et un condenseur Un réseau (diapositive) Un écran.

Expérience Faire remarquer que, contrairement au prisme de verre, le réseau dévie davantage les radiations rouges. On observe sur l’écran des spectres d’ordre 1, 2, … symétriques par rapport à une bande centrale blanche. Les radiations infrarouges et ultraviolettes sont mises en évidence par les détecteurs.

• Réponse aux questions 1. On observe des radiations rouge, orange, jaune, verte, bleue et violette. Les infrarouges et les ultraviolets ne sont pas visibles à l’œil. 2. Avec le laser, on n’observe que des radiations rouges : pas d’infrarouges ni d’ultraviolets.

■■Activité 2 Le verre arrête-t-il toutes les radiations lumineuses ? • Matériel Une lampe à ultraviolets (UV) type philatéliste - Une lampe à infrarouge (IR) Une lame de verre sur support - Un détecteur d’infrarouges - Un détecteur d’ultraviolets. CHAPITRE 16 - Comment faut-il se protéger des rayons du Soleil ? • 61

• Réponse aux questions 1. Les rayonnements infrarouges ne sont pas arrêtés par le verre ordinaire alors que les rayonnements ultraviolets le sont en très grande partie. On peut utiliser des verres de lunettes traités pour éliminer 100 % des UV et vérifier leur qualité.

2. Le Soleil envoie un rayonnement lumineux très puissant et très riche en IR et en UV. Pour observer directement le Soleil, même masqué lors d’une éclipse, il faut donc utiliser des verres noirs traités anti UV.

■■Activité 3 De quoi dépend l’éclairement fourni par une lampe ? • Matériel Un luxmètre Des lampes de 25 W, 40 W, 60 W et 100 W Un diaphragme sur support Un mètre de salle de classe.

Expérience Mesures de l’éclairement obtenues en fonction de la distance : d (m)

0,30

0,40

0,60

0,80

1,00

1,20

1,40

E (lx)

920

530

240

145

92

65

48

11,1

6,25

2,78

1,56

1

0,69

0,51

1 d2

(m −2 )

Graphiques correspondants :

E (lx)

E (lx)

1100

1100

1000

1000

800

800

600

600

400

400

200

200

0

0

0,2

0,4

0,6

0,8

1,0

1,2

1,4

d (m)

0

1 (m–2) d2 0

2

4

6

8

10

12

14

   

• Réponse aux questions Expérience 1

1. L’éclairement se mesure avec un luxmètre. L’unité d’éclairement est le lux de symbole : lx. 2. l’éclairement est d’autant plus important que la puissance électrique de la lampe est grande. Les lampes de forte puissance : 60 W et 100 W sont éblouissantes et on ne peut pas les fixer longtemps.

Expérience 2

3. L’éclairement est proportionnel à

1 d2

donc la relation correcte est : E =

62 • CHAPITRE 16 - Comment faut-il se protéger des rayons du Soleil ?

k d2

.

Les documents ■■Les rayonnements ultraviolets • Réponse aux questions  1. Effets positifs des UV : ils facilitent le bronzage, la synthèse de la vitamine D. Ils permettent de traiter le rachitisme, le psoriasis, l’eczéma… Dangers pour la peau et les yeux : vieillissement et flétrissement de la peau allant jusqu’au cancer. Pour les yeux : cataractes, … jusqu’à la cécité.

2. Les UV sont d’autant plus dangereux que leur longueur d’onde est petite. Heureusement les UV A (λ < 200 nm) sont absorbés par la couche d’ozone.

3. Métiers de plein air : agriculteurs, charpentiers… Sports de montagne ou de mer : alpinisme, ski, voile…

Les exercices Tester ses connaissances Q.C.M. 1 : A B et C ;  2 : C ;  3 : A, B et C ;  4 : A et C ; 5 : B ;  6 : A et C ;  7 : A ;  8 : A et C.

Tester ses capacités 1. Mesurer un éclairement a. L’unité d’éclairement est le lux, de symbole : lx. b. À 50 cm, E = 1 203 lx ; à 1,40 m, E = 154 lx. 2. Rayons lumineux a. Longueur d’onde des radiations jaunes : 580 nm. b. 520 nm → vert ; 750 nm → rouge. c. f = 9x1014 Hz : ce sont des UV ; λ = 200 nm : ce sont des UV ; λ = 950 nm : ce sont des infrarouges ; λ = 1 µm (1 000 nm) : ce sont des infrarouges.

Appliquer le cours 3. À vous de résoudre a. Pour f = 7,5x1014 Hz, λ = Soit 400 nm.

c 3 × 108 = = 4 × 10 −7 m. f 7, 5 × 1014

c 3 × 108 Pour f Hz, λ = = = 1 × 10 −8 m. f 3 × 1016 Soit 10 nm. Finalement 10 nm < λ < 400 nm. b. Ce sont des radiations ultraviolettes. = 3x1016

4. Thermographie de bâtiments 3 × 108 c a. Pour λ = 7,5 µm, f = = = 4 × 1013 Hz. λ 7, 5 × 10 −6 3 × 108 c = = 2, 3 × 1013 Hz. λ 13 × 10 −6 Ce sont des infrarouges. b. La thermographie montre que les parties vitrées présentent une température plus élevée que le reste Pour λ = 13 µm, f =

du bâtiment : il faut prévoir un double vitrage ou changer les huisseries.

5. Indice UV a. La longueur de l’ombre dépend de la position du Soleil dans le ciel. Si le Soleil est haut, la température de l’air est élevée et l’ombre est très courte : l’indice UV est alors élevé. b. La connaissance de l’indice UV permet de prévoir la durée d’exposition et le type de protection.

6. Danger des UV a. Les UV A et UV B, de très faibles longueurs d’onde, sont les plus dangereux pour les yeux.

b. Les UV endommagent les cellules de la cornée, opacifient le cristallin et entraînent des cataractes puis des cécités. c. Protection : des verres anti UV à 100 %.

7. Lampe d’atelier a. Rendement lumineux : ΦL 6200 K=

P

=

75

= 82, 7 lm/W

b. Éclairement de la surface : ΦL 6200 E=

= = 1033 lx . 6 S c. L’éclairement est presque satisfaisant et s’approche de la valeur souhaitée de 1 050 lx.

8. Éclairement d’une lampe a. Flux lumineux : ΦL = K. P = 120 × 210 = 25 200 lm. b. Eclairement en O : E=

k d2

soit k = E . d2 = 160 × 25 = 4 000.

Donc à la distance LM = 6,53 m : k 4000 E= 2 = = 94 lx. d 6,532

CHAPITRE 16 - Comment faut-il se protéger des rayons du Soleil ? • 63

9. Danger d’un laser a. La radiation de longueur d’onde 632 nm a une couleur rouge orangée. b. Aire de la tache : A = π × (3 × 10 –4)2 = 2,8 × 10 –7 m2. La puissance par unité de surface : P/A = 7 140 W/m2. c. Cette puissance par unité de surface est 21 fois supérieure à celle du rayonnement solaire. d. Ce rayonnement est dangereux pour l’œil.

Exercices à caractère professionnel 10. Éclairage public a. Ces spectres ne sont pas continus. b. La lampe à vapeur de sodium haute pression émet une lumière blanche.

c. La lampe à vapeur de mercure et la lampe aux iodures métalliques émettent une lumière bleutée (présence de radiations bleues et violettes). d. La lampe à vapeur de mercure et la lampe aux iodures métalliques émettent des rayonnements ultraviolets de longueurs d’onde inférieures à 400 nm (plage des UV : de 10 à 400 nm).

11. Projecteur de spectacle a. Flux lumineux : ΦL = K. P = 34 × 300 = 10 200 lm. k on tire k = E . d2 . d2 Pour E = 7 600 lx et d = 5 m : k = 7 600 × 52 = 1,9 ×105 lx/m2. En appliquant la formule de l’éclairement d’une surΦ face E = L on calcule l’aire S et le diamètre D des S sections éclairées.

b. De E =

E (lx)

7 600

1 900

840

470

(m2)

1,34

5,36

12,1

21,7

1,31

2,61

3,97

5,25

S

Diamètre de la surface éclairée D (m)

Les diamètres des surfaces éclairées correspondent bien à ceux indiquées par le constructeur.

12. Soudure et sécurité a. Les rayonnements vont des ultraviolets (200 nm) aux infrarouges (1 400 nm).

b. Les rayonnements les plus dangereux sont les UV. c. Pour λ = 440 nm : λ 440 × 10-9 −15 T440 =

c

=

3 × 108

= 1, 47 × 10

s.

λ 240 × 10-9 = = 8 × 10 −16 s. c 3 × 108 d. Le rayonnement le plus dangereux est celui de plus faible longueur d’onde, donc celui de 240 nm. e. L’indice du verre assurant le moins de protection est 4 ; celui assurant le plus de protection est 13. f. Le verre clair, d’indice 4, permet de préparer aisément le poste de travail avant de souder. Pour λ = 240 nm, T240 =

13. Éclipse de Soleil (Démarche d’investigation) L’observation d’une éclipse de Soleil est dangereuse pour l’œil, même si le Soleil est en partie caché. Des rayons lumineux provenant du Soleil parviennent directement dans l’œil. Il faut se protéger en utilisant des verres noirs traités contre les UV et proposés pour l’observation des éclipses de Soleil.

17 Quels sont les principaux constituants du lait ? HS5

Les activités ■■Activité 1 Comment identifier les lipides dans le lait ? • Matériel Une feuille de papier blanc - Trois pipettes - Trois petits béchers - Un sèche-cheveux

• Produits Lait entier - Lait demi-écrémé - Lait écrémé

Expérience Une tache translucide indélébile apparaît sur le papier en présence de lipides avec le lait entier et le lait demi-écrémé. Avec le lait entier, la tache de gras translucide est plus dense.

64 • CHAPITRE 17 - Quels sont les principaux constituants du lait ?

• Réponses aux questions 1. Le lait entier et le lait demi-écrémé contiennent des lipides. Le lait écrémé ne contient pas de matière grasse, donc pas de lipide.

2. Le lait entier contient davantage de lipides.

■■Activité 2 Comment identifier des protéines dans le lait ? • Matériel Deux béchers - Un agitateur - Des tubes à essai - Plusieurs pipettes Des gants, une blouse et des lunettes de protection - Matériel de filtration : filtre, porte-filtre, potence

• Produits Lait entier Acide éthanoïque concentré Soude à 5 mol/L Solution de sulfate de cuivre à 5 × 10 –2 mol/L

Expérience 1 L’ajout d’acide éthanoïque, suivi d’une filtration, permet de séparer le caillé et le petit-lait.

Expérience 2 Le test du biuret (addition de sulfate de cuivre et de soude) permet de mettre en évidence des protéines.

• Réponse aux questions Le test du biuret montre qu’il y a beaucoup de protéines dans le caillé, et peu dans le petit lait.

■■Activité 3 Comment identifier des glucides dans le lait ? • Matériel Un bécher - Trois tubes à essai - Un bain-marie - Une spatule

• Produits Lait - Caillé - Petit-lait - Liqueur de Fehling

Expérience La réaction à la liqueur de Fehling est délicate à mettre en œuvre. Il vaut mieux la réaliser avec un bain-marie plutôt qu’avec un bec bunsen.

• Réponse à la question Le lait et le petit-lait contiennent des glucides.

■■Activité 4 Comment identifier l’eau et quelques sels minéraux dans le lait ? • Matériel Une soucoupe - Pipettes - Deux tubes à essai

• Produits Lait - Petit-lait - Sulfate de cuivre anhydre Solution d’oxalate d’ammonium - Solution de nitrate d’argent

Expérience 1 Les élèves connaissent le test de l’eau avec le sulfate de cuivre anhydre (test vu en classe de seconde et au collège).

Expérience 2 Les ions calcium se retrouvent dans le caillé, associés aux protéines. Il peut être difficile d’observer un précipité blanc d’oxalate de calcium dans le lait qui a la même couleur, contrairement au petit-lait qui est moins blanc. CHAPITRE 17 - Quels sont les principaux constituants du lait ? • 65

• Réponses aux questions Expérience 1

1. Le lait contient de l’eau. Expérience 2

2. Il y a peu d’ions calcium dans le petit-lait. Un précipité blanc de chlorure d’argent montre que le petit-lait contient des ions chlorure.

3. Le lait est une source de lipides, de protéines, de glucides, de sels minéraux. C’est un aliment complet.

Les documents ■■Nomenclature • Réponse aux questions : 1. CH3 — CH2 —OH  Cette molécule possède un groupe – OH : c’est un alcool. De plus elle contient deux atomes de carbone : c’est l’éthanol. 4

3

O

1 I

2

2. CH3 — CH2 —CH2 — C

U OH

Cette molécule possède un groupement COOH : c’est un acide carboxylique. De plus elle comporte 4 atomes de carbone : c’est l’acide butanoïque.

O || 3. H — C —CH2 — CH2 — CH3 Cette molécule possède un groupement COH : c’est un aldéhyde. 1 2 3 4 De plus elle comporte 4 atomes de carbone : c’est le butanal. 1

2



|| O

3

4

4. CH3 — C — CH2 — CH3

 ette molécule comporte un groupement CO : c’est une cétone. C De plus elle comporte 4 atomes de carbone : c’est le butan-2-one.

Les exercices Tester ses connaissances

Tester ses capacités

Q.C.M. 1 : B ;  2 : B ;  3 : C ;  4 : A et C ;  5 : B.

1. Composés organiques dans le lait a. On a identifié les aldéhydes et les cétones. b. Le lait contient des aldéhydes.

2. Nomenclature a. Molécules

1

2

3

4

5

Noms

Ethanal

Ethane

Propanone

Ethanol

Acide éthanoïque

b. Noms des composés

A : butanone

B : propan-2-ol

C : acide propanoïque

Formules semidéveloppées

CH3 — C — CH2 — CH3 || O

OH  | CH3 — CH – CH3

O I CH3 — CH2 — C U OH

66 • CHAPITRE 17 - Quels sont les principaux constituants du lait ?

D : butanal

O I CH3 — CH2 — CH2 — C U H

3. De la formule brute à la formule semi-développée Noms des composés

Méthanol

Propanal

Propanone

Acide butanoïque

Formules brutes

CH4O

C3H6O

C3H6O

C4H8O2

Formules semidéveloppées

CH3 —OH

CH3 — C — CH3 || O

O I CH3 — CH2 — CH2 — C U OH

O I CH3 — CH2 — C U H

Appliquer le cours

Molécule 2 :

4. Les groupes fonctionnels a : fonction alcool ; b : fonction aldéhyde ; c : fonction cétone ; d : acide carboxylique.

O I CH3 — CH2 — CH2 — C U OH

Molécule 3 :

5. Modèles moléculaires

CH3 — CH2 —CH2 — CH2 — OH

a. Molécule 1 :

O I CH3 — CH2 — CH2 — C U H

b. Molécule 1 : fonction aldéhyde ; molécule 2 : fonction acide carboxylique ; molécule 3 : fonction alcool. c. Molécule 1 : butanal ; molécule 2 : acide butanoïque ; molécule 3 : butan-1-ol.

6. Nomenclature des alcools Nom du composé Méthanol

Propan-2-ol

Pentan-2-ol

Butan-1-ol

Pentan-3-ol

Formule développée

Formule semi-développée

H  | H — C —OH  | H

CH3 — OH

H H H  |  |  | H—C—C—C—H  |  |  | H O H  | H

CH3 — CH — CH3  | OH

H H H H H  |  |  |  |  | H—C—C—C—C—C—H  |  |  |  |  | H O H H H  | H

CH3 — CH — CH2 — CH2 — CH3  | OH

H H H H  |  |  |  | H — C — C — C — C —O — H  |  |  |  | H H H H

CH3 — CH2 — CH2 — CH2 —OH

H H H H H  |  |  |  |  | H—C—C—C—C—C—H  |  |  |  |  | H H O H H  | H

CH3 — CH2 — CH — CH2 — CH3  | OH

CHAPITRE 17 - Quels sont les principaux constituants du lait ? • 67

7. Aldéhydes et cétones a. Formules développées des composés :

- de l’acide méthanoïque : H — C — OH || O

Molécule 1 : O I H—C U H

- de l’acide éthanoïque : CH3 — C — OH || O

Molécule 2 : H O  | I H—C—C U  | H H

- de l’acide propanoïque : O I CH3 — CH2 — C U OH

Molécule 3 : CH3 — C — CH3 || O

- de l’acide butanoïque :

O I CH3 — CH2 —CH2 — C U OH

Molécule 4 : CH3 — CH2 — C — CH3 || O

b. Groupe fonctionnel dans chaque molécule : Molécule 1 : fonction aldéhyde O I H—C U H Molécule 2 : fonction aldéhyde H O  | I H—C—C U  | H H Molécule 3 : fonction cétone CH3 — C — CH3 || O Molécule 4 : fonction cétone CH3 — CH2 — C — CH3 || O

c. Les molécules 1 et 2 sont des aldéhydes ; les molécules 3 et 4 sont des cétones. d. Nom des composés : molécule 1 : méthanal ; molécule 2 : éthanal ; molécule 3 : propanone ; molécule 4 : butan –2-one.

8. Acides carboxyliques O I

a. Le groupement — C boxylique.

U OH

caractérise un acide car-

b. Formules semi-développées :

9. À vous de résoudre a. Le composé n’est pas un aldéhyde ni une cétone. b. Le composé est acide : c’est un acide carboxylique.

c. La formule semi-développée du composé est : CH3 — C — OH || O

d. C’est l’acide éthanoïque.

10. Digestion du lait a. Formules brutes : – du glucose : C6H12O6 ; –­ du galactose : C6H12O6. Le glucose et le galactose ont la même formule brute. b. La molécule de glucose et de galactose contiennent chacune un groupement aldéhyde et cinq groupements alcool : H U C=O | H — C — OH | HO — C — H | H — C — OH | H — C — OH | H — C — OH | H Glucose

H U C=O | H — C — OH | HO — C — H | HO — C — H | H — C — OH | H — C — OH | H Galactose

c. Une solution de galactose mélangée avec de la liqueur de Fehling et placée dans un bain-marie donne un précipité rouge-brique caractéristique des aldéhydes. Le galactose est un aldéhyde.

68 • CHAPITRE 17 - Quels sont les principaux constituants du lait ?

11. Dosage de l’acide lactique dans un lait a. On a prélevé et dosé un volume Vo = 20 mL de lait. b. La solution titrante est une solution de soude de concentration molaire Csoude = 0,05 mol/L. c. La phénolphtaléine est l’indicateur coloré utilisé lors du dosage. Il sert à déterminer le volume équivalent de soude lors du changement de couleur de la solution. d. À l’équivalence, la solution est de couleur rose. e. La concentration de l’acide lactique dans le lait se calcule avec la formule : Cm = Co . MAH . MAH est la masse molaire moléculaire de l’acide lactique : MAH = 90 g/mol, Co est la concentration molaire de l’acide lactique : C .V 0,05 × 6,7 Co =  soude E =  = 16,75 × 10–3 mol/L. 20 V0 Soit Cm = 16,75 × 90 = 1,5 g/L.

f. Le degré d’acidité du lait dosé est égal à 15 °D. g. Le lait dosé est frais, car son acidité est comprise entre 13 °D et 18 °D.

12. Chromatographie a. Une chromatographie sur couche mince est une technique qui permet de séparer et d’identifier les différentes espèces chimiques contenues dans une substance. b. Le lait contient comme glucide du lactose, car la tache correspondant au lait migre à la même hauteur que celle du lactose.

Exercices à caractère professionnel 13. Analyse de laits de consommation a. Le lait entier est le lait nº 2, car il contient davantage de lipides (matières grasses). Le lait demi-écrémé est le lait nº 1. Le lait écrémé est le lait nº 3, car il contient quelques traces de lipides (pratiquement pas de matières grasses). b. Le lactose est le glucose le plus abondant dans le lait. c. La caséine et l’albumine sont les principales protéines du lait.

14. Fabrication du fromage a. Les deux groupes fonctionnels présents dans la molécule de l’acide lactique sont : fonction alcool H OH O  |  | I H—C—C—C U  |  | H H OH

fonction acide carboxylique

b. L’équation chimique équilibrée s’écrit : C12H22O11 + H2O → 4 CH3 – CHOH – COOH

c. M(lactose) = 12 × 12 + 22 + 11 × 16 = 342 g/mol. d.

50 = 0,15 mol. 342

Un litre de lait contient 0,15 mol de lactose.

e. Comme 1 % de lactose se décompose en acide lactique, donc 0,15 × 10 –2 mol de lactose se transforme en acide lactique. La quantité d’acide lactique formée est : n = 0,15 × 10 –2 × 4 = 0,6 × 10–2 mol. La masse d’acide lactique obtenue est donc : m = n.M(acide lactique) = 0,6 × 10 –2 × 90 = 0,54 g.

15. Conservation du lait a. Laissé à l’air libre, le pH du lait diminue, car le lactose se transforme en acide lactique.

b. Le lait devient acide et caille. c. Le degré Dornic de ce lait est égal à 25 °D. À la température ambiante, ce lait est liquide. Si on le chauffe, il caille.

16. Fromage et yaourt (Situation problème) Quand le lait caille, il se sépare en deux parties : le lactosérum (petit-lait) et le caillé. Le caillé est constitué des lipides du lait et de la grande partie des protéines initialement contenues dans le lait. L’acidification du lait par un acide n’est pas la seule façon d’obtenir le caillé. La texture du lait coagulé dépend du type de coagulation. - Dans la fabrication du yaourt, l’apport de bactéries vivantes (Streptococcus thermophilus et Lactobacillus bulgaricus) dans le lait transforme le sucre du lait, le lactose, en acide lactique. Cette acidification, lente et régulière, provoque la coagulation des protéines en un gel ferme donnant la texture lisse et aromatisée des yaourts. - Le caillé peut être aussi obtenu par la coagulation du lait par les enzymes de la présure. Une fois égoutté, il donne le fromage blanc. Le caillé, pressé et affiné, permet aussi la fabrication de fromages (emmental, gruyère …) - Ajoutées au lait, les bactéries lactiques et la présure le transforment en caillé de camembert. Deux types de coagulation sont conjuguées : la coagulation enzymatique (due à la présure), la coagulation acide (due aux bactéries). Ainsi, il y a une grande diversité de produits possibles à partir du même produit « lait ».

CHAPITRE 17 - Quels sont les principaux constituants du lait ? • 69

18 Comment fabriquer un détergent ? Quel est son rôle ? HS6

Les activités ■■Activité 1 Comment fabriquer un savon ? • Matériel - Ballon 250 cm3 ; chauffe-ballon ; réfrigérant à eau ; support élévateur ; - Verre à pied ; filtre Büchner ; trompe à eau ; - Support, noix de serrage (2), pinces (2)

• Produits - Huile alimentaire ; solution de soude très concentrée ; éthanol ; - Solution de chlorure de sodium.

Expériences L’expérience est réalisée par le professeur sous la hotte. Elle dure une bonne vingtaine de minutes et peut être mise en marche avant le cours de façon à pouvoir récupérer le savon durant la leçon.

• Réponse aux questions 1. Le savon précipite lors du relargage, lorsqu’on verse le mélange réactionnel dans une solution concentrée de chlorure de sodium.

2. L’équation de saponification est : R — COO — CH2 | R — COO — CH   +   3(Na+ + HO–)     →    3(R — COO– + Na+) | R — COO — CH2

+

CH2OH — CHOH — CH2OH

ester d’acide gras +

+

alcool

soude

savon

■■Activité 2 Quel est le rôle d’un détergent ? • Matériel - Cylindre muni de trois fils ; dynamomètre très sensible ; support élévateur ; - Cristallisoirs (2) ; bouts de laine (2).

• Produits Liquide à vaisselle (teepol) ; huile.

• Réponse aux questions Expérience 1

1. Avec un anneau métallique de poids P = 0,05 N, et de diamètre D = 0,1 m, la valeur indiquée par le dynamomètre est de l’ordre de 0,1 N avec l’eau du robinet.

2. Avec de l’eau additionnée de liquide vaisselle, la valeur indiquée par le dynamomètre est de l’ordre de 0,07 N

3. Le détergent abaisse la tension superficielle de l’eau. Expérience 2

4. Les bouts de laine coulent dans le cristallisoir contenant de l’eau additionnée de détergent. Des gouttes d’huile apparaissent dans le cristallisoir ne contenant pas de détergent.

70 • CHAPITRE 18 - Comment fabriquer un détergent ? Quel est son rôle ?

5. En abaissant la tension superficielle de l’eau, le détergent pénètre à l’intérieur des fibres de la laine ou du tissu. Les molécules divisent les salissures grasses en les entourant et les dispersent dans l’eau de lavage.

Les documents ■■La pollution par les détergents • Réponse aux questions 1. Les trois composés essentiels des détergents modernes sont : - les agents tensioactifs qui diminuent la tension superficielle de l’eau ; - les agents tampons qui maintiennent la valeur du pH lors du lavage ; - les adoucissants ou adoucisseurs.

2. Ils réagissent avec les sels de calcium et de magnésium pour réduire la dureté des eaux de lavage. 3. L’eutrophisation, provoquée par des excès de phosphates, consiste en la prolifération d’algues qui appauvrissent l’eau en dioxygène dissous, empêchant la survie de la faune.

Les exercices Tester ses connaissances

4. Saponification des esters gras R — COO — CH2 | R — COO — CH + 3(Na++HO–) → 3(R — COO– + Na+) + CH2OH — CHOH — CH2OH | R — COO — CH2

Q.C.M. 1 : B ;  2 : C ;  3 : C ;  4 : C ;  5 : B ;  6 : B.

Tester ses capacités

Ester d’acide gras + soude →

1. La fonction ester a. Il existe trois fonctions ester dans chaque molé-

savon

+

alcool

5. Molécule tensioactive a.

cule.

b. Fonctions ester : R — COO — CH2 | R — COO — CH | R — COO — CH2

partie hydrophile

partie hydrophobe

b. Le mot «  hydrophobe » signifie « repoussé par l’eau » et le mot «hydrophile » signifie « attiré par l’eau »

2. L’éthanoate de propyle Sa formule brute est C5H10O2 et sa formule développée est : H O H H H ||  |  |  |  | H—C—C—O—C—C—C—H  |  |  |  |     H H H H

3. Hydrolyse de l’éthanoate d’éthyle a. La réaction d’estérification est la réaction inverse de la réaction d’hydrolyse.

b. La réaction d’hydrolyse de l’éthanoate d’éthyle est : CH3 – COO – C2H5 + H2O → CH3 – COOH + C2H5 – OH

6. Les pouvoirs des détergents a. Le pouvoir mouillant est le pouvoir des molécules de pénétrer entre les fibres de la laine ou du tissu. Le pouvoir émulsifiant est le pouvoir des molécules de diviser les salissures grasses. Le pouvoir dispersant est le pouvoir des molécules de disperser les salissures grasses dans l’eau de lavage. b. Pouvoir mouillant : les parties hydrophobes des molécules de détergent pénètrent dans les fibres et chassent l’air. On voit des bulles d’air s’élever à la surface du liquide. Pouvoir émulsifiant : les molécules de détergent encerclent les salissures et les pénètrent par leur partie hydrophobe.

CHAPITRE 18 - Comment fabriquer un détergent ? Quel est son rôle ? • 71

Pouvoir dispersant : les particules graisseuses détachées du tissu et enrobées d’une micelle sont maintenues en suspension dans l’eau de lavage et évacuées par l’eau de rinçage.

  (a)

(b)

(c)

molécules tensioactives :

eau

huile

coupe d’une micelle

7. Les pictogrammes de sécurité



c. Schéma d’une gouttelette d’huile entourée de

(d)

Le pictogramme (a) signifie que le produit est irritant Le pictogramme (b) signifie que le produit doit être conservé hors de portée des enfants. Le pictogramme (c) signifie qu’il faut éviter le contact du produit avec les yeux et qu’en cas de contact, il faut rincer abondamment avec de l’eau. Le pictogramme (d) signifie que le produit ne doit pas être ingéré. En cas d’ingestion, il faut consulter un médecin.

8. Dilution du liquide à vaisselle

12. Fabrication d’un savon a. La formule brute de la stéarine est C57H110O6 b. On reconnaît trois groupes fonctionnels « ester » dans la stéarine.

c. Le nom de la réaction qui permet d’obtenir un savon est la saponification.

d. L’équation s’écrit :

C17H35 — COO — CH2 | C17H35 — COO — CH + 3(Na+ + HO–) → 3(C17H35 — COO– + Na+) + | C17H35 — COO — CH2 stéarine  +

soude



savon

CH2OH | CHOH | CH2OH

+ glycérol

On prélève 1 mL de la solution de détergent que l’on introduit dans une fiole jaugée de 100 mL. On ajoute de l’eau distillée jusqu’au niveau du trait de jauge.

13. Le palmitate de sodium a. Le terme hydrophile signifie « attiré par l’eau » et

Appliquer le cours

b. Les groupes hydrophile et hydrophobe de cet

9. L’expérience du poivre magique

c. Le savon nettoie mieux les graisses d’un tissu que

a. Position des molécules tensioactives du liquide à

le terme hydrophobe signifie « repoussé par l’eau ». ion sont respectivement (COO–) et (C15H31).

couche monomoléculaire en surface

l’eau pure car, d’une part, les molécules de savon abaissent la tension superficielle de l’eau et augmentent donc le pouvoir mouillant et d’autre part, du fait de leur pouvoir émulsifiant, elles peuvent entourer les salissures de graisses et mieux les éliminer.

b. Les parties hydrophiles des molécules s’installent

14. L’oléine a. Le groupe fonctionnel présent dans la molécule

vaisselle à la surface de l’eau :

à la surface de l’eau et déplacent les particules de poivre qui sont rejetées vers la périphérie.

10. À vous de résoudre a. Le coefficient A’ est donné par la relation : T F −P = l 2π.D 0,083 − 0,066 A’ = = 45 × 10–3 N/m . 2π × 0,06 A’ =

b. On constate que le détergent abaisse le coefficient de tension superficielle de l’eau.

11. Formule d’un savon a. Les espèces ioniques obtenues en dissolvant ce savon dans l’eau sont : C17H33 – COO– et Na+. b. La partie hydrophile de l’anion provenant de ce savon est le groupement (COO–) et la partie hydrophobe, le radical C17H33.

d’oléine est le groupe fonctionnel « ester ». Il y a trois groupes fonctionnels dans la molécule. b. La réaction qui permet de fabriquer un savon à partir d’un corps gras est une saponification. c. La formule du savon provenant de la réaction entre l’oléine et la potasse (K+ + HO–) est : (C17H33 — COO– + K+)

Exercices à caractère professionnel 15. Les pouvoirs d’un détergent a. Le schéma (a) représente le pouvoir mouillant. C’est la partie hydrophile de la molécule tensioactive qui est orientée vers l’eau. La partie hydrophobe pénètre aisément dans le tissu. b. Le schéma (b) représente le pouvoir émulsifiant. La partie hydrophobe de la molécule tensioactive pénètre aisément la particule de graisse.

72 • CHAPITRE 18 - Comment fabriquer un détergent ? Quel est son rôle ?

c. le schéma (c) représente le pouvoir dispersant. La boule de molécules tensioactives entourant chaque salissure grasse est appelée une micelle.

16. Fabrication industrielle a. Une mole de palmitine nécessite trois moles d’hydroxyde de sodium pour former trois moles de palmitate de sodium et une mole de glycérol. a. La masse de palmitine contenue dans les 916 kg 44 d’huile de palme (au kg près) est : 916 × = 403 kg. 100 b. La masse molaire de la palmitine est : 51 M(C) + 98 M(H) + 6 M(O) = 806 g/mol. c. La quantité de matière (en moles) de palmitine est : 403000 = 500 moles. 806 d. La quantité de matière (en moles) de savon obtenu est : 3 × 500 = 1 500 moles. e. La masse molaire du palmitate de sodium est : 16 M(C) + 31 M(H) + 2 M(O) + M (Na) = 278 g/mol. La masse de savon obtenu est 278 × 1 500 = 417 000 g = 417 kg.

17. Détergent commercial a. L’axe vertical est gradué en millinewton par mètre (symbole m.N/m).

b. La valeur du coefficient A de tension superficielle pour l’eau pure est A = 72 m.N/m soit : A = 0,072 N/m. c. Le pouvoir mouillant du détergent n’augmente plus, à partir du pourcentage de 0,1 %. 0,1 d. Le pourcentage correspond à : × 5 = 0,005 L. 100 Il faut donc verser 5 mL de détergent dans 5 L d’eau.

18. Investigation L’eau d’Hépar est une eau minérale « dure », car elle contient une grande quantité d’ions calcium et magnésium. C’est une eau très calcaire. Les savons ne se dissolvent pas, ou très peu, dans une eau « dure ». Ils ne moussent pas. Le rôle d’un adoucissant est donc d’éliminer les ions calcium et magnésium pour éviter la formation de calcaire. Certaines personnes utilisent du vinaigre blanc dilué.

19 Comment fabriquer un arôme ou un parfum ? HS6

Les activités ■■Activité 1 Comment fabriquer un arôme à l’odeur de banane : l’éthanoate d’isoamyle ? • Matériel - Erlenmeyer ; plaque chauffante ; condenseur à air (long tube de verre) - Bain marie ; bécher ; ampoule à décanter ; languette de papier filtre - Supports (2), noix de serrage (2), pinces (1), tige pour ampoule à décanter (1).

• Produits - Acide éthanoïque ; alcool isoamylique (3-méthylbutan-1-ol) - Acide sulfurique concentré ; solution de chlorure de sodium.

Expérience Attention, les réactifs sont dangereux. L’expérience est réalisée par le professeur sous la hotte. Elle dure une bonne vingtaine de minutes et peut être mise en marche avant le cours de façon à pouvoir récupérer l’arôme de banane durant la leçon.

• Réponse aux questions 1. Le condenseur à air permet de récupérer les réactifs et les produits qui se vaporisent. 2. Le relargage permet de dissoudre l’acide éthanoïque et l’alcool isoamylique qui n’ont pas réagi dans la phase aqueuse et de bien séparer la phase organique qui contient l’ester.

3. La phase organique a une odeur de banane. CHAPITRE 19 - Comment fabriquer un arôme ou un parfum ? • 73

4. L’équation de la réaction chimique s’écrit :

O || H3C — C — OH

+ H — O — CH2 — C4H9 →



acide éthanoïque

+

O || H3C — C — O — CH2 — C4H9 

+

H2O

éthanoate d’isoamyle

+

eau

alcool isoamylique

5. Pour fabriquer un arôme de banane, on fait réagir de l’acide éthanoïque avec de l’alcool isoamylique.

■■Activité 2 Comment préparer un parfum à l’odeur de jasmin : l’éthanoate de benzyle ? • Matériel - Ballon de 250 mL ; chauffe-ballon ; réfrigérant à eau ; support élévateur - Bécher de 250 mL ; ampoule à décanter ; languette de papier filtre - Supports (2), noix (3), pinces (2), tige pour ampoule à décanter (1).

• Produits - Acide éthanoïque ; alcool benzylique ; acide sulfurique concentré ; - Grains de pierre ponce ; solution de chlorure de sodium ; - Solution de carbonate de calcium ; sulfate de sodium anhydre.

• Réponse aux questions 1. L’eau salée permet de dissoudre l’acide éthanoïque et l’alcool benzylique qui n’ont pas réagi. On peut ainsi séparer l’éthanoate de benzyle qui est beaucoup moins soluble dans l’eau salée. C’est le relargage.

2. Lorsque le dégagement de CO2 a cessé, on peut dire que le carbonate de calcium a neutralisé toute l’acidité du mélange réactionnel.

3. L’équation de la réaction chimique s’écrit :

O || H3C — C — OH

+ H — O — CH2 — C6H5 →



acide éthanoïque

+

alcool benzylique

O || H3C — C — O — CH2 — C6H5 

+

H2O

éthanoate de benzyle

+

eau

4. On peut fabriquer un parfum à l’odeur de jasmin en faisant réagir de l’acide éthanoïque avec de l’alcool benzylique.

Les documents ■■Les arômes et les goûts 1. Les quatre goûts fondamentaux sont : le sucré, le salé, l’acide et l’amer. 2. La vanilline est un arôme naturel. 3. L’éthylvanilline est un arôme artificiel.

74 • CHAPITRE 19 - Comment fabriquer un arôme ou un parfum ?

Les exercices Tester ses connaissances

c. H3C — COO — CH2 — CH3 : l’éthanoate d’éthyle : d. H3C — CH2 — COO — CH2 — CH3 : le propanoate

Q.C.M. 1 : C ;   2 : B ;  3 : C ;  4. B.

d’éthyle.

Tester ses capacités

Appliquer le cours

1. L’acide éthanoïque (ou acétique)

8. L’aspartame

a. La molécule d’acide éthanoïque renferme le groupement caractéristique acide carboxylique.

b.

O || H3C — C — OH

2. Le méthanoate d’éthyle a. la molécule renferme le groupement caractéristi-

Les groupes fonctionnels présents dans la molécule d’aspartame sont : a. le groupe fonctionnel acide carboxylique : — COOH b. le groupe fonctionnel cétone : — CO — c. le groupe fonctionnel ester : — COO — CH3.

9. La palmitine a. Les trois groupes caractéristiques ester sont :

que ester.

b.



C15H31 — COO — CH2 | C15H31 — COO — CH | C15H31 — COO — CH2

O || H — C — O — CH2 — CH3

3. L’éthanoate d’éthyle a. Cette réaction s’appelle une estérification. b. L’équation de cette réaction s’écrit :

b. On peut dire que la palmitine est un triester, car

H3C — COOH + H3C — CH2 — OH → H3C — COO — CH2 — CH3 + H2O

10. À vous de résoudre a. La formule semi-développée de l’acide méthanoï-

4. L’éthanoate d’isoamyle a. La formule semi-développée de l’acide correspon-

CH3 — CH — CH2 — CH2 — OH | CH3

que est H — COOH. La formule semi-développée de l’éthanol est : CH3 — CH2 — OH. b. L’équation de la réaction d’estérification s’écrit : H — COOH + CH3 — CH2 — OH → H — COO — CH2 — CH3 + H2O c. L’ester formé se nomme : le méthanoate d’éthyle. Sa formule brute est C3H6O2.

5. L’éthanoate de méthyle a. La formule semi-développée de l’acide carboxyli-

11. Fabrication du glycérol a. La molécule de glycérol contient trois groupe-

dant est : CH3 — COOH.

b. La formule semi-développée de l’alcool correspondant est :

que qui a servi à sa préparation est : CH3 — COOH. C’est l’acide éthanoïque. b. La formule semi-développée de l’alcool qui a servi à sa préparation est : CH3OH. C’est le méthanol.

6. Le méthanoate de méthyle Le méthanoate de méthyle a pour formule semidéveloppée : H — COO — CH3. Sa formule brute est : C2H4O2. Sa formule développée est : O H ||  | H—C—O—C—H      | H

elle possède trois fonctions ester.

ments alcool.

b. La molécule d’oléine contient trois groupements ester.

c. L’équation d’hydrolyse de l’oléine s’écrit :

H33C17 — COO — CH2 | H33C17 — COO — CH + 3 H2O → CH2OH — CHOH — CH2OH + 3 H33C17 — COOH | H33C17 — COO — CH2 Oléine + eau → glycérol + acide oléique

7. Nom des esters

12. Un ester à l’odeur de rhum a. Les réactifs d’une réaction d’estérification sont un

a. H — COO — CH3 : le méthanoate de méthyle b. H3C — COO — CH3 : l’éthanoate de méthyle

acide carboxylique et un alcool. On obtient un ester et de l’eau.

CHAPITRE 19 - Comment fabriquer un arôme ou un parfum ? • 75

b. Pour former le méthanoate d’éthyle, on doit utiliser de l’acide méthanoïque et de l’éthanol. c. L’équation de la réaction s’écrit : H — COOH + CH3 — CH2 — OH → H — COO — CH2 — CH3 + H2O

dans cette molécule sont le groupe alcool (a) et le groupe ester (b). b. L’acide parahydroxybenzoïque a pour formule semi-développée : O C

HO

13. Un ester au parfum de pomme a. La molécule possède le groupe caractéristique ester : O || CH3 — CH2 — CH2 — C — O — CH3

c. Le nom du réactif B est le propanol. Sa formule semi-développée est CH3-CH2-CH2-OH. d. L’équation de la réaction conduisant à la formation du parahydroxybenzoate de propyle est :

b. La famille organique du réactif B est la famille alcool. c. Le réactif A a pour formule semi-développée : CH3 — CH2 — CH2 — COOH. Son nom est l’acide butanoïque. Le réactif B a pour formule semi-développée : CH3 — OH. Son nom est le méthanol. d. L’équation de la réaction chimique est : CH3 — CH2 — CH2 — COOH + CH3 — OH → CH3 — CH2 — CH2 — COO — CH3 + H2O On obtient aussi de l’eau.

14. Un ester à l’arôme d’abricot a. La formule semi-développée de l’acide propanoïque est : CH3 — CH2 — COOH ; sa formule brute est : C3H6O2 b. Le groupement caractéristique est le groupement ester : O CH3 || | H3C — CH2 — C — O — CH2 — CH2 — CH — CH3

c. La formule semi-développée du composé A qui a réagi avec l’acide propanoïque est : CH3 — CH — CH2 — CH2 — OH | CH3 d. Sa molécule contient le groupe fonctionnel alcool. e. L’équation de la réaction entre l’acide propanoïque et le composé A est : CH3 – CH2 – COOH + CH3 – CH – CH2 – CH2 – OH





|

CH3

O C

HO

OH

  + CH3 —CH2 — CH2 —OH O

→ HO



C

O

CH2

CH2

CH3

  + H2O

Exercices à caractère professionnel 16. Fabrication d’un parfum : l’essence de Wintergreen (arbuste d’Amérique) a. C’est le groupe fonctionnel acide carboxylique de l’acide salicylique qui réagit avec le méthanol.

b. La réaction d’estérification s’écrit : HO — C6H4 — COOH + HO — CH3 → HO — C6H4 — COOCH3 + H2O

17. Fabrication d’un médicament : l’aspirine a. Les groupes fonctionnels qui existent dans la molécule d’acide salicylique sont : le groupe fonctionnel acide carboxylique et le groupe fonctionnel alcool. b. L’acide éthanoïque réagit avec le groupe fonctionnel alcool de l’acide salicilyque. c. La formule semi-développée de l’aspirine est : H3C — COO — C6H4 — COOH d. Il y a plusieurs millénaires, les Sumériens prenaient un ancêtre de l’aspirine.

18. Le polyéthylène téréphtalate (P.E.T.) a. Deux groupes fonctionnels acide carboxylique existent dans l’acide benzène-1,4-dicarboxylique.

b. Deux groupes fonctionnels alcool existent dans

→ H3C – CH2 – COO – CH2 – CH2 – CH – CH3 + H2O



OH

|

CH3

15. Des esters dans les cosmétiques a. Les groupes fonctionnels (a) et (b) qui se trouvent

l’éthane-1,2-diol.

c. L’équation d’estérification entre une molécule d’acide et une molécule d’alcool s’écrit : HO2C – C6H4 – CO2H + HO – CH2 – CH2 – OH → HO2C – C6H4 – COO – CH2 – CH2 – OH + H2O

76 • CHAPITRE 19 - Comment fabriquer un arôme ou un parfum ?

20 Comment dévier la lumière ? SL1 Les activités ■■Activité 1 Comment se réfléchit la lumière ? Comment se réfracte la lumière ? • Matériel  Un disque gradué ; une source de lumière avec une fente ou un laser muni d’une diapo avec tige de verre pour obtenir un faisceau plan (dessin ci-contre) Un miroir ; un demi-cylindre de verre ou de plexiglas.

laser

diapo faisceau plan

tige de verre ruban adhésif

• Réponse aux questions Expérience 1

1. La relation est : i = r. Expérience 2

2. Le rayon réfracté existe toujours si le rayon incident se propage de l’air dans le verre. 3. La valeur de l’angle limite dépend du matériau transparent, par exemple 42° pour le verre crown. 4. Le rayon lumineux se rapproche de la normale dans le milieu le plus réfringent (de plus grand indice).

■■Activité 2 Qu’est-ce que le phénomène de réflexion totale ? Comment déterminer l’angle limite de réfraction ? • Matériel  Un disque gradué ; une source de lumière avec une fente ou un laser muni d’une diapo avec tige de verre pour obtenir un faisceau plan Un demi-cylindre de verre ou de plexiglas ; un laser ; une tige de plexiglas coudée.

• Réponse aux questions  Expérience 1

1. On peut voir un rayon réfracté tant que l’angle d’incidence reste inférieur à l’angle limite (vu dans l’activité 1).

2. L’angle réfracté disparaît pour l’angle d’incidence égal à l’angle limite : on observe un rayon réfléchi très lumineux, d’où la dénomination « réflexion totale ».

3. Pour le plexiglas (n = 1,40) iL = 46°; pour le verre crown (n = 1,52) iL = 41°; pour l’eau (n = 1,33) iL = 49°.

Expérience 2

4. Le rayon lumineux sort de l’autre côté de la tige coudée, même pour des inclinaisons importantes du faisceau à l’entrée. Ceci est dû à la grande différence d’indice entre les deux milieux, l’air (n = 1) et le plexiglas (n = 1,40).

5. C’est le même phénomène que l’on observe dans les fontaines lumineuses et dans les fibres optiques : dans une fibre optique, le rayon lumineux ne peut se propager que si son inclinaison est faible car les indices du cœur et de la gaine sont très proches (voir DOC page 226 et Exercice 11).

CHAPITRE 20 - Comment dévier la lumière ? • 77

Les documents ■■Les fibres optiques dans les télécommunications • Réponse aux questions 1. Le diamètre du cœur est tel que :

1 6 mm  D  mm 100 100

2. Pour ncoeur = 1,500 et ngaine = 1,485 on a sin θ ≤ nc2 − nG2 soit θ inférieur ou égal à 12,2 °. La valeur maximale de l’inclinaison est de 12,2°.

3. Durée du trajet : t =

d 13500 ×103 = = 0,0675 s. V 2 ×108

Les exercices Tester ses connaissances épingles

Q.C.M. 1 : C ;  2 : A et C ;  3 : A et C ;  4 : B ;  5 : A et C.

5 mm

Tester ses capacités

DEL

œil

α lame de verre

1. Réflexion de la lumière Le rayon lumineux issu du laser arrive sur le miroir sous un angle d’incidence i = 58 ° et se réfléchit sous l’angle de réflexion r = 58°. Donc i = r.

2. Réfraction de la lumière

La translation D mesure environ 5 mm. b. La translation n’est pas proportionnelle à l’angle α.

a. i1 = angle d’incidence ; i2 : angle de réfraction. b. Tableau :

5. Prisme a. Construction :

i1 (°)

0

30

45

60

i2 (°)

0

19

28

35

39

sin i1

0

0,50

0,71

0,87

0,97

sin i2

0

0,33

0,47

0,57

0,63

1,52

1,51

1,53

1,54

sini1 sini2

75

sini1 n2 c. = sin i2 n1



n1 = 1 et n2 ≈ 1,52 , indice de réfraction du verre.

3. Du verre dans l’air a. L’angle réfracté i2 augmente quand l’angle d’incidence i1 augmente. b. Le rayon disparaît à partir de 40°, donc pour 38° < iL  40°.

4. Lame de verre a. Construction :

78 • CHAPITRE 20 - Comment dévier la lumière ?

D = 40°

DEL prisme œil

b. La déviation Dˆ = 40° .

6. Connexion d’une fibre optique a. La tension uE est alternative car produite par un GBF en sélectionnant un signal carré. Sa période T = 5 ms et sa fréquence f = 200 Hz. b. La fibre optique transmet le signal avec la même fréquence. c. La tension uD est redressée sur une alternance car l’émetteur est une diode (une DEL).

7. Lire un schéma a. Fibre à saut d’indice → 2 ; fibre à gradient d’indice → 1 (voir Doc page 226).  = CDE  = ... Le phénomène de réflexion b.  ABC = BCD totale subie par le rayon lumineux se propage tout au long de la fibre.

Appliquer le cours

sin iL =

nGaine nCoeur

=

1,536   soit iL = 85,9°. 1,540

c. Si i = 85,9° alors α = 90 – i = 4,1°. d. La seconde loi de Descartes donne : 1.sin θ = 1,540.sin α soit sin θ = 0,110 et θ = 6,3°.

e. À partir de l’ouverture numérique de la fibre, sin θ  0,1109 soit θ  6, 4°. La relation est bien vérifiée.

8. Réflexion sur un miroir

12. Capteur de position A. Étude du prisme En I, l’angle limite de réfraction est tel que : n 1 sin iL = 2 = soit iL = 41,1°. n1 1,52

A DEL

B

Comme i = 45°, il y a réflexion totale en I. miroir

9. À vous de résoudre a. L’angle d’incidence en I est de 45° car  ABC = 45°. b. En I, l’angle limite est tel que : sin iL =

nair nverre

=

1 = 0,658 soit iL = 41°. 1,5

Comme i1 > iL, le rayon lumineux ne se réfracte pas dans l’air ; il se reflète totalement avec un angle de 45°. Il subira une seconde réflexion totale en J, sur la face AC. c. Construction : B

I

air

verre

C A

J

10. Rayons réfractés air

30 °

22°

eau n = 1,33

30 °

19,5°

air

verre n = 1,5

30 °

verre n = 1,5

48,6° air

Exercices à caractère professionnel 11. Fibre optique a. Pour que le rayon lumineux se propage tout au long de la fibre, il faut qu’il subisse une réflexion totale en B. b. L’angle limite iL est tel que :

B. Étude du dispositif a. Tant que le prisme B C est immergé dans l’air, le rayon lumineux verre émis par la diode subit I J milieu une réflexion totale en ambiant I puis une seconde en A J et le détecteur reçoit le signal lumineux. b. Lorsque l’eau atteint le point I, l’angle limite de réfraction est tel que : n 1, 33 sin iL’ = 2 = soit iL’ = 61,0°.Le rayon se réfracte n1 1, 52 dans l’eau et le détecteur ne reçoit pas de lumière.

c. La hauteur de l’eau atteint : H0 + BC/4 = 80,5 cm. Tant que l’eau n’atteint pas 80,5 cm dans la cuve, le détecteur reçoit le signal lumineux. Dès que l’eau atteint le point I, le rayon lumineux n’atteint plus le détecteur. Celui-ci commande un circuit électronique…

13. Vision des objets (Démarche d’investigation) a. Le crayon brisé Appelons A l’extrémité du air crayon immergée dans l’eau. Cherchons où se trouve l’image du point A A’ eau donnée par la lame d’eau contenue dans le verre. Plusieurs rayons issus de A A convergent en A’, image de A. Le rayon vertical n’est pas dévié : A’ se trouve sur cette verticale. On constate que A’ est plus proche de la surface libre que A. Étudions le rayon lumineux issu de l’extrémité A immergée du crayon. Traçons son parcours dans l’eau puis dans l’air, en direction de l’œil de l’observateur.

CHAPITRE 20 - Comment dévier la lumière ? • 79

observateur L’observateur voit en fait le point A’ dans le prolongement de OI. La partie immergée du crayon est vue selon le pointillé qui n’est plus dans la même direction que la partie émergée : le crayon semble brisé.

O

air

B

I

A’

b. Le chasseur De la même façon, le chasseur voit le poisson sous un angle erroné et l’imagine plus près de la surface qu’il ne l’est en réalité : sa flèche passera au-dessus du poisson. Pour éviter ce phénomène, le chasseur doit se tenir à la verticale du poisson : tous les angles sont alors nuls.

eau A

21 Comment transmettre un son ? SL2/SL3 Les activités ■■Activité 1 À quelle condition un son se propage-t-il ? • Matériel Cloche à vide ; support ; pompe à vide. Téléphone portable ; pressiomètre ; sonomètre.

• Réponse aux questions 1. Le son s’entend de moins en moins bien lorsque le vide est de plus en plus poussé. Lorsque l’on fait le vide sous la cloche, la densité de l’air diminue donc le son se propage de plus en plus difficilement.

2. Dans cet exemple, c’est l’air qui est l’élément nécessaire à la propagation du son. En règle générale, il faut un milieu matériel pour que la propagation du son soit possible.

■■Activité 2 Comment déterminer la vitesse et la longueur d’onde d’un son ? • Matériel Deux microphones ; un oscilloscope ; une règle. Un haut-parleur ; un GBF.

• Réponse aux questions 1. Pour chaque fréquence, les distances λ1 et λ2 sont égales. 2. Lorsque la fréquence augmente, la longueur d’onde λ diminue. λ est constant quelle que soit la position du microphone M2 par rapport à M1. T λ 4. L’unité du rapport est le mètre par seconde (m/s). Ce rapport représente donc la vitesse du son T dans l’air.

3.

5. Un son se propage dans un milieu matériel sous la forme d’une onde périodique. Dans l’air, le son se propage à une vitesse proche de 340 m/s.

■■Activité 3 Comment transmettre un son par l’intermédiaire de la lumière ? • Matériel Ensemble « Téléphone par fibre optique »

• Réponse aux questions 1. - Le microphone émetteur M1 convertit l’onde sonore en onde électrique.

80 • CHAPITRE 21 - Comment transmettre un son ?

- Le module pré-amplificateur amplifie le courant électrique. - Le convertisseur tension/infrarouge convertit le signal électrique en signal optique. - La fibre optique transmet le signal optique à la vitesse de la lumière dans le verre soit environ 200 000 km/s. - Le convertisseur infrarouge-tension convertit le signal lumineux en signal électrique. - L’amplificateur augmente la tension du signal électrique émis par le convertisseur. - Le haut-parleur convertit le signal électrique en onde sonore.

2. Oui, le son produit par le haut-parleur est le même que le son produit près de M1. 3. Oui une fibre optique peut faire partie d’une installation permettant la transmission de sons. 4. Une fibre optique transporte un message sous forme de signaux lumineux. Le convertisseur placé à la sortie de la fibre permet d’exploiter le message sous forme de son, d’image ou d’autres types de données.

5. Pour transmettre un son ou un autre type d’information sur de longues distances (communications intercontinentales), on peut utiliser des systèmes à fibres optiques. Comme dans cette expérience, les informations sont codées sous forme de signaux lumineux puis acheminées via une fibre optique déposée au fond de l’océan. La fibre optique offre l’avantage de transmettre à grande vitesse (200 000 km/s) et avec une grande fidélité le signal de départ.

Les documents • Réponse aux questions a. Pour émettre ou recevoir une onde radioélectrique, il faut disposer d’une antenne. b. Une onde radio se déplace dans le vide ou dans l’air à une célérité c ≈ 300 000 km/s. c. L’antenne réceptrice convertit l’onde radio en signal électrique qui alimente un haut-parleur.

Les exercices Tester ses connaissances Q.C.M. 1) B, C ; 2) A, B ; 3) B, C ; 4) A, C ; 5) A, B, C ; 6) A, B ; 7) B, C.

Tester ses capacités 1. Exploitation de l’activité 1 a. La mise sous vide du téléphone empêche la transmission du son.

b. La transmission du son nécessite un milieu matériel élastique. L’absence de molécules d’air dans le milieu ne permet donc pas le transport de l’onde mécanique comme dans l’air ou dans l’eau.

2. Vitesse du son dans l’air a. La durée t mise par l’onde sonore pour aller d’un microphone à l’autre est de 2,9 ms.

3. Exploitation de l’activité 2 Fréquence (Hz)

2 000

4 000

6 000

10 000

15 000

longueur d’onde (cm)

17

8,5

5,7

3,4

2,2

a. Le symbole de la longueur d’onde est λ (lambda). b. 18 16 14 12 10 8 6 4 2 0

Longueur d’onde (en cm)

0

2000 4000 6000 8000 10000 12000 14000 16000

Fréquence f (en Hz)

d 1 b. V =  = = 344 . La vitesse de propagation t 2,9 ×10 −3

c. La longueur d’onde est inversement proportion-

du son dans l’air est d’environ 344 m/s.

nelle à la fréquence. CHAPITRE 21 - Comment transmettre un son ? • 81

4. Transformation de formule Longueur d’onde λ

Célérité du son V

Période du son T

0,425 m

340 m/s

1,25 × 10–3 s

2m

5 000 m/s

4 × 10–4 s

0,1 cm

0,02 m/s

0,05 s

5. Expérience : loi de la réflexion d’ultrasons a. La réflexion se fait dans une seule direction, contrairement à la diffusion qui se produit dans toutes les directions. b. Comme dans la réflexion d’un rayon lumineux, on peut écrire : i = r.

6. Chaîne à compléter Compléter le schéma ci-dessous en utilisant les expressions suivantes : convertisseur signal lumineux → signal électrique ; convertisseur signal électrique → signal lumineux ; convertisseur signal électrique → son ; convertisseur son → signal électrique ; Partie émission

source sonore

convertisseur son signal électrique (microphone + amplificateur)

convertisseur signal électrique signal lumineux

convertisseur signal lumineux signal électrique

convertisseur signal électrique son (amplificateur + haut-parleur)

d. Les angles i et r sont égaux si la droite en pointillé est perpendiculaire au fond.

10. Un train arrive-t-il ? a. Le personnage pose son oreille sur le rail car le bruit provoqué par le déplacement du train sur les rails se propage dans le rail à une plus grande vitesse et avec une plus faible atténuation que dans l’air. d 2 b. t =  = = 0,38. L’onde sonore émise par le train V 5,2 met 0,38 seconde pour parcourir 2 km.

11. Corriger les affirmations si nécessaire. a. Faux : Une fibre optique transporte des informations sous forme de signaux lumineux.

b. Vrai : Dans une fibre optique, un rayon lumineux parcourt 200 000 kilomètres en 1 seconde.

c. Faux : Apollo est un câble sous marin contenant

fibre optique

Partie récepteur

Il faut donc utiliser τ1/2 et : 1500 ×13 ×10 −3 d2 =  = 97,5 m. 2 Le filet est donc à 19,5 mètres de la surface et à 97,5 mètres du fond. V 1500 c. λ =  = = 10 −2 m m. f 150000

récepteur

Appliquer le cours 7. La guerre des étoiles Oui cette affirmation est exacte. Dans l’espace, la pression est nulle. Par conséquent, les sons ne se propagent pas.

8. Avion de chasse d 1200 alors d = V. t ; V =  = 0,33 km/s ; t 3600 d = 0,33 × 4 = 1,32 km. Anaïs est donc environ à 1,32 km de l’avion. Si V = 

9. Sonar a. Un sonar est un émetteur-récepteur d’ondes sonores qui se réfléchissent sur des obstacles rencontrés dans le milieu environnant (fond marin, surface de l’eau, banc de poissons, …). En mesurant la durée entre l’émission et la réception d’une onde sonore et connaissant la vitesse de propagation de l’onde sonore dans le milieu, le sonar permet de déterminer la distance qui le sépare des obstacles. 1500 × 26 ×10 −3 b. d = V. t  d’où  d1 =  = 19,5 m. 2 (Attention, la distance parcourue est un aller-retour.

82 • CHAPITRE 21 - Comment transmettre un son ?

des fibres optiques et reliant Lannion en France à New York aux États-Unis. Il mesure environ 6 000 km. Par conséquent, une information transitant par ce câble traverse l’Atlanti6000 que en environ = 0,03 s. 200000 d. Vrai : Avant d’être transmis par l’intermédiaire d’une fibre optique, un son doit subir une conversion (en signal électrique puis en signal lumineux).

Exercices à caractère professionnel 12. Contrôle qualité par ultrasons d 2 × 4,7 ×10 −2 = = 18 × 10–6 s. V 5200 b. Pour diminuer la longueur d’onde du signal, l’opérateur doit augmenter la fréquence du signal

a. t = 2

13. Décalage sonore a. Le même coup de marteau provoque l’apparition d’une onde sonore dans la canalisation mais aussi dans l’eau qu’elle contient. Comme la vitesse du son est différente selon le milieu, les deux ondes n’arrivent pas en même temps au niveau du récepteur. d b. Si V =  alors d = V. t. Pour l’acier, Vacier = 5 200 m/s, t donc on peut écrire : d = 5 200 t1. c. De même, pour la propagation dans l’eau, Veau = 1 500 m/s, donc d = 1 500 t2.

d. Sur l’oscillogramme le décalage entre t1 et t2 est de 4,4 divisions soit ∆t = 4,4 × 0,5 = 2,2 ms. Par conséquent, t2 = t1 + 2,2 × 10–3. e. 5200 t1 = 1 500 t2 5 200 t1 = 1 500 (t1 + 2,2 × 10–3) (5200-1500) t1 = 1 500 × 2,2× 10–3 1500 × 2,2 ×10 −3 = 8,9× 10–4 s = 0,89 ms 3700 donc d = 5 200 × 8,9× 10–4 = 4,63 mètres. t1 = 

14. Thermocline (situation problème) a. La thermocline est, dans l’océan, la zone de transition thermique rapide entre les eaux superficielles (chaudes) et les eaux profondes (froides). b. Un sonar émet des sons et écoute leur écho. Si un bâtiment utilise seulement un sonar de coque, les sons émis se réfléchissent sur la thermocline. Ce sonar est donc « aveugle » pour tout ce qui se passe sous la thermocline. Afin de contourner ce problème, il faut utiliser des sonars immergés sous la thermocline.

22 Pourquoi les objets sont-ils colorés ? SL5 Les activités ■■Activité 1 Comment obtenir les couleurs de l’arc-en-ciel ? • Matériel  Une boîte à lumière blanche munie d’une fente ; un prisme ; un écran ; une lentille convergente.

Expérience On réalise la décomposition de la lumière blanche par réfraction à l’aide d’un prisme. La recomposition, de la lumière blanche est obtenue en interceptant le faisceau lumineux sortant du prisme à l’aide d’une lentille convergente.

• Réponses aux questions 1. On observe sur l’écran des lumières allant du rouge au violet : rouge, orange, jaune, vert, bleu, indigo, violet. La lumière de couleur violette est la plus déviée par le prisme.

2. Lorsqu’on intercale une lentille convergente entre le prisme et l’écran pour intercepter le faisceau lumineux sortant du prisme, on observe sur l’écran une tache blanche.

■■Activité 2 Comment produit-on des images colorées sur un écran ? • Matériel  Une boîte à lumière blanche munie de trois volets réfléchissants ; des filtres rouge, bleu et vert ; un écran.

Expérience On réalise la synthèse additive en superposant les lumières rouge, bleue et verte émises par une boîte à volets orientables munis de filtres.

• Réponses aux questions  1. La superposition des lumières rouge et bleue donne une lumière magenta ; celle des lumières rouge et verte donne une lumière jaune, et celle des lumières bleue et verte donne une lumière cyan. 2. La superposition des lumières rouge, verte et bleue donne de la lumière blanche. 3. La superposition des lumières colorées émises par des projecteurs munis de filtres donnent des ombres de différentes couleurs.

■■Activité 3 Comment produit-on des images colorées sur une affiche ? • Matériel  Un rétroprojecteur ; une lampe de lumière blanche. CHAPITRE 22 - Pourquoi les objets sont-ils colorés ? • 83

Des filtres jaune, magenta, cyan, rouge et bleu : une boîte d’optique noire fabriquée avec une boîte à chaussures ; un cavalier fabriqué avec un morceau de carton plié de couleur rouge

Expérience 1 On réalise la synthèse soustractive à l’aide d’un rétroprojecteur et de filtres jaune, magenta et cyan que l’on superpose sur le plateau du rétroprojecteur. Les élèves peuvent ainsi observer sur le tableau blanc les couleurs des lumières transmises par les superpositions des filtres.

Expérience 2 Pour l’étude des couleurs des objets, on éclaire un cavalier en carton de couleur rouge soit avec de la lumière blanche émise par une lampe torche, soit avec des lumières colorées (il suffit de mettre un filtre coloré devant la lampe). On interprète le résultat par l’absorption et la diffusion des lumières colorées du cavalier éclairé.

• Réponse aux questions  Expérience 1

1. Un filtre jaune transmet une lumière jaune ; un filtre magenta transmet une lumière magenta. 2. Quand on superpose un filtre jaune et un filtre magenta et que l’on éclaire en lumière blanche, la lumière transmise est rouge. Quand on superpose un filtre jaune et un filtre cyan, la lumière transmise est verte. Quand on superpose un filtre magenta et un filtre cyan, la lumière transmise est bleue. Quand on superpose les trois filtres magenta, cyan et jaune, aucune lumière n’est transmise (c’est le noir).

3. Cette synthèse des couleurs est qualifiée de soustractive car les filtres absorbent (enlèvent) des couleurs à la lumière blanche (composée de lumières colorées) pour obtenir une lumière d’une nouvelle couleur.

Expérience 2

4. Éclairé en lumière blanche, le cavalier rouge apparaît rouge ; éclairé en lumière jaune (composée de vert et de rouge), il apparaît rouge ; éclairé en lumière bleue, il semble noir.

5. La couleur d’un objet dépend de la couleur de la lumière qui l’éclaire.

Les exercices Tester ses connaissances

b. Pour recomposer la lumière blanche, on intercale une lentille convergente entre la prisme et l’écran. On observe alors une tache blanche sur l’écran.

Q.C.M. 1 : A et B ;  2 : B et C ;  3 : C ;  4 : B ;  5 : B ;  6 : A.

2. Analyse de la lumière avec un spectroscope a. Dans les spectres observés avec un spectroscope

Tester ses capacités 1. Analyse de la lumière avec un prisme a. Schéma de l’expérience : écran

boîte à lumière blanche

prisme

Sur l’écran, on observe un spectre continu de couleurs rouge, orange, jaune, verte, bleue, indigo et violette.

à réseau, on retrouve les lumières colorées qui composent le spectre de la lumière blanche : les lumières violette, indigo, bleue, verte, jaune, orange et rouge. b. Un spectroscope permet de décomposer la lumière.

3. Spectre de la lumière blanche Les lumières d’onde correspondent à : 0,455 µm = 455 nm → le bleu ; 0,580 µm = 580 nm → le jaune ; 0,720 µm = 720 nm → le rouge.

4. Superposition de lumières colorées Figure a : à l’intersection des faisceaux lumineux

84 • CHAPITRE 22 - Pourquoi les objets sont-ils colorés ?

rouge et vert, on observe une lumière jaune. Figure b : à l’intersection des faisceaux lumineux bleu et rouge, on observe une lumière magenta. Figure c : à l’intersection des faisceaux lumineux bleu et vert, on observe une lumière cyan.

5. Filtres monochrome Figure a : le filtre rouge absorbe le vert et le bleu ; la lumière est rouge à la sortie du filtre rouge. Figure b : le filtre magenta absorbe la lumière verte ; les lumières rouge et bleue traversent le filtre, donc la lumière est magenta à la sortie du filtre. Figure c : le filtre jaune absorbe la lumière bleue ; les lumières rouge et verte traversent le filtre, donc la lumière est jaune à la sortie du filtre. Figure c : le filtre cyan absorbe la lumière rouge ; les lumières bleue et verte traversent le filtre, donc la lumière est cyan à la sortie du filtre.

6. Superposition de filtres

2 : le filtre rouge a été utilisé car toutes les autres couleurs ont été absorbées par le filtre. Spectres du bas : 3 : le filtre vert a été utilisé car toutes les autres couleurs ont été absorbées par le filtre. 4 : le filtre cyan a été utilisé car le rouge et le jaune ont été absorbés par le filtre.

10. Couleur des objets Le chat bleu éclairé en lumière blanche apparaît bleu, il absorbe le rouge et le vert et diffuse le bleu. Le chat jaune éclairé en lumière blanche apparaît jaune, il absorbe le bleu et diffuse le vert et le rouge. Le chat noir éclairé en lumière blanche apparaît noir, il absorbe les trois couleurs rouge, verte et bleue et ne diffuse aucune couleur. Le chat blanc éclairé en lumière cyan (vert + bleu) apparaît cyan, il diffuse le vert et le bleu. Le chat bleu éclairé en lumière jaune (rouge + vert) apparaît noir, il absorbe le rouge et le vert et ne diffuse aucune couleur. Le chat blanc éclairé en lumière jaune (rouge + vert) apparaît jaune, il diffuse le rouge et le vert.

Figure a : le filtre magenta absorbe la lumière verte et diffuse les lumières rouge et bleue, le filtre jaune absorbe la lumière bleue et diffuse la lumière rouge donc à la sortie des deux filtres la lumière est rouge. Figure b : le filtre magenta absorbe la lumière verte et diffuse les lumières rouge et bleue, le filtre cyan absorbe la lumière rouge et diffuse la lumière bleue donc à la sortie des deux filtres la lumière est bleue. Figure c : le filtre jaune absorbe la lumière bleue et diffuse les lumières rouge et verte, le filtre cyan absorbe la lumière rouge et diffuse la lumière verte donc à la sortie des deux filtres la lumière est verte.

lumière bleue apparaît bleue car le vert est absorbé. La partie 2 (blanc) éclairée par des lumières bleue et rouge apparaît magenta. La partie 3 (vert) éclairée par une lumière rouge apparaît noire car elle absorbe le rouge. b. Il s’agit de la synthèse additive.

Appliquer le cours

12. Impression d’affiches a. Pour que le nom « 201-mobile » apparaisse en

7. À vous de résoudre a. 0,58 µm = 0,58 × 10–6 m. 3 ×108 c c donne f = =  La relation λ = λ f 0,58 ×10 −6 = 5,2 × 1014 Hz. b. C’est une radiation de couleur jaune.

8. Peinture a. En mélangeant des gouaches de couleurs : - cyan et magenta, on obtient du bleu ; - cyan et jaune, on obtient du vert ; - jaune et magenta, on obtient du rouge. b. En mélangeant des gouaches de couleurs cyan, magenta et jaune, on obtient du noir.

9. Filtres Spectres du haut : 1 : le filtre magenta a été utilisé car sa couleur complémentaire, le vert, a été absorbée par le filtre.

Exercices à caractère professionnel 11. Logo a. La partie 1 (cyan : vert + bleu) éclairée par une

bleu sur l’affiche, l’imprimeur doit utiliser les encres de couleurs cyan et magenta b. Pour que les ailes de la voiture apparaissent en noir, l’imprimeur doit utiliser les trois encres de couleurs jaune, cyan et magenta. Pour que le reste de la voiture apparaisse en rouge sur l’affiche, il doit utiliser les encres de couleurs jaune et magenta. b. C’est la synthèse soustractive.

13. Écran et couleur a. Les pastilles sont de couleurs rouge, verte et bleue. b. Pour obtenir un point rouge à l’écran, seules les pastilles rouges éclairent, les pastilles vertes et bleues étant éteintes. Pour obtenir un point vert à l’écran, il faut que les pastilles vertes éclairent, les pastilles rouges et bleues soient éteintes.

CHAPITRE 22 - Pourquoi les objets sont-ils colorés ? • 85

c. Lorsque aucun des trois luminophores n’est

14. Lessive (Démarche d’investigation)

éclairé, on obtient du noir. d. Il s’agit de la synthèse additive des lumières. e. 1 024 × 768 = 786 432 pixels ; 786 432 × 3 = 2 359 296 luminophores. L’écran comporte 786 432 pixels, soit 2 359 296 luminophores

En vieillissant, le linge jaunit. Les fabricants de lessive mettent une teinture bleutée dans leurs détergents pour masquer ce défaut. Le linge, ainsi, diffuse de la lumière jaune et bleue. Ces deux couleurs sont complémentaires, et donnent de la lumière blanche en s’additionnant.

23 Comment fonctionne un haut-parleur ? SL6 Les activités ■■Activité 1 Quelles sont les caractéristiques d’un champ magnétique d’un aimant ou d’une bobine ? Expérience 1

• Matériel Limaille de fer et plaques transparentes - Un aimant droit - Une aiguille aimantée - Un solénoïde - On peut remplacer la limaille de fer par une plaque transparente contenant de petits barreaux de fer ou placer la limaille entres deux plaques en plexiglas que l’on pose sur l’aimant. - On montre qu’une aiguille aimantée est un détecteur de champ magnétique. De plus, elle indique la direction et le sens du vecteur champ.

Expérience 2

• Matériel Un teslamètre et sa sonde - Un ampèremètre - Un rhéostat - Un générateur continu - Des cordons Tableau des résultats pour une bobine de 200 spires, 40 cm de longueur et 4 cm de diamètre : B est mesuré à l’intérieur du solénoïde. I (A)

0,52

1

2

2,9

4,1

5,1

B (mT)

0,35

0,7

1,3

1,9

2,7

3,4

• Réponse aux questions Expérience 1

1. Le spectre magnétique d’un aimant droit est semblable à celui d’un solénoïde, à l’extérieur. 2. En un point, la bobine exerce sur une aiguille aimantée une action mécanique à distance. Ce phénomène dépend de la position du point par rapport à la bobine : il est visible au voisinage de la bobine.

3. Les lignes de champ sortent du pôle nord de l’aimant et entrent au pôle sud. C’est la même chose pour le solénoïde : on dit qu’elles sortent de sa face nord pour entrer par sa face sud. Il possède donc deux pôles.

Expérience 2

4. Le champ magnétique n’est pas constant en tout point de l’espace entourant le solénoïde. 5. À l’intérieur du solénoïde la valeur du champ magnétique est constante sauf au voisinage des extrémités.

6. Lorsque l’intensité I du courant augmente, B augmente proportionnellement.

86 • CHAPITRE 23 - Comment fonctionne un haut-parleur ?

■■Activité 2 Quelle est l’action d’un champ magnétique sur un conducteur parcouru par un courant ? • Matériel Expérience 1 Un cadre conducteur mobile autour d’un axe - Un aimant en U - Un rhéostat Un interrupteur - Un générateur continu - Des cordons

Expérience 2 Un GTBF - Une chaise ou un tabouret - Deux aimants en U - Un support et une noix de serrage Une bobine soutenue par quatre élastiques - Un chronomètre - Des cordons

• Réponse aux questions Expérience 1

1. Le cadre subit une action mécanique. 2. Il faut qu’un de ses côtés soit immergé dans un champ magnétique et soit traversé par un courant. 3. La déviation • croît lorsque l’intensité du courant augmente. • est inversée lorsque le champ magnétique est inversé • est inversée lorsque le sens du courant est inversé.

Expérience 2



4. La bobine se déplace orthogonalement au fil et au vecteur B (champ radial). 5. La bobine se déplace, car chaque fil subit une action mécanique qui change de sens lorsque le sens du courant s’inverse.

6. On règle la fréquence des oscillations de la bobine en modifiant la fréquence de la tension délivrée par un générateur.

7. Pour faire vibrer une bobine traversée par un courant, il faut : - que cette bobine puisse effectuer des déplacements selon son axe et autour de sa position d’équilibre (élastiques ou autre système) ; - qu’elle soit parcourue par un courant alternatif ; - qu’elle soit immergée dans un champ magnétique dirigé suivant le rayon d’une spire (champ radial). C’est un couplage électromagnétique : on convertit de l’énergie électrique (courant électrique) en énergie mécanique (mouvement vibratoire).

Les documents ■■Les haut-parleurs électrodynamiques • Réponse aux questions 1. Les parties essentielles d’un haut-parleur sont : - un aimant annulaire permanent ; - une bobine légère placée dans l’entrefer de l’aimant ; - une membrane solidaire de la bobine pouvant se déplacer selon son axe autour d’une position d’équilibre.

2. La bande passante de ce tweeter va de 4 000 Hz à 30 000 Hz. 3. η = 

Pa Pe

=

12 = 0,24, soit 24 %. 50

4. Ce haut-parleur ne rend pas les sons aigus puisque sa bande passante est inférieure à 5 000 Hz. 5. Un filtre est un circuit électronique qui modifie un signal pour ne laisser passer que certaines fréquences : CHAPITRE 23 - Comment fonctionne un haut-parleur ? • 87

- pour les aigus, un passe-haut ne laisse passer que celles qui sont au-dessus d’une fréquence B déterminée (il atténue les autres) ; a. S d - pour les graves, un passe-bas atténue les aigus ; – I + F - pour les mediums, un passe-bande ne laisse passer qu’une bande de fréquence.

N

e

Les exercices

+

N



F

Tester ses connaissances Q.C.M. 1 : A et C ;  2 : B et C ;  3 : A ;  4 : B et C ;  5 : A et C ;  6 : A ; 7 : B et C ;  8 : A et B (Pour l’item C, c’est vrai si la longueur totale correspond à la longueur de la partie soumise au champ magnétique, mais ce n’est pas vrai en général) ;  9 : A, B et C.



d

b.

S N

+

d

c.

F

I e

B

S

4. Principe du haut-parleur a. Le schéma complété : aimant permanent

Tester ses capacités

pôles

bobine

S

1. Champ magnétique dans une bobine

face nord

N

S

bâti

face sud

N

S

I

N

S B

A

membrane souple

dôme

P

b. Propositions corrigées :

I

I

- la membrane vibre avec la même fréquence que celle de la tension d’entrée de la bobine. - La force électromagnétique produite par le champ magnétique de l’aimant permanent fait vibrer la bobine solidaire de la membrane. - La bobine oscille parce qu’elle est alimentée par un courant périodique et alternatif.

2. Influence de l’intensité du courant a. B = f (I) 4 3,5 3 2,5 2 1,5 1 0,5 0

B

e

I

B (mT)

5. Différencier les haut-parleurs a. 15 000 Hz : aigu ; 700 Hz : médium ; 100 Hz : grave. b. Tweeter : 3e courbe (verte) ; médium : 2e courbe (rouge) ; boomer : 1re courbe (violette). c. Humain : médium ; violon : tweeter ; basse : boomer. 0

1

2

3

4

5

6

I (A)

b. On en déduit B = 0,67I avec B en mT et I en A.

Appliquer le cours

B est proportionnel à I.

6. Bobines longues a. Les lignes de champ

3. Déplacement d’une tige conductrice

a.

b.

N

S N

B +

S



+

N

F e



Id

F

N

e



a.

B

d

b.

c.

S c.

Fun haut-parleur ? 88 • CHAPITRE 23 - Comment fonctionne I e

B

S

B I

d

I

b. Les pôles et le sens du courant

S N

+

I

I

S

d.

N S I

B N I

I

7. Solénoïde a. Face 2 : nord ; face 1 : sud.

b. Ce choix ne permet pas d’écouter dans de bonnes conditions les sons graves.

4 π × 10 −7 × N .I 4π ×10 −7 × 800 ×1,2 b. B =  =  l 0,40 = 3,0 × 10 –3 T. c. Si I’ = 3I alors B’ = 3B = 9 × 10 –3 T. d. Avec un courant alternatif de période 20 ms, toutes les 10 ms, les faces sud et nord s’inversent.

8. Aimant en U a.

B

b.

I S

F

F

c.

I N

N

B S

F S

N

B

I

G

= 80 × 10 –3 × 8 × 5 × 10–2 = 3,2 × 10–2 N. c. Si on inverse les polari- M tés du générateur, les sens du courant et de la force électromagnétique sont inversés.

K B

l = 5 cm F

I

tige conductrice

2 000 Hz à 6 000 Hz. b. Ce haut-parleur ne convient pas pour transmettre, avec un niveau d’intensité sonore convenable, des fréquences très aiguës ou très graves.

13. Installation d’un autoradio (Démarche d’investigation) Pour obtenir Zeq = 16 Ω, on effectue le branchement ci-contre.

Exercices à caractère professionnel 10. Que choisir ? a. La plus large bande de fréquence est [1 000 Hz ; 18 000 Hz] ; c’est celle du modèle Phonix.

intense que dans une direction perpendiculaire à l’axe. b. Pour 6 kHz, on a 70 dB dans l’axe de ce hautparleur. c. Avec 8 kHz à 60° de l’axe, on obtient 40 dB. d. Ce haut-parleur n’émet pas de la même façon dans toutes les directions, car les courbes enregistrées ne sont pas des demi-cercles. Pour 10 kHz, on a un niveau d’intensité sonore supérieur à 60 dB dans une région faisant 10° de part et d’autre de son axe (un secteur de 20° environ).

12. Bande passante a. Pour L > 70 dB, on a une bande passante qui va de

9. À vous de résoudre : Rails de Laplace a. Schéma ci-contre. b. F = B.I.L.sinα

11. Directivité : réponse angulaire a. Dans l’axe de ce haut-parleur le son est plus

Z1 = Z2 = 16 + 16 = 32 Ω. 1 1 1 1 On a = + = Z eq 32 32 16 soit Zeq = 16 Ω.

16Ω 16Ω

16Ω 16Ω

24 Comment fonctionne un microphone ? SL6 Les activités ■■Activités 1 Comment produire une tension alternative aux bornes d’une bobine ? • Matériel Deux aimants en U - Un support et une noix de serrage - Une bobine – Une chaise ou un tabouret Quatre élastiques - Un oscilloscope ou système EXAO - Un chronomètre - Deux cordons

• Réponse aux questions 1. On observe une tension variable sur l’écran lorsque la bobine se déplace : pour qu’elle apparaisse, il faut que la bobine soit en mouvement et coupe le champ magnétique radial produit par les deux pôles nord des aimants accolés.

2. La fréquence des oscillations de la bobine est égale à celle de la tension déterminée à partir de son enregistrement. CHAPITRE 24 - Comment fonctionne un microphone ? • 89

3. En frappant légèrement sur la membrane du microphone, la bobine qui en est solidaire se met en mouvement : le système de rappel lui permet d’osciller.

4. Le microphone électrodynamique convertit un signal sonore en signal électrique selon le phénomène d’induction : une tension induite prend naissance aux bornes d’une bobine placée dans un champ magnétique et mise en vibrations par les ondes sonores. C’est la fonction inverse d’un autre transducteur électroacoustique : le haut-parleur.

■■Activité 2 Quel est le sens du courant induit ? • Matériel Une bobine - Deux DEL montées tête bêche - Un aimant droit - Deux cordons Le sens du courant dans le circuit est donné par la DEL qui s’éclaire. On en déduit les faces de la bobine en utilisant la règle simple qui est donnée.

• Réponse à la question Lorsqu’on approche d’une bobine le pôle nord d’un aimant, le courant qui se crée est tel que la bobine présente une face nord à l’aimant : l’effet de ce courant s’oppose à la cause qui lui a donné naissance. Lorsqu’on éloigne d’une bobine le pôle nord d’un aimant, le courant qui se crée est tel que la bobine présente une face sud à l’aimant : l’effet de ce courant s’oppose également à la cause qui lui a donné naissance.

Les documents ■■Les microphones • Réponse aux questions 1. La tension efficace U = σ . p = 5 × 0,8 = 4 mV. 2. Pour L = –40 dB la bande passante est de 30 Hz à 10 000 Hz. 3. Pour éliminer le maximum de sons provenant de l’arrière du microphone, on utilise un microphone à directivité unidirectionnelle.

Les exercices Tester ses connaissances

b. Si le sens du déplacement est inversé, le sens du courant induit l’est également.

Q.C.M.

2. Oscillations d’une bobine

1 : B ; 2 : A et C ; 3 : A ; 4 : B et C ; 5 : A et C.

a. On peut visualiser une tension aux bornes de la

Tester ses capacités

bobine si elle se déplace devant l’aimant en faisant varier le flux qu’elle embrasse.

1. Déplacement d’une bobine

b. La bobine présente une face sud lorsqu’elle des-

a. Schéma : déplacement de la bobine

N N

c. On obtient l’oscillogramme d’une tension alter-

S

S

I

cend vers le pôle sud de l’aimant ; une face nord lorsqu’elle remonte.

I

native : les oscillations de la bobine provoquent l’inversion des faces nord et sud qui apparaissent sur la bobine : le sens du courant induit qui la traverse commande cette inversion.

d. Avec 5 oscillations de la bobine par seconde, on obtiendra une tension de fréquence 5 Hz.

90 • CHAPITRE 24 - Comment fonctionne un microphone ?

3. Bande passante a. Courbe réponse en sensibilité : Niveau (dB)

bobine ; S : surface d’une spire (m2) ; B : valeur du  champ magnétique (T) ; θ : angle entre B et la nor male n à la surface. b. Φa = 102 × 3 × 10 –2 × 2 × 10–3 × 1 = 6 × 10–3 Wb. Φb = 6 × 10 –3 × cos 35° = 4,9 × 10 –3 Wb. Φc = 6 × 10 –3 × cos 90° = 0 Wb.

Fréquence (Hz)

10

0

7. Flux magnétique a. B = N.B.S.cosθ avec N : nombre de spires de la

100

1 000

10 000 100 000

-10 -20 -30 -40

8. Prise de voix a. 1) Omnidirectionnelle ; 2) bidirectionnelle ; 3) uni-

-50 -60 -70

directionnelle (cardioïde).

b.

b. La bande passante : 100 Hz à 10 000 Hz.

Appliquer le cours 4. Autrement dit Loi de Faraday

produisant

d’un groupe

d’un duo

unidirectionnelle

omnidirectionnelle

bidirec­ tionnelle

Période : T = 5 × 0,5 × 10 –3 = 2,5 × 10–3 s. U 6 ×10 −3 Tension efficace : U =  m ax = = 4,2 × 10 –3 V. 2 2 Fréquence : f = 400 Hz. b. La fréquence appartient à la bande passante du microphone : son fonctionnement est convenable. L −41 c. L = 20 log σ donne log σ = = = –2,05 20 20

qui fait circuler dans un circuit fermé

des effets (pôles, forces ...)

d’une voix

9. À vous de résoudre a. Amplitude : Umax = 3 × 2 × 10 –3 = 6 × 10–3 V.

Une variation induit une tension e de flux dans un circuit qui s’opposent à la cause qui leur a donné naissance

Prise Type de directivité

un courant induit

σ = 8,9 × 10 –3 V/Pa soit 8,9 mV/Pa.

Loi de Lenz

5. Tension induite a. Le schéma N

Pour U = 4,2 × 10 –3 V on a : p = 

S N

I

2

1 S

I

N

S

(90° ; –15 dB). l’axe frontal à l’axe transversal.

d. C’est un microphone de type unidirectionnel donc

6. Courant induit a. Il apparaît un courant induit dans la bobine 1, car l’éloignement de la bobine 2 ou la fermeture de l’interrupteur de la bobine 2 provoque une variation de flux dans la bobine 1. b. Schéma donnant le sens du courant induit dans les spires de la bobine 1 :

il ne capte pas les bruits provenant de l’arrière.

Exercices à caractère professionnel 11. Directivité en huit a. La directivité de chaque microphone est omnidirectionnelle.

a +



mA

b. À 90° de l’axe des microphones (sur l’axe de symé-

b

I

S

10. Diagramme de directivité a. La directivité est unidirectionnelle de type car-

c. Le niveau de sensibilité décroît lorsqu’on passe de

= 1 V.

2 ×10 −1

= 0,47 Pa.

b. (axe frontal ; 0 dB) – (30° ; –5 dB) – (60° ; –10 dB) –

b. Valeur moyenne de la tension induite : E = 

8,9 ×10 −3

dioïde.

déplacement

(450 − 250)×10 −3

4,2 ×10 −3

+ I

NS

déplacement de la bobine bobine 2

bobine 1

N S



mA

I I bobine 2

N N

S bobine 1

trie), il n’y aura pas de prise des sons émis par le percussionniste. c. Pour conserver la même directivité, on peut remplacer ce montage par un microphone ayant une directivité bidirectionnelle.

CHAPITRE 24 - Comment fonctionne un microphone ? • 91

12. Acousticien a. Schéma de la chaîne sonore depuis la scène jusqu’au public. microphone

amplificateur de tension

hautparleur

e. La directivité est unidirectionnelle.

14. Sensibilité a. L’intervalle correspond au niveau L dont la sensibilité est –57 dB ⩽ L ⩽ –52 dB.

b. –54 dB correspond à σ = 2 mV/Pa. Si p = 0,08 Pa, alors U = σ.p = 2 × 0,08 = 0,16 mV.

Une chaîne de transormation acoustique comprend au moins un microphone, un amplificateur et un haut-parleur.

15. Choisir l’impédance a. La fréquence de référence est de 1 kHz. b. Schéma : 120°

b. On peut considérer que la bande passante de la

60° 30°

150°

chaîne va de 50 Hz à 12 000 Hz.

c. Le public ne percevra pas les sons aigus. Pour

180°

améliorer la chaîne et rendre les aigus (les cymbales par exemple), il faut ajouter un tweeter.

0° 330°

210°

13. Notice technique

240°

a. C’est un microphone électrodynamique composé d’une bobine solidaire d’une membrane qui peut vibrer dans l’entrefer d’un aimant. b. On utilise le principe de l’induction électromagnétique pour expliquer son fonctionnement. c. Sa bande passante allant de 90 Hz à 13 000 Hz ne couvre pas le domaine des fréquences audibles par l’oreille humaine (30 Hz – 20 000 Hz).

d. σ = 1,7 mV/Pa ; p = 

90°

0,85 ×10 −3 1,7 ×10 −3

= 0,5 Pa.

270°

300°

c. Ce microphone réduit les sons parasites provenant de l’arrière parce qu’il est unidirectionnel.

d. On se placera sur la sortie basse impédance (Z = 200 Ω). Le niveau de sensibilité sera L = –73 dB et sa sensibilité σ est telle : L = 20 log   σ ; log   σ = –73/20 = –3,65. D’où σ = 103,65 σ = 0,22 mV/Pa.

16. Aurélie (Démarche d’investigation) Les minichaînes Hi-fi offrent aujourd’hui plus de performances que les imposants modèles du passé. Voici quelques éléments à connaître. Les éléments

Leur utilité

L’amplificateur

- Il permet d’amplifier les signaux électriques qu’il reçoit. - Une puissance de 2 × 25 W est suffisante pour une chambre (la puissance n’est pas synonyme de qualité) ; - Son impédance doit être voisine de l’impédance des haut-parleurs.

Les enceintes

Les haut-parleurs doivent restituer des sons de bonne qualité. La largeur de la bande passante et la réponse angulaire doivent être suffisantes.

Le Dolby

Permet une écoute plus claire et sans souffle.

Les connections : • Les prises « surround » • Les prises « jack » • Les prises USB

• Permettent de brancher des enceintes supplémentaires derrière l’auditeur pour recréer une ambiance. • Permettent de brancher par exemple un microphone ou un casque d’écoute baladeur audio … • Pour un ordinateur, une clé, …

La compatibilité WiFi

Offre une interconnexion des appareils sans fil en simplifiant la connectique

Le tuner

Détecte les ondes radio : PO (petites ondes) – GO (grandes ondes) – FM (modulation de fréquence). Le RDS (Radio Data System) permet d’afficher le nom et la fréquence de la station.

Les lecteurs multisupports

Très pratiques pour écouter les musiques stockées sur CD – carte SD – clé USB – iPod – iPhone – MP3 – MP4 - …

92 • CHAPITRE 24 - Comment fonctionne un microphone ?

La télécommande

Permet de sélectionner à distance le mode d’utilisation et d’effectuer différents réglages (volume, …).

La maintenance

ATTENTION RISQUE D’ÉLECTROCUTION-NE PAS OUVRIR ! ATTENTION : POUR RÉDUIRE LE RISQUE D’ÉLECTROCUTION, NE PAS RETIRER LE COUVERCLE NI LE FOND. CET APPAREIL NE CONTIENT AUCUNE PIÈCE RÉPARABLE PAR L’UTILISATEUR. CONSULTER LE PERSONNEL QUALIFIÉ POUR TOUTE RÉPARATION.

La mise au rebut

Pour la collecte des déchets : ne peut se déposer avec les déchets ménagers.

25 Comment une image est-elle captée par un système d’imagerie numérique ? SL7

Les activités ■■Activité 1  Comment évolue la caractéristique d’une photorésistance avec l’éclairement ? • Matériel Système exao ; capteur tension ; capteur intensité ; photorésistance Générateur de courant alternatif ; interrupteur ; luxmètre

• Réponse aux questions : 1. Les coefficients directeurs des droites représentent les valeurs des résistances de la photorésistance dans chacune des situations.

2. Lorsque l’éclairement de la photorésistance augmente, la résistance diminue. 3. Non, la résistance R et l’éclairement E ne sont pas des grandeurs proportionnelles.

■■Activité 2 Comment fonctionne une photodiode ? • Matériel Un générateur de courant continu 6V ; un ampèremètre Une photodiode ; une résistance de 10 kΩ.

• Réponse aux questions 1. Lorsque la photodiode est éclairée, l’intensité du courant inverse est proportionnelle à l’éclairement reçu.

2. Chaque photodiode du capteur fournit une intensité qui est fonction de l’éclairement qu’elle reçoit. Un système électronique mesure ces intensités et mémorise l’éclairement. Il reproduit alors sur l’écran l’image qui s’est formée sur le capteur.

■■Activité 3 Comment fonctionne un capteur photographique « CCD » ? • Matériel Un capteur CCD ; un stylo de couleur noire ou des écrans comme indiqués dans le sujet. CHAPITRE 25 - Comment une image est-elle captée par un système d’imagerie numérique ? • 93

• Réponse aux questions Expérience 1

1. Cette valeur dépend de la distance entre l’écran et le capteur, de la luminosité. On peut cependant dire que pour une couleur foncée, la tension est plus importante que pour le blanc.

2. La largeur d’un rectangle sur l’écran influe directement sur la largeur du pic correspondant sur l’oscillogramme.

Expérience 2

3. Une mire est constituée de plusieurs bandes de niveau de gris différents. Plus le niveau de gris est important, plus la hauteur du pic est importante.

Les documents ■■Comment obtenir une image en couleur avec un appareil photo numérique ? • Réponse aux questions 1. L’objectif permet de former une image nette sur le capteur. Le capteur photosensible convertit l’éclairement de chaque pixel de l’image en intensité électrique. Le système électronique connecté au capteur utilise les informations électriques données par le capteur pour reconstituer l’image.

2. Le rouge, le vert et le bleu sont les trois couleurs primaires. Aucune de ces trois couleurs n’est obtenue par addition des deux autres couleurs primaires.

3. La synthèse additive des couleurs consiste à combiner des lumières colorées afin d’obtenir une nouvelle couleur.

Les exercices Tester ses connaissances

b. On voit sur le graphique que plus la photodiode

Q.C.M.

est éclairée, plus la partie horizontale de la courbe est basse donc plus l’intensité négative est importante.

1 : A, B ;  2 : B ;  3 : A, C ;  4 : C.

2. Mesure d’éclairement

Tester ses capacités

a. Ce luxmètre indique 39 lux. b. L’unité de mesure est le lux (lx).

1. Caractéristiques d’une photodiode a. Voir schéma page suivante. Intensité (µA)

Intensité I1 dans l’obcurité (µA) Intensité I2 pour E = 180 lx (µA) Intensité I3 pour E = 1500 lx (µA)

60 20

–4

–2

que l’autre boîtier est l’afficheur.

3. Quel éclairement ?

40

Tension (V) –6

c. Le boîtier ayant le disque blanc est le capteur alors

Situation

Éclairement (lx)

Grand magasin

0,2

Obscurité

105

–40

Salle de classe

600

–60

Pleine lune

0,2

–80

Plein soleil

400

0 –20

2

–100

94 • CHAPITRE 25 - Comment une image est-elle captée par un système d’imagerie numérique ?

Appliquer le cours 4. Courant inverse d’une photodiode a. U = R. I d’où I1 = 

U 0,554 = = 5,54 × 10–6 A. R 10 4

U 1,35 = = 1,35 × 10–4 A. R 10 4 c. Lors de la deuxième mesure, l’intensité du courant fourni par la photodiode était beaucoup plus importante par conséquent, on peut en déduire que l’éclairement était plus important.

b. I2 = 

5. À vous de résoudre a. L’augmentation de l’éclairement fait augmenter la tension à vide de la photopile.

b. Pour un éclairement de 200 lux, la tension à vide U0 est de 270 mV.

c. Pour un éclairement de 0 à 20 lux, l’augmentation de la tension à vide est proportionnelle à l’éclairement.

6. Choisir une photodiode a. Pour chaque courbe, l’abscisse du maximum

b.

Mot binaire 10011010 code-barres

Mot binaire 11001001 code-barres

oscillogramme

oscillogramme

9. Découvrir des capteurs photosensibles Principe de fonctionnement d’un numériseur de document ou scanner Une source lumineuse suivie d’un miroir se déplace sous la plaque de verre sur laquelle est déposée le document à numériser. Le rayon lumineux se reflète sur le document puis par un jeu de miroirs arrive sur des capteurs CCD qui convertissent le rayonnement lumineux en information numérique. plaque de verre

représente la longueur d’onde à laquelle le composant est le plus sensible.

source lumineuse (mobile)

b. Pour réaliser un luxmètre pour rayonnement visible il faut utiliser la photodiode BPW21 car un rayonnement visible a une longueur d’onde comprise entre 380 et 780 nm. Pour réaliser un luxmètre pour rayonnement dans le proche infrarouge il faut utiliser la photodiode BPW34 et pour un rayonnement dans l’infrarouge plus lointain, il faut utiliser la photodiode BP104.

7. Capteur CCD a. Le code barre correspondant à l’oscillogramme est :

b. Le code binaire correspondant à l’oscillogramme est : 001010110.

c. Certains pics ont une valeur maximale plus importante que les autres, car le noir est plus intense que pour les autres.

8. Code-barres a. Allure de l’oscillogramme lié au code barre :

document à scanner

miroir (fixe) capteurs CCD

miroir se déplaçant avec la source lumineuse

Principe de fonctionnement d’un photocopieur Le photocopieur est constitué d’un cylindre métallique recouvert d’une couche d’un matériau ayant la particularité d’être électriquement résistant à l’état normal mais de devenir conducteur quand il est éclairé. La première opération consiste à charger positivement la couche de semi-conducteur. Ensuite l’image à reproduire est projetée via un objectif et des miroirs sur le tambour photosensible. Les zones claires de cette image rendent la surface semi-conductrice électriquement neutre. Ces zones ne sont donc plus chargées électriquement. L’étape suivante consiste à appliquer une poudre très fine de carbone noir chargé négativement (le toner). Celle-ci ne va adhérer au tambour que sur les zones qui sont encore chargées positivement (zones sombres de l’image). On fait ensuite passer une feuille de papier contre le tambour. La feuille est ensuite chauffée pour fixer définitivement la poudre de carbone qui fond et s’incruste dans les fibres du papier. Principe de fonctionnement d’un détecteur de présence photoélectrique Ce type de détecteur est composé d’un émetteur et

CHAPITRE 25 - Comment une image est-elle captée par un système d’imagerie numérique ? • 95

d’un récepteur d’infrarouges. La détection d’une personne ou d’un objet se fait par coupure ou variation du faisceau lumineux qui provoque la variation de la résistance d’une photorésistance. Principe de fonctionnement d’un allumeur de réverbère automatique. e– 7,5 V

15 V



11. Fourche optique a. Oscillogramme (1) de gauche : période 20 ms. Oscillogramme (2) de droite : période 10 ms.

b. Mesure (1) : nombre de boites à l’heure 3, 6 × 106 = 180000 180 000 boites/h. 20 Mesure (2) : nombre de boites à l’heure = 

= 

S

R1

e+

12. Domotique a. Le composant photosensible choisi par Arthur est

US

R2 U e+

3, 6 × 106 = 360 000 360000 boites/h. 10

une photorésistance. Dans ce montage comprenant un amplificateur opérationnel monté en comparateur, R1 représente une résistance réglable (pour le réglage de la sensibilité) et R2 une photorésistance. La tension sur l’entrée inverseuse de l’AO (e–) est constante. La tension sur l’entrée non inverseuse de l’AO (e+) dépend des valeurs des résistances de R1 et R2 : - si R2 > R1, Ue+ > Ue–  donc Us = +Usat - si R1 > R2, Ue+ < Ue–  donc Us = –Usat - si R2 = R1, Ue+ = Ue–  donc Us bascule de –Usat à +Usat ou de +Usat à –Usat. L’utilisation de la tension de sortie de l’AO par un système électronique permet de piloter l’allumage des réverbères.

Exercices à caractère professionnel 10. Détecteur de lumière a.

U (V)

2,5 2

fait varier la tension aux bornes d’une résistance. À partir d’une courbe d’étalonnage, il est donc possible de relier la tension mesurée et l’éclairement.

c. 12 11,5 11 10,5 10 9,5 9 8,5 8 7,5 7

UR1 (V)

0

500

1 000

1 500

2 000

2 500

E (lx)

d. Non ces deux grandeurs ne sont pas proportionnelles car la représentation graphique n’est pas une droite passant par l’origine. e. Il doit saisir une consigne de 9 V. f. Oui l’ordinateur donnera l’autorisation d’allumer car c’est lorsque que l’éclairement est faible que la tension UR1 est basse.

13. Démarche d’investigation : Comment réaliser un spectrophotomètre ? a. Une courbe d’étalonnage décrit l’évolution d’une

1,5 1 0,5 0

b. La variation de la résistance de la photorésistance

grandeur en fonction d’une autre.

b. Mickael prévoit d’utiliser : 0

100

200

300

400

500

600

700

800

longueur d’onde (nm)

b. La sensibilité maximale du récepteur correspond à une radiation de 600 nm.

c. Si la tension mesurée est 1,9 V, la longueur d’onde est de 550 nm donc la couleur du rayonnement est entre le vert et le jaune.

- - - -

une photorésistance, un ohmmètre, un tube à essai, une lampe. c. Ce montage doit être placé dans une boîte noire afin que les mesures ne soient pas perturbées par l’éclairement «normal » de la salle de manipulation.

96 • CHAPITRE 25 - Comment une image est-elle captée par un système d’imagerie numérique ?